Você está na página 1de 145

SPECIAL PATHOLOGY

KMU PAST PAPERS SOLVED SEQS

SALMAN KHAN | REHMAN MEDICAL COLLEGE


No part of this publication is to be reproduced
for purposes of sale, you may transmit by any
form or by any means, electronic, mechanical,
photocopying, recording or otherwise, without
the prior permission of the author, this eBook is
specifically for educational purposes only.
Contents 1

CARDIOVASCULAR SYSTEM 2

HEMATOPOIETIC AND LYMPHOID SYSTEMS 29

LUNG 50

GASTROINTESTINAL TRACT, LIVER AND PANCREAS 64

KIDNEY & MALE - FEMALE


GENITAL SYSTEM AND LOWER URINARY TRACT 91

ENDOCRINE SYSTEM, BREAST, BONES, JOINTS AND CNS 119

References
1. Robbins Basic Pathology (9th Edition)
2. Goljan - Rapid Review Pathology (fourth edition)
3. Harsh Mohan Textbook of Pathology (Seventh Edition)

KMU Past Papers Solved SEQs


Cardiovascular System 2

Q-1
a) Define Atherosclerosis. 2
b) Enumerate the risk factors leading to atherosclerosis. 4
c) What is the role of lipids in atherosclerosis? 4

ANS:
a) ATHEROSCLEROSIS:
is intimal lesions (atheroma) composed of soft grumous lipid cores (mainly cholesterol and
cholesterol esters, with necrotic debris) covered by fibrous caps which can mechanically
obstruct vascular lumen and are prone to rupture, resulting in catastrophic vessel
thrombosis. Plaques also weaken the underlying media, sometimes leading to aneurysm
formation.
b) RISK FACTORS:
A. Non-modifiable (Constitutional)
1. Genetic abnormalities
2. Family history
3. Increasing age
4. Male gender
B. Modifiable
1. Hyperlipidemia
2. Hypertension
3. Cigarette smoking
4. Diabetes
5. Inflammation
C. Additional Risk Factors
1. Inflammation
2. Raised CRP level
3. Hyper-homocystenemia
4. Metabolic syndromes (insulin resistance, hypertension, dyslipidemia (elevated LDL
and depressed HDL), hypercoagulability, and a pro-inflammatory state)
5. Abnormality in lipoprotein A levels
6. Elevated levels of pro-coagulants
7. Other (lack of exercise, stress full life style like type A personality
c) ROLE OF LIPIDS:
The mechanisms by which dyslipidemia contributes to atherogenesis include the
following:
1. Chronic hyperlipidemia, particularly hypercholesterolemia,  increase local
oxygen free radical production  directly impair endothelial cell function
2. oxygen free radicals accelerate NO decay  damping its vasodilator activity.
3. With chronic hyperlipidemia, lipoproteins accumulate within the intima, and
produce oxidized LDL and cholesterol crystals.
4. macrophages or endothelial cells generate oxygen free radicals  oxidize LDL.
5. Oxidized LDL is ingested by macrophages through the scavenger receptor  foam
cell formation.
6. Oxidized LDL stimulates the local release of growth factors, through activation of
the inflammasome and release, cytokines, and chemokines, increasing monocyte
recruitment,
7. Oxidized LDL is cytotoxic to endothelial cells and smooth muscle cells leads to
endothelial cells activation and smooth muscles cells proliferation.

KMU Past Papers Solved SEQs


Cardiovascular System 3

8. Extracellular cholesterol crystals activate innate immune cells such as monocytes


and macrophages.

Q-2
a) Write a brief note on the cardiac enzymes 3.5
b) How will you diagnose type 2 diabetes mellitus in the lab? 3
c) Enumerate the renal function tests 3.5

ANS:
a) CARDIAC ENZYMES:
Certain proteins and enzymes are released into the blood from necrotic heart muscle after
acute MI. these are
1. Creatine Kinase iso-enzyme MB (CK-MB)
 2 forms—while CK-MB1 is extra-cardiac form CK-MB2 is the myocardial form.
 A ratio of CK-MB2: CK-MB1 above 1.5 is highly sensitive for the diagnosis of acute MI
 CK-MB appears within 4 to 8 hours, peaks at 24 hours, and disappears within 1.5 to 3
days.
 Sensitivity and specificity is 95%
 In Re-infarction reappearance of CK-MB after 3 days
2. Cardiac troponins I (cTnI) and T (cTnT)
 cTnI and cTnT appear within 3 to 12 hours, peak at 24 hours, and disappear within 7
to 10 days.
 Sensitivity of 84% to 96% and a specificity of 80% to 95%.
 Doesn’t use to detect re-infarction (troponins cannot because they are normally
increased for 10 days)
3. Lactate dehydrogenase (LDH)
 LDH-1 is myocardial-specific.
 ratio of LDH-1: LDH-2 above 1 will help in making a diagnosis.
 LDH levels begin to rise after 24 hours, reach peak in 3 to 6 days and return to normal
in 14 days.
4. Myoglobin
 first cardiac marker to become elevated after myocardial infarction,
 not cardio-specific
 return to normal within 24 hours of attack of acute MI.
b) LAB DIAGNOSIS OF DIABETES MELLITUS
Given somewhere else**
c) RENAL FUNCTION TESTS
1. Urine Analysis
i) Physical examination (output, color, specific gravity, pH, osmolality)
ii) Chemical constituents (protein, glucose, hemoglobin ketones bilirubin
urobilinogen) electrophoresis.
iii) Bacteriologic examination culture and sensitivity.
iv) Microscopy (sediments i.e blood cells, cells casts and crystals)
2. Concentration and Dilution Tests
i) Concentration test (fluid deprivation test)
ii) Dilution test (excess fluid intake test)
3. Blood Chemistry
i) Urea
ii) Blood urea nitrogen (BUN)

KMU Past Papers Solved SEQs


Cardiovascular System 4

iii) Creatinine
iv) Beta2-microglobulin
v) BUN/creatinine (Cr) ratio normal ratio
4. Renal Clearance Test
i) Inulin or mannitol clearance test
ii) Creatinine clearance
iii) Urea clearance
iv) Para-aminohippuric acid (PAH) clearance

Q-3
a) Define infective endocarditis 2
b) Name the predisposing factors of infective endocarditis 3
c) What is the criteria of lab? diagnosis of infective endocarditis 5

Ans:
a) INFECTIVE ENDOCARDITIS
Infective or bacterial endocarditis (IE or BE) is serious infection of the valvular and
mural endocardium caused by different forms of microorganisms and leads to the
destruction of the underlying cardiac tissues which is characterized by typical infected
and friable vegetations composed of necrotic debris, thrombus, and organisms.
b) PREDISPOSING FACTORS:
1. Conditions initiating transient bacteremia, septicemia and pyaemia.
i) Periodontal infections, tooth extraction and other dental procedures.
ii) Upper and lower respiratory tract infections including bacterial pneumonias.
iii) Intravenous drug abuse.
iv) Cardiac catheterization and cardiac surgery for implantation of prosthetic
valves, prostatic valves implants, damaged endocardium due to jet streams
and pacemaker line.
v) Hemodialysis
2. Underlying heart disease.
i) Chronic rheumatic valvular disease in about 50% cases.
ii) Congenital heart diseases in about 20% cases. These include VSD, sub-aortic
stenosis, pulmonary stenosis, bicuspid aortic valve, Coarctation of the aorta,
and TOF and PDA.
iii) Other causes are syphilitic aortic valve disease, atherosclerotic valvular
disease and floppy mitral valve,
3. Host factors
i) Impaired specific immunity in lymphomas.
ii) Leukemia.
iii) Cytotoxic therapy for various forms of cancers and transplant patients.
iv) Deficient functions of neutrophils and macrophages.
v) diabetes mellitus
vi) alcohol abuse
vii) HIV infection.
c) INVESTIGATION CRITERIA:
A. clinical finding:
1. Fever is the most consistent sign (90% of cases).
2. Immuno-complex vasculitis:
i) glomerulonephritis (nephritic type)

KMU Past Papers Solved SEQs


5

ii) Roth spot (irregular red area with central white dot (retinal hemorrhages)
3. Micro-embolization findings
i) Splinter hemorrhages are linear hemorrhages that are present in the nail
beds
ii) Janeway lesions are painless areas of hemorrhage on the palms and soles of
the feet
iii) Osler nodes are painful hemorrhagic nodules on the pads of the fingers or
toes
iv) Infarctions may occur in different tissue sites ie spleen kidneys and brain.
4. Heart murmurs (regurgitant types)
5. Splenomegaly is present if endocarditis is subacute.

B. Laboratory findings.
1. Positive blood cultures are present in 80% of cases. (Low percentage reflects
that the patient took antibiotics when the cultures are drawn.
2. Neutrophilic leukocytosis occurs in acute IE.
3. Monocytosis occurs in subacute IE.
4. Echocardiography or trans-esophageal echocardiography to detect vegetation
on the valves

Q-4
a) What is Rheumatic fever? 2
b) Give pathogenesis of acute Rheumatic heart disease 4
c) Enumerate squeal of Rheumatic heart disease 4

ANS:
a) RHEUMATIC FEVER:
Rheumatic fever (RF) is a systemic, post streptococcal (group A β-hemolytic), non-suppurative
inflammatory disease, principally affecting the heart, joints, central nervous system, skin and
subcutaneous tissues.
Rheumatic heart disease (RHD) is the cardiac manifestation of rheumatic fever which is
associated with inflammation of all parts of the heart, (pancarditis) valvular inflammation and
scarring.
b) PATHOGENESIS:
1. A susceptible host, on being encountered with group A Streptococcus infection, usually
pharyngitis, mounts an autoimmune reaction by formation of autoantibodies against
bacteria.
2. Streptococcal epitopes present on the bacterial cell wall, cell membrane and the
streptococcal M protein, are immunologically identical to human molecules on myosin,
keratin, actin, laminin, vimentin and N-acetylglucosamine.
3. The autoantibodies cause damage to human tissues due to cross-reactivity between
epitopes in the components of bacteria and the host. Injury is caused by
 Type II antibody-mediated hypersensitivity reaction
 activation of complement and Fc receptor
 CD4+ T cytokine-mediated inflammatory responses.

KMU Past Papers Solved SEQs


Cardiovascular System 6

4. Molecular mimicry and cross-reactivity between streptococcal M protein in particular and


the human molecules forms the basis of autoimmune damage to human target tissues in
RHD cardiac muscle and valves
c) SEQUEAL OF RHEUMATIC HEART DISEASE
1. Myocarditis,
2. Chronic valvular deformities, especially the mitral stenosis,
3. Arrhythmias (atrial fibrillation) due to involvement of the conduction system of the heart
4. Chronic rheumatic carditis
5. Infective endocarditis.
6. Cardiac hypertrophy or dilation,
7. CHF,
8. Thromboembolic complications due to atrial mural thrombi.

Q-5
a) What is Rheumatic fever? 2
b) Give a brief account of morphology of Rheumatic carditis 4
c) laboratory diagnosis of rheumatic carditis 3

ANS:
a) RHEUMATIC FEVER:
Rheumatic fever (RF) is a systemic, post streptococcal (group A β-hemolytic), non-suppurative
inflammatory disease, principally affecting the heart, joints, central nervous system, skin and
subcutaneous tissues.
b) MORPHOLOGY:
1. Aschoff bodies having following features
a. Grossly: spheroidal or fusiform distinct tiny structures, occurring in the interstitium of the
heart in RF
b. Microscopically
1. increase in acid mucopolysaccharide,
2. collagen fibers disintegration (fibrinoid degeneration).
3. collection of T cells, plasma cells
4. granulomas with central fibrinoid necrosis and are surrounded by palisade of
cardiac histiocytes (Anitschkow cells) and multinucleate Aschoff cells.
5. These are large mononuclear cells having abundant cytoplasm central nuclei with
chromatin condensed to form a slender, wavy ribbon (so-called caterpillar cells).
2. Aschoff bodies can be found in any of the three layers of the heart
1. The pericardium involvement shows inflammatory cells infiltrate in subserosal
connective tissue and fibrinous exudate, which generally resolves without sequelae
some time undergoes organization.
2. The myocardial involvement shows scattered Aschoff bodies within the interstitial
connective tissue.
3. Valve involvement results in fibrinoid necrosis and fibrin deposition along the lines of
closure vegetations (verrucae), firmly attached so that they are not likely to get
detached to form emboli. hyalinization and calcification of valve leaflet in late stages.
4. endocardium involvement result in MacCallum’s patch (lesions are map-like area of
thickened, roughened and wrinkled)
3. Chronic rheumatic heart disease
a. Grossly

KMU Past Papers Solved SEQs


Cardiovascular System 7

1. valve cusps and leaflets become permanently thickened and retracted


2. leaflet thickening, commissural fusion and shortening (base of leaflets are mobile while
margins are puckered and thick with narrow orifice called “purse string puckering”
3. thickening and fusion of the chordae tendineae
4. “fishmouth” or “buttonhole” stenosis in mitral valve
b. Microscopically
1. fibrosis occurs in Aschoff nodules and become fibrocollagenous scar with little
cellularity,
2. neovascularization and diffuse fibrosis
c) DIAGNOSIS:
A. Major criteria: (JONES)
1. Carditis
2. Polyarthritis
3. Chorea (Sydenham’s chorea)
4. Erythema marginatum
5. Subcutaneous nodules
B. Minor criteria:
1. Fever
2. Arthralgia
3. Previous history of RF
4. Laboratory findings of elevated ESR, raised C-reactive protein, and leukocytosis
5. ECG finding of prolonged PR interval.
C. Laboratory diagnosis:
1. Positive throat culture for group A streptococcus,
2. Titer of Antistreptolysin O and S,
3. Titer of Anti-streptokinase,
4. Titer of Anti-strepto hyaluronidase
5. Titer of anti DNAase B).

Q-6A 65 years old man has sudden onset of severe abdominal pain. On examination his H.R:
110/min, B.P 145/100mmHg and diminished pulses in lower extremities. There is pulsatile
abdominal mass. Serum creatinine kinase is normal.
a) What is the most probable diagnosis? 1
b) What is the pathogenesis of this disorder and the complications? 4+5

ANS:
a) ABDOMINAL AORTIC ANEURYSM
b) PATHOGENESIS:
(1) Atherosclerosis  weakens the vessel wall.
(2) hypertension  increase vessel wall stress along with vessel diameter.
(3) Vessel lumen fills with atheromatous debris and blood clots ECM degradation by
inflammatory infiltrate having proteolytic enzymes. (cystic medial degeneration)
(4) structural defects in connective tissue,

a) Inadequate or abnormal connective tissue synthesis


 Mutations in TGF-β receptors  defective elastin and collagen synthesis
 Marfan syndrome; defective fibrillin  abnormal TGF- β sequestration 
dysregulated signaling.
 Ehlers-Danlos syndrome; Defective type III collagen synthesis

KMU Past Papers Solved SEQs


Cardiovascular System 8

b) Excessive connective tissue degradation


 Increased MMP expression
 decreased TIMP expression
c) Cystic medial degeneration; Loss of smooth muscle cells and degenerative changes
due to ischemia of media
and narrowing of vasa vasorum.
d) absence of vasavasorum in abdominal aorta
(5) acquired conditions
a) trauma
b) vasculitis
c) mycotic; due to embolization of septic embolus, extension of lesion and direct
infection by organism.
d) tertiary syphilis; involve vasavasorum and subsequent immune response to them
 obletreative endarteritis  compromise blood flow to media.
Complications:
1. Obstruction of a vessel branching off the aorta (e.g., the renal, iliac, vertebral, or
mesenteric arteries), resulting in distal ischemia of the kidneys, legs, spinal cord, or
gastrointestinal tract, respectively
2. Embolism from atheroma or mural thrombus
3. Impingement on adjacent structures, e.g., compression of a ureter or erosion of
vertebrae by the expanding aneurysm
4. An abdominal mass (often palpably pulsating) that simulates a tumor
5. Rupture into the peritoneal cavity or retroperitoneal tissues, leading to massive,
often fatal hemorrhage

Q-7Define atherosclerosis. Draw and label components of atheromatous plaque.


Enumerate the risk factors for atherosclerosis. 2+3+5
ANS:
a) ATHEROSCLEROSIS:
is intimal lesions (atheroma) composed of soft grumous lipid cores (mainly cholesterol and
cholesterol esters, with necrotic debris) covered by fibrous caps which can mechanically obstruct
vascular lumina and are prone to rupture, resulting in catastrophic vessel thrombosis. Plaques
also weaken the underlying media, sometimes leading to aneurysm formation.
b) COMPONENTS OF ATHEROMATOUS PLAQUE:
1) cells, including smooth muscle cells, macrophages, and T cells
2) extracellular matrix, including collagen, elastic fibers, and proteoglycans, lipoproteins,
plasma and plasmin.
3) intracellular and extracellular lipid
c) RISK FACTORS:
Given somewhere else**

Q-8
a) Enumerate the cardiac enzymes along with their clinical significance. 5
b) Give a brief account of renal functions tests. 5
ANS:

KMU Past Papers Solved SEQs


Cardiovascular System 9

a) CARDIAC ENZYMES:
Given somewhere else**
b) RENAL FUNCTION TESTS
Given somewhere else**

Q-9A 42 years old male smoker with hypertension was admitted with history of recent onset chest
pain, on examination patient had elevated serum lipids and C reactive proteins,
a) What is the underlying pathological lesion?
b) Briefly write the pathogenesis of this lesion,
c) List the complications of the above condition,
ANS:
A) ATHEROSCLEROSIS:
B) PATHOGENESIS:
1. Endothelial injury:
Due to
i) Hyperlipidemia
ii) Hypertension
iii) Smoking toxins
iv) Homocysteine
v) Immune complexes
Leads to
i) Increase permeability
ii) Leukocyte adhesion
iii) Thrombosis
2. Platelets adhesion leads to thrombosis
3. Monocyte adhesion and migration to intima differentiate to macrophages
4. Lipid accumulation in intima which causes
i) Oxidized LDL formation by action of ROS and causes
Foam cell formation
Release growth factors cytokines and chemokines
ii) Cholesterol ester formation  inflammasome activation interleukin 1
release  inflammation
5. Inflammatory cells activation
i) Expression of VCAM-I on endothelial cells  migration of T cells (to
produce interferon Gamma) and Monocytes (to produce monocytes
cytokines and growth factors)
6. Infections ie
i) Herpes virus
ii) CMV
iii) Chlamydial infections
7. Smooth muscles proliferation:
i) Smooth muscles either derived from intima or from circulation
ii) Activated by PDGF (platelets derived growth factor), FGF (fibroblast growth
factor), TGF a (tissue growth factor alpha)
iii) Activated smooth muscles produce ECM
c) COMPLICATIONS:
given somewhere else**

KMU Past Papers Solved SEQs


Cardiovascular System 10

Q-10
a) Name the main forms of endocarditis 3
b) What is Rheumatic fever? 2
c) Briefly describe the pathogenesis of Rheumatic carditis 5
ANS:
a) FORMS OF ENDOCARDITIS:
1. Non infective:
i) Rheumatic endocarditis
ii) Atypical verrucous (Libman-Sacks) endocarditis
iii) Non-bacterial thrombotic (cachectic, marantic) endocarditis
2. Infective:
i) Bacterial endocarditis
ii) Other infective types (tuberculous, syphilitic, fungal, viral, rickettsial)

b) RHEUMATIC FEVER
Rheumatic fever (RF) is a systemic, post streptococcal (group A β-hemolytic), non-
suppurative inflammatory disease, principally affecting the heart, joints, central nervous
system, skin and subcutaneous tissues.

c) PATHOGENESIS:
given somewhere else**

Q-11
a) What is the difference between arteriosclerosis and atherosclerosis? 2
b) What are the risk factors of atherosclerosis? 3
c) How does typical atheroma appear? 5
ANS:
a) DIFFERENCE BETWEEN ARTERIOSCLEROSIS AND ATHEROSCLEROSIS
Arteriosclerosis means “hardening of the arteries”; it is a generic term reflecting arterial
wall thickening and loss of elasticity. Three distinct types are recognized, each with
different clinical and pathologic consequences:
Arteriolosclerosis affects small arteries and arterioles and may cause downstream
ischemic injury. The variants forms are:
1) Hyaline
2) Hyperplastic arteriolosclerosis.
3) Monekeberg medial sclerosis
4) Senile arteriosclerosis
Atherosclerosis is characterized by the presence of intimal lesions called atheroma (or
atheromatous or atherosclerotic plaques).
Atheromatous plaques are raised lesions composed of soft grumous lipid cores (mainly
cholesterol and cholesterol esters, with necrotic debris) covered by fibrous caps.
b) RISK FACTORS
Given somewhere else**
c) ATHEROSCLEROTIC PLAQUE (ATHEROMA):

KMU Past Papers Solved SEQs


Cardiovascular System 11

Atheromatous plaques are white to yellow raised lesions; they range from 0.3 to 1.5 cm in
diameter but can coalesce to form larger masses. Atherosclerotic plaques are patchy,
usually involving only a portion of any given arterial wall; on cross-section, therefore, the
lesions appear “eccentric”.
Atherosclerotic plaques have three principal components:
(1) cells, including smooth muscle cells, macrophages, and T cells;
(2) extracellular matrix, including collagen, elastic fibers, and proteoglycans; and
(3) intracellular and extracellular lipid.

Morphology:
Gross Appearance:
1. Atherosclerotic plaques are patchy, usually involving only a portion of any given
arterial wall.
2. Atheromatous plaques are white to yellow raised lesions
3. Thrombus superimposed on ulcerated plaques imparts a red-brown color.
4. Cut section of the plaque reveals firm, white fibrous cap and a central necrotic core
Microscopically:
1. the lesions appear eccentric involve only one side of the lumen.
2. Superficial luminal part of the fibrous cap is covered by endothelium, and is
composed of smooth muscle cells, dense connective tissue and extracellular matrix
containing proteoglycans and collagen Where the cap meets the vessel wall (the
“shoulder”) is a more cellular area containing macrophages, T cells, and smooth
muscle cells.
3. Deep to the fibrous cap is a necrotic core, containing lipid (primarily cholesterol and
cholesterol esters), necrotic debris, lipid-laden macrophages and smooth muscle
cells (foam cells), fibrin, variably organized thrombus, and other plasma proteins.
4. cholesterol clefts are present
5. The periphery of the lesions shows neovascularization
6. The media deep to the plaque there is fibrosis secondary to smooth muscle atrophy
and loss.
7. atheroma contains abundant lipid, but some so-called fibrous plaques are
composed almost exclusively of smooth muscle cells and fibrous tissue.
8. Plaques undergoes degeneration, synthesis and degradation of ECM (remodeling),
and thrombus organization.
9. Complicated plaques include calcification, ulceration, thrombosis, hemorrhage and
aneurysmal dilatation.

Q-12 A 46 years old man experienced sudden retrosternal chest pain with radiation to his left
arm. Laboratory studies show raised serum troponin and CKMB.
a) What is your most likely diagnosis? 1
b) During the next 24 hours which three important complications he is most likely to experience.4
c) Write three important relevant points of each complication 5
ANS:
a) MYOCARDIAL INFARCTION
b) COMPLICATIONS
1) Cardiogenic shock occurs in 7% of cases.
 left ventricular failure manifested

KMU Past Papers Solved SEQs


Cardiovascular System 12

 hypotension,
 pulmonary congestion
 pulmonary edema
2) Arrhythmias
 Ventricular premature contractions are the most common arrhythmia.
 Most common cause of death is ventricular fibrillation.
 Heart block
 The risk of serious arrhythmias (e.g., ventricular fibrillation) is greatest in the first
hour and declines thereafter.
3) Congestive heart failure
 Usually occurs within the first 24 hours
 failing heart can no longer efficiently pump the blood delivered to it by the venous
circulation. increased end-diastolic ventricular volume, leading to increased end-
diastolic pressures elevated venous pressures.
 forward failure due to inadequate cardiac output
 backward failure due to increased congestion of the venous circulation.

Q-13 A 55 years old diabetic suddenly experiences sharp stabbing pain and epigastrium with
sweating and begins to faint. Emergency investigation shows increased cardiac troponin I
a) Name the disease, this patient is suffering from? 1
b) Write down only four important risk factors of this condition 2
c) Mention only four causes of this condition including the cause in majority of cases 2
d) Which other important laboratory investigation you need to augment the diagnosis? 5

ANS:
a) MYOCARDIAL INFARCTION
b) RISK FACTORS
1. Age: Men ≥45 years old, women ≥55-year-old
2. Family history of premature coronary artery disease or stroke
3. Lipid abnormalities
Low-density lipoprotein (LDL) >160 mg/dL
High-density lipoprotein (HDL) <40 mg/dL
4. Smoking tobacco,
5. HTN,
6. diabetes mellitus (DM)
c) CAUSES OF MI
1. Atherosclerotic causes:
i) Coronary atherosclerotic plaque rupture (acute coronary syndrome)
ii) Superimpose coronary artery thrombosis
iii) intra-mural hemorrhage.
iv) Cicatric stenosis
2. Non atherosclerotic causes:
i) Coronary vasospasm,
ii) Embolization from mural thrombi, valve vegetation.
iii) Vasculitis (polyarteritis nodosa, kawasaki disease)
iv) Coronary ostial stenosis,
v) Aortic dissection involving coronary arteries.
vi) Thrombotic diseases; antithrombin III deficiency, polycythemia vera
vii) Amyloidosis

KMU Past Papers Solved SEQs


Cardiovascular System 13

viii) Sickle cell disease


ix) Trauma and outside compression
x) Shock, exercise, intense emotion.
d) LABORATORY INVESTIGATION
1. Clinical features
2. ECG findings:
3.Serum cardiac markers

1. clinical features:
i) Pain: Usually sudden, severe, crushing and prolonged, substernal or precordial in
location, unrelieved by rest or nitroglycerin, often radiating to one or both the
arms, neck and back.
ii) Indigestion: Pain is often accompanied by epigastric or substernal discomfort
interpreted as ‘heartburn’ with nausea and vomiting.
iii) Apprehension: The patient is often terrified, restless and apprehensive due to
great fear of death.
iv) Shock: Systolic blood pressure is below 80 mmHg; lethargy, cold clammy limbs,
peripheral cyanosis, weak pulse, tachycardia or bradycardia are often present.
v) Oliguria: Urine flow is usually less than 20 ml per hour.
vi) Low grade fever: Mild rise in temperature occurs within 24 hours and lasts up to
one week, accompanied by leukocytosis and elevated ESR.
vii) Acute pulmonary edema: Some cases develop severe pulmonary congestion due
to left ventricular failure and develop suffocation, dyspnea, orthopnea and
bubbling respiration.
2. ECG findings:
 ST segment elevation in acute MI (termed as STEMI)
 T wave inversion and appearance of wide deep Q waves
 Loss of R wave amplitude.
3.Serum cardiac markers
a) Serial testing for creatine kinase isoenzyme MB (CK-MB)
 2 forms CK-MB2 is the myocardial form while CK-MB1 is extra-cardiac form.
 A ratio of CK-MB2: CK-MB1 above 1.5 is highly sensitive for the diagnosis of acute
MI
 CK-MB appears within 4 to 8 hours, peaks at 24 hours, and disappears within 1.5
to 3 days.
 Sensitivity and specificity is 95%
 In Re-infarction reappearance of CK-MB after 3 days
b) Serial testing for cardiac troponins I (cTnI) and T (cTnT)
 cTnI and cTnT appear within 3 to 12 hours, peak at 24 hours, and disappear
within 7 to 10 days.
 Sensitivity of 84% to 96% and a specificity of 80% to 95%.
 Doesn’t use to detect re-infarction (troponins cannot because they are normally
increased for 10 days)
c) Lactate dehydrogenase (LDH)
 LDH-1 is myocardial-specific.
 LDH levels begin to rise after 24 hours, reach peak in 3 to 6 days and return to
normal in 14 days.
 Estimation of ratio of LDH-1: LDH-2 above 1 is reasonably helpful in making a
diagnosis.

KMU Past Papers Solved SEQs


Cardiovascular System 14

d) Myoglobin
 first cardiac marker to become elevated after myocardial infarction,
 not cardio-specific
 return to normal within 24 hours of attack of acute MI.

Q-14
a) Define aneurysm 1
b) What are the most important causes of aneurysm? 3
c) Write the salient features of pathogenesis of abdominal aortic aneurysm 3
d) Enumerate the complications of aortic aneurysm. 3

ANS:
a) ANEURYSM:
An aneurysm is defined as a permanent abnormal dilatation of a blood vessel occurring
due to congenital or acquired weakening or destruction of the vessel wall.
b) CAUSES OF ANEURYSM:
1. Atherosclerosis: more dominant factor in abdominal aortic aneurysms
2. Hypertension: is associated with ascending aortic aneurysms.
3. Weak vessel walls due to
 trauma
 vasculitis (polyarthritis nodosa and Kawasaki disease)
 congenital defects (TGF-β receptors, Marfan syndrome, Ehlers-Danlos
syndrome)
4. Infections:
 Mycotic aneurysms, (bacteremia from salmonella gastroenteritis)
 Tertiary syphilis, involving vasavasorum and subsequent immune response to
them.
5. Pregnancy
c) PATHOGENESIS
Given somewhere else**
d) COMPLICATIONS OF AORTIC ANEURYSM:
Given somewhere else**

Q-15
a) What conditions are termed acute coronary syndrome? 2
b) Which coronary vessels may be affected? 4
c) What is the role of acute plaque change in acute coronary syndrome? 4

ANS:
a) ACUTE CORONARY SYNDROME:
The term acute coronary syndrome is applied to any of the three catastrophic
manifestations of ischemic heart disease(IHD)
1) unstable angina,
2) acute myocardial infarction(MI),
3) sudden cardiac death(SCD).
b) VESSELS AFFECTED:
1. Left anterior descending (LAD),  (40-50%)
2. Right coronary artery (RCA)  (30-40 %)
3. Left circumflex (LCX),  (15-20 %)

KMU Past Papers Solved SEQs


Cardiovascular System 15

Sometimes, secondary branches also are involved i.e.,


4. Diagonal branches of the LAD,
5. Obtuse marginal branches of the LCX,
6. Posterior descending branch of the RCA).
c) ACUTE PLAQUE CHANGES
1. Sudden disruption of partially occlusive plaque vulnerable plaque.
2. Rupture, fissuring, or ulceration of plaques can expose highly thrombogenic
constituents or underlying sub-endothelial basement membrane, leading to rapid
thrombosis.
3. In addition, hemorrhage into the core of plaques can expand plaque volume,
thereby acutely exacerbating the degree of luminal occlusion.
4. Plaques that contain large atheromatous cores and thin overlying fibrous caps
deficiency of smooth muscle cells or large numbers of inflammatory cells are
termed “vulnerable.” To rupture.
5. Blood pressure and platelet reactivity, local vasospasm and increase adrenergic
stimulation due to intense emotional stress

Q-16
a) What are the principal components of atheroma? 1.5
b) What changes can occur in atherosclerotic plaque? 2
c) Write the morphology of fatty streaks and atherosclerotic plaque? 1.5 +1.5
d) What are the four major consequences of atherosclerosis? Enumerate in descending order, the
vessels most frequently involved in atherosclerosis. Which vessels are usually spared? 1+1.5 +1

ANS:
a) PRINCIPAL COMPONENTS OF ATHEROMA (ATHEROSCLEROTIC PLAQUE)
1. cells, including smooth muscle cells, macrophages, and T cells;
2. Extracellular matrix, including collagen, elastic fibers, and proteoglycans
3. intracellular and extracellular lipid.
b) CHANGES OCCUR IN ATHEROSCLEROTIC PLAQUE:
1. Rupture/fissuring, exposing highly thrombogenic plaque constituents
2. Erosion/ulceration, exposing the thrombogenic sub-endothelial basement membrane
to blood
3. Hemorrhage into the atheroma, expanding its volume
c) MORPHOLOGY OF
Fatty streaks:
Grossly:
Fatty streaks begin as minute yellow, flat macules that coalesce into elongated lesions, 1
cm or more in length
Microscopically: They are composed of lipid-filled foamy macrophages smooth muscles
cells and some lymphocytes.
Atherosclerotic plaque:
Gross Appearance:
1. Atherosclerotic plaques are patchy, usually involving only a portion of any given
arterial wall.
2. Atheromatous plaques are white to yellow raised lesions
3. Thrombus superimposed on ulcerated plaques imparts a red-brown color.
4. Cut section of the plaque reveals firm, white fibrous cap and a central necrotic core

KMU Past Papers Solved SEQs


Cardiovascular System 16

Microscopically:
1. The lesions appear eccentric involve only one side of the lumen.
2. Superficial luminal part of the fibrous cap is covered by endothelium, and is composed
of smooth muscle cells, dense connective tissue and extracellular matrix containing
proteoglycans and collagen where the cap meets the vessel wall (the “shoulder”) is a
more cellular area containing macrophages, t cells, and smooth muscle cells.
3. Deep to the fibrous cap is a necrotic core, containing lipid (primarily cholesterol and
cholesterol esters), necrotic debris, lipid-laden macrophages and smooth muscle cells
(foam cells), fibrin, variably organized thrombus, and other plasma proteins.
4. Cholesterol clefts are present
5. The periphery of the lesions shows neovascularization
6. The media deep to the plaque there is fibrosis secondary to smooth muscle atrophy and
loss.
7. Atheroma contains abundant lipid, but some so-called fibrous plaques are composed
almost exclusively of smooth muscle cells and fibrous tissue.
8. Plaques undergoes degeneration, synthesis and degradation of ECM (remodeling), and
thrombus organization.
9. Complicated plaques include calcification, ulceration, thrombosis, hemorrhage and
aneurysmal dilatation.
d) CONSEQUENCES OF ATHEROSLEROTIC PLAQUE:
1. Myocardial infarction
2. Cerebral infarction (stroke)
3. Aortic aneurysm
4. Peripheral vascular disease (gangrene of extremities)
5. Ischemic bowl disease
Most extensively involved vessels are: (In descending order)
1. the infrarenal abdominal aorta,
2. the coronary arteries,
3. the popliteal arteries,
4. the internal carotid arteries,
5. the vessels of the circle of Willis.
Vessels usually spared:
1. Vessels of the upper extremities
2. mesenteric and renal arteries,

Q-17
(a) Define Rheumatic fever. 2
(b) Write the morphology of cardiac manifestations of acute Rheumatic fever. 4.5
(c) Which valvular abnormality is most commonly seen in RHD. 1
(d) What are the Jones criteria for diagnosis of acute RHD? 2.5

ANS:
a) RHEUMATIC FEVER (RF)
Rheumatic fever is a systemic, post streptococcal (group A β-hemolytic), non-suppurative
inflammatory disease, principally affecting the heart, joints, central nervous system, skin
and subcutaneous tissues.

KMU Past Papers Solved SEQs


Cardiovascular System 17

b) MORPHOLOGY:
c) Given somewhere else**
d) Mitral valve stenosis
e) DIAGNOSIS:
Given somewhere else**

Q-18 A 11 years old child presents with fever, joint pain & swelling along with shortness of
breath. He has a history of sore throat few weeks back. On examination he has a mid-diastolic
murmur & his left knee is swollen & tender. Lab work up shows raised ESR & raised ASO titer.
Echo shows mitral stenosis.
a) What is the most likely diagnosis? 2
b) Name the distinctive histological lesions in the heart in this condition &
briefly write its morphology 1+2
c) Enumerate at least four complications of the above condition 2
d) Briefly describe pathogenesis of acute rheumatic heart disease. 3

ANS:
a) ACUTE RHEUMATIC FEVER
b) ASCHOFF BODIES
Morphology:
a. Grossly: spheroidal or fusiform distinct tiny structures, occurring in the interstitium of
the heart in RF
b. Microscopically
1. Collagen fibers disintegration (fibrinoid degeneration).
2. Collection of t cells, plasma cells
3. Granulomas with central fibrinoid necrosis and are surrounded by palisade of
cardiac histiocytes (anitschkow cells) and multinucleate aschoff cells.
4. These are large mononuclear cells having abundant cytoplasm central nuclei with
chromatin condensed to form a slender, wavy ribbon (so-called caterpillar cells).
c) COMPLICATIONS OF RHEUMATIC HEART DISEASE
1. Myocarditis,
2. Chronic valvular deformities, especially the mitral stenosis,
3. Arrhythmias (atrial fibrillation) due to involvement of the conduction system of the
heart
4. Chronic rheumatic carditis
5. Infective endocarditis.
6. Cardiac hypertrophy or dilation,
7. Congestive heart failure.
8. Thromboembolic complications due to atrial mural thrombi.

d) PATHOGENESIS:
Given somewhere else**

KMU Past Papers Solved SEQs


Cardiovascular System 18

Q-19 A 63-year-old woman has the sudden onset of "knife-Like" pain in the chest radiating to the
back. She has previous history of poorly controlled hypertension. On examination, she has
blood pressure of 150/100mmHg. A chest radiograph reveals a widened mediastinum.
Laboratory findings include. Normal serum creatine kinase MB.
a) Which one of the following three is the most likely diagnosis? 2
(i) Aortic Dissection (ii) Infective endocarditis (iii) Myocardial infarction

b) Write the important morphological points related to the lesion 6


c) Give two important risk factors related to the diagnosis. 2
ANS:
a) AORTIC DISSECTION
b) MORPHOLOGY:
Grossly:
1. In most dissections, the intimal tear marking the point of origin is found in the
ascending aorta
2. The tears are transverse or oblique in orientation and with sharp, jagged edges.
3. Extend retrograde toward the heart and Distally, as far as the iliac and femoral
arteries,
4. Tear lies between the middle and outer thirds of the media.
5. Double-barreled aorta form (the dissecting hematoma reenters the lumen of the
aorta through a second distal intimal tear, creating a second vascular channel within
the media)
6. Over time, these channels become endothelialized to give rise to chronic
dissections.
histologically
1. lesion is the cystic medial degeneration, characterized by smooth muscle layer
dropout and necrosis, elastic tissue fragmentation,
2. accumulations of amorphous proteoglycan-rich ECM
3. Inflammation is characteristically absent.

c) RISK FACTORS:

A. Hypertensive state About 90%


B. Non-hypertensive cases
1. Marfan’s syndrome, an autosomal dominant disease with genetic defect in fibrillin
which is a connective tissue protein required for elastic tissue formation.
2. Development of cystic medial necrosis of Erdheim, especially in old age.
3. Iatrogenic trauma during cardiac catheterization or coronary bypass surgery.
4. Pregnancy, for some unknown reasons.

PEARLS:
Aortic Dissection can be classified as:
1) Stanford classification
Type A (lesion proximal to subclavian artery)
Type B (lesion distal to subclavian artery)
2) Debacky
Type I (lesion proximal to subclavian artery involve all aorta)

KMU Past Papers Solved SEQs


Cardiovascular System 19

Type II (lesion proximal to subclavian artery involve only proximal aorta)


Type III (lesion distal to subclavian artery)

Q-20
a) Write down the major criteria for diagnosis Of rheumatic fever 4
b) Name Ischemic Heart Diseases? 2
c) Discuss lab investigations in a case of myocardial 4

ANS:
a) MAJOR CRITERIA:
1. Carditis
2. Polyarthritis
3. Chorea (Sydenham’s chorea)
4. Erythema marginatum
5. Subcutaneous nodules
b) ISCHEMIC HEART DISEASES:
1. Angina pectoris (most common type)
2. Chronic ischemic heart disease (CIHD)
3. Sudden cardiac death
4. Acute myocardial infarction (AMI)
c) LABORATORY INVISTIGATIONS:
1. Clinical features
2. ECG findings:
3. Serum cardiac markers

1. clinical features:
i) Pain: Usually sudden, severe, crushing and prolonged, substernal or precordial
in location, unrelieved by rest or nitroglycerin, often radiating to one or both the
arms, neck and back.
ii) Indigestion: Pain is often accompanied by epigastric or substernal discomfort
interpreted as ‘heartburn’ with nausea and vomiting.
iii) Apprehension: The patient is often terrified, restless and apprehensive due to
great fear of death.
iv) Shock: Systolic blood pressure is below 80 mmHg; lethargy, cold clammy limbs,
peripheral cyanosis, weak pulse, tachycardia or bradycardia are often present.
v) Oliguria: Urine flow is usually less than 20 ml per hour.
vi) Low grade fever: Mild rise in temperature occurs within 24 hours and lasts up to
one week, accompanied by leukocytosis and elevated ESR.
vii) Acute pulmonary edema: Some cases develop severe pulmonary congestion due
to left ventricular failure and develop suffocation, dyspnea, orthopnea and
bubbling respiration.
2. ECG findings:
 ST segment elevation in acute MI (termed as STEMI)
 T wave inversion and appearance of wide deep Q waves
 Loss of R wave amplitude.
3.Serum cardiac markers
a) Serial testing for creatine kinase isoenzyme MB (CK-MB)
 2 forms CK-MB2 is the myocardial form while CK-MB1 is extracardiac form.
 A ratio of CK-MB2: CK-MB1 above 1.5 is highly sensitive for the diagnosis of acute
MI

KMU Past Papers Solved SEQs


Cardiovascular System 20

 CK-MB appears within 4 to 8 hours, peaks at 24 hours, and disappears within 1.5
to 3 days.
 Sensitivity and specificity is 95%
 In Re-infarction reappearance of CK-MB after 3 days
b) Serial testing for cardiac troponins I (cTnI) and T (cTnT)
 cTnI and cTnT appear within 3 to 12 hours, peak at 24 hours, and disappear
within 7 to 10 days.
 Sensitivity of 84% to 96% and a specificity of 80% to 95%.
 Doesn’t use to detect re-infarction (troponins cannot because they are normally
increased for 10 days)
c) Lactate dehydrogenase (LDH)
 LDH-1 is myocardial-specific.
 LDH levels begin to rise after 24 hours, reach peak in 3 to 6 days and return to
normal in 14 days.
 Estimation of ratio of LDH-1: LDH-2 above 1 is reasonably helpful in making a
diagnosis.
d) Myoglobin
 first cardiac marker to become elevated after myocardial infarction,
 not cardio-specific
 return to normal within 24 hours of attack of acute MI.

Q-21
a) Define Aneurysm. 1
b) Where does atherosclerotic aneurysm occur most frequently?
Enumerate its clinical consequences. 1+3
c) Write the salient features of pathogenesis of abdominal aortic aneurysm. 5
ANS:
a) ANEURYSM:
An aneurysm is defined as a permanent abnormal dilatation of a blood vessel occurring
due to congenital or acquired weakening or destruction of the vessel wall.
b) ABDOMINAL AORTA
Clinical consequences/ complications:
Given somewhere else**
c) PATHOGENESIS:
given somewhere else**

Q-22 A 55-year-old known hypertensive for last 20 years comes to your clinic with dyspnea,
orthopnea, non- productive cough, and confusion. On physical examination there is
generalized edema, rales heard over lung bases. S3 heart sound is heard on auscultation. Echo
revealed left ventricular hypertrophy.
a) What is the most likely diagnosis? 2
b) Name some important causes of this condition? 2
c) Briefly discuss its morphology. 6

ANS:
A) LEFT-SIDED HEART FAILURE

KMU Past Papers Solved SEQs


Cardiovascular System 21

B) CAUSES:
1. ischemic heart disease (IHD),
2. systemic hypertension,
3. mitral or aortic valve disease,
4. restrictive pericarditis
5. primary diseases of the myocardium; myocarditis, cardiomyopathies, amyloidosis).

C) MORPHOLOGY:
heart: morphology depends on the underlying disease process,
 Grossly: myocardial infarction or valvular deformities may be present. the left ventricle
usually is hypertrophied and can be dilated with left atrial enlargement,
 The microscopic changes are primarily of myocyte hypertrophy with interstitial fibrosis
of variable severity.

Lungs.
 Grossly: congestion and edema as well as pleural effusion, the lungs are heavy and boggy,
 Microscopically: show perivascular and interstitial transudates, alveolar septal edema,
and accumulation of edema fluid in the alveolar spaces.
red cells extravasate from the leaky capillaries into alveolar spaces, where they are
phagocytosed by macrophages, break down of hemoglobin leads to the appearance of
hemosiderin-laden alveolar macrophages (heart failure cells)
These cells appear rusty brown on H and E stain while Blue on Prussain Blue stain

Q-23 11 years old child presents with fever, joint pain & swelling along with shortness of
breath. He has a history of sore throat few weeks back. On examination he has a mid-diastolic
murmur & his left knee is swollen & tender. Lab work up shows raised ESR & raised ASO titer.
Echo shows mitral stenosis.
a) What is the diagnosis? 2
b) Name the distinctive histological lesions in the heart in this condition
& briefly explain its morphology 4
c) Enumerate complications of the above condition. 2.5
d) Briefly describe pathogenesis of the above condition. 1.5

ANS:
a) RHEUMATIC FEVER
b) ASCHOFF BODIES
Morphology:
a. Grossly: spheroidal or fusiform distinct tiny structures, occurring in the interstitium of
the heart in RF
b. Microscopically
1. increase in acid mucopolysaccharide,
2. collagen fibers disintegration (fibrinoid degeneration).
3. collection of T cells, plasma cells

KMU Past Papers Solved SEQs


Cardiovascular System 22

4. granulomas with central fibrinoid necrosis and are surrounded by palisade of


cardiac histiocytes (Anitschkow cells) and multinucleate Aschoff cells.
5. These are large mononuclear cells having abundant cytoplasm central nuclei with
chromatin condensed to form a slender, wavy ribbon (so-called caterpillar cells).
c) COMPLICATIONS OF RHEUMATIC HEART DISEASE
1. Myocarditis,
2. Chronic valvular deformities, especially the mitral stenosis,
3. Arrhythmias (atrial fibrillation) due to involvement of the conduction system of the
heart
4. Chronic rheumatic carditis
5. Infective endocarditis.
6. Cardiac hypertrophy or dilation,
7. Congestive heart failure.
8. Thromboembolic complications due to atrial mural thrombi.

d) PATHOGENESIS:
Given somewhere else**

Q-24 A 60 years old man presented with fever, chills, weakness & lassitude. On examination
the physician detected a pan systolic murmur. There were splinter hemorrhages in the nails.
Echocardiogram revealed vegetations on mitral valve with mitral regurgitation.
a) What is the diagnosis? 2
b) Enumerate the organisms responsible for the condition. 2
c) Enlist the diagnostic criteria for diagnosis of the disease. 6

ANS:
A) INFECTIVE ENDOCARDITIS
B) ORGANISMS:
1. Streptococci and staphylococci (90%)
2. Staphylococcus aureus, pneumococci, gonococci, b-streptococci and enterococci in ABE.
3. Streptococcus viridans, (normal flora of the mouth and pharynx) commonest in sabe.
4. Other less common etiologic agents include streptococcus bovis (normal flora of
gastrointestinal tract) and staphylococcus epidermidis (commensal of the skin),
5. Enterococci called hacek group (haemophilus, actinobacillus, cardiobacterium, eikenella,
and kingella), all commensals in the oral cavity.
6. More rarely fungi are involved.

C) DIAGNOSTIC CRITERIA:
A. Clinical findings:
1. Fever is the most consistent sign (90% of cases).
2. Immuno-complex vasculitis:
iii) glomerulonephritis (nephritic type)
iv) Roth spot (irregular red area with central white dot (retinal hemorrhages)
3. Micro-embolization findings

KMU Past Papers Solved SEQs


Cardiovascular System 23

v) Splinter hemorrhages are linear hemorrhages that are present in the nail beds
vi) Janeway lesions are painless areas of hemorrhage on the palms and soles of the feet
vii) Osler nodes are painful hemorrhagic nodules on the pads of the fingers or toes
viii) Infarctions may occur in different tissue sites ie spleen kidneys and brain.
4. Heart murmurs (regurgitant types)
5. Splenomegaly is present if IE is subacute.
B. Laboratory findings
1. Positive blood cultures are present in 80% of cases.
2. Neutrophilic leukocytosis occurs in acute IE.
3. Monocytosis occurs in subacute IE.
4. Mild anemia is most frequently due to anemia of chronic disease
5. Echocardiography or transesophageal echocardiography to detect vegetations on the
valves

Q-25 A 60-year-old man brought with history of hypertension for 20 years presenting with
excruciating fearing pain which start in the anterior chest and radiates to the back between
the scapula.
a) What is the most probable diagnosis? 2
b) Enumerate complications of the condition. 4
c) Enumerate the etiological factors for the above condition 4

ANS:
A) AORTIC DISSECTION:
B) COMPLICATIONS OF CONDITION;
1. The most common cause of death is rupture of the dissection into the pericardial, pleural,
or peritoneal cavity.
2. Retrograde dissection into the aortic root also can cause fatal disruption of the aortic
valvular apparatus
3. compression of the coronary arteries.
4. tamponade, aortic insufficiency, and myocardial infarction.
5. Obstruction of great arteries of the neck and the renal, mesenteric, or iliac arteries,
6. compression of spinal arteries can cause transverse myelitis.

c) RISK FACTORS:

A. Hypertensive state About 90%


B. Non-hypertensive cases
1. Marfan’s syndrome, an autosomal dominant disease with genetic defect in fibrillin
which is a connective tissue protein required for elastic tissue formation.
2. Development of cystic medial necrosis of Erdheim, especially in old age.
3. Iatrogenic trauma during cardiac catheterisation or coronary bypass surgery.

KMU Past Papers Solved SEQs


Cardiovascular System 24

4. Pregnancy, for some unknown reasons.

Q-26 A 55-year-old man with a 10 years history of diabetes comes to you, with severe chest
pain radiating to the left arm, the patient complains of palpitations, sweating and vomiting. On
examination there is ST segment elevation in lead I, II & AVF- on auscultation there is S3 heart
sound.
a) What is the most likely diagnosis? 2
b) Discuss briefly the morphological changes during 1st week of the above condition. 3
c) Enumerate complication of the above mentioned disease. 5

ANS:
A) MYOCARDIAL INFARCTION
B) MORPHOLOGY:
1) 1 -4 − hours
 Grossly: None
 Light microscopy: Usually none.
2) 4–12 hours
 Grossly Occasionally dark mottling
 Light microscopy: Beginning coagulation necrosis; edema; hemorrhage
3) 12–24 hours
 Grossly: Dark mottling
 Light microscopy: Ongoing coagulation necrosis; pyknosis of nuclei; hypereosinophilic
appearance of myocytes; marginal contraction band necrosis; beginning neutrophilic
infiltrate
4) 1–3 days
 Grossly: Mottling with yellow-tan infarct center
 Light microscopy: Coagulation necrosis with loss of nuclei and striations; interstitial
infiltrate of neutrophils
5) 3–7 day
 Grossly: Hyperemic border; central yellow-tan softening
 Light microscopy: Beginning disintegration of dead myofibers, with dying neutrophils;
early phagocytosis of dead cells by macrophages at infarct border

C) COMPLICATIONS:
1. Arrhythmias
2. Congestive heart failure
3. Cardiogenic shock
4. Mural thrombosis and thromboembolism
5. Right ventricular infarction
6. Chamber dilation
7. Cardiac aneurysm
8. Papillary muscle dysfunction (mitral regurgitation)
9. Myocardial Rupture

KMU Past Papers Solved SEQs


Cardiovascular System 25

10. Pericarditis
11. Post myocardial infarction syndrome (Dressler’s syndrome characterized by
pneumonitis)
12. Progressive late heart failure (chronic ischemic heart diseases)

Q-27
a) Enumerate the modifiable and non-modifiable risk factors for atherosclerosis. 4
b) Enumerate the pathogenic events in the response to injury hypothesis of atherosclerosis. 3
c) Briefly describe the morphology of an atherosclerotic plaque. 3
ANS:
a) RISK FACTORS:
Given somewhere else**
b) PATHOGENIC EVENTS IN THE RESPONSE TO INJURY
1. Endothelial injury—and resultant endothelial dysfunction—leading to increased
permeability, leukocyte adhesion, and thrombosis
2. Platelet adhesion
3. Monocyte adhesion to the endothelium, migration into the intima, and differentiation
into macrophages and foam cells
4. Accumulation of lipoproteins (mainly oxidized LDL and cholesterol crystals) in the
vessel wall
5. Lipid accumulation within macrophages, which release inflammatory cytokines
6. Smooth muscle cell recruitment due to factors released from activated platelets,
macrophages, and vascular wall cells
7. Smooth muscle cell proliferation and ECM production
c) ATHEROSCLEROTIC PLAQUE MORPHOLOGY
 Gross Appearance:
1. Atherosclerotic plaques are patchy, usually involving only a portion of any given
arterial wall.
2. Atheromatous plaques are white to yellow raised lesions
3. Thrombus superimposed on ulcerated plaques imparts a red-brown color.
4. Cut section of the plaque reveals firm, white fibrous cap and a central necrotic core
 Microscopically:
1. the lesions appear eccentric involve only one side of the lumen.
2. Superficial luminal part of the fibrous cap is covered by endothelium, and is
composed of smooth muscle cells, dense connective tissue and extracellular matrix
containing proteoglycans and collagen Where the cap meets the vessel wall (the
“shoulder”) is a more cellular area containing macrophages, T cells, and smooth
muscle cells.
3. Deep to the fibrous cap is a necrotic core, containing lipid (primarily cholesterol and
cholesterol esters), necrotic debris, lipid-laden macrophages and smooth muscle
cells (foam cells), fibrin, variably organized thrombus, and other plasma proteins.
4. cholesterol clefts are present
5. The periphery of the lesions shows neovascularization
6. The media deep to the plaque there is fibrosis secondary to smooth muscle atrophy
and loss.

KMU Past Papers Solved SEQs


Cardiovascular System 26

7. atheroma contains abundant lipid, but some so-called fibrous plaques are
composed almost exclusively of smooth muscle cells and fibrous tissue.
8. Plaques undergoes degeneration, synthesis and degradation of ECM (remodeling),
and thrombus organization.
9. Complicated plaques include calcification, ulceration, thrombosis, hemorrhage and
aneurysmal dilatation.

Q-28 A 50 years old man had an attack of acute MI 3 months back. Now he presented to the doctor
with atypical chest pain, getting worse on recumbency. On examination he had prominent
friction rub.
a) What is your diagnosis? 2
b) Discuss its morphology. 5
c) Enumerate its clinical features. 3

ANS:
a) FIBRINOUS PERI CARDITIS
B) MORPHOLOGY:
 Grossly: The cardiac surface is characteristically covered by dry or moist, shaggy,
fibrinous exudate which gives ‘bread and butter’ appearance.
dense fibrous scar tissue bridges the pericardial space
 Histologically, the accumulated extravascular fibrin appears as an eosinophilic
meshwork of threads or sometimes as an amorphous coagulum, extensive fibrin-rich
exudates are replaced by an ingrowth of fibroblasts and blood vessels.
C) COMPILATIONS:
1. Acute pericarditis will cause immediate hemodynamic complications if it elicits a large
effusion resulting in cardiac tamponade, declining cardiac output and consequent shock
2. Chronic constrictive pericarditis will lead to right-sided venous distention and low
cardiac output

Q-29 A 2-weeks old newborn is brought to the physician because his lips have turned blue on
three occasions during feeding, he also sweats during feeding, a grade 3/6 harsh systolic
ejection murmur is heard at left upper sternal border x-ray film shows a small-boot shaped
heart.
a) What is the most likely Diagnosis? 2
b) Name the four cardinal feature of the above condition. 4
c) Briefly discuss morphology of this condition. 4

ANS:
A) TETRALOGY OF FALLOT
B) CARDINAL FEATURE
1. VSD;

KMU Past Papers Solved SEQs


Cardiovascular System 27

2. right ventricular outflow tract obstruction (subpulmonic stenosis);


3. overriding of the VSD by the aorta; and
4. right ventricular hypertrophy

C) MORPHOLOGY:
1. The heart is large and “boot-shaped” as a consequence of right ventricular hypertrophy
while The left-sided cardiac chambers are of normal size
2. the proximal aorta is dilated, while the pulmonary trunk is hypoplastic.
3. The VSD usually is large and lies in the vicinity of the membranous portion of the
interventricular septum;
4. the aortic valve lies immediately over the VSD (overriding aorta)
5. there is narrowing of the infundibulum (subpulmonic stenosis) and pulmonary valve
stenosis or complete atresia of the valve and the proximal pulmonary arteries.

Q-30 A 65-year-old man has sudden onset of severe abdominal pain. On examination his heart
rate is 110 beats per minute, Blood pressure 145/100 mg Hg and diminished pulses in lower
extremities, there is pulsatile abdominal mass, serum creatinine kinase is normal.
a) Name the condition. 2
b) Name the important causes of the above condition. 4
c) Enumerate four complications of above condition. 4

ANS:
a) ABDOMINAL AORTIC ANEURYSM:
b) CAUSES OF ABDOMINAL AORTIC ANEURYSM:
1. Atherosclerosis: more dominant factor in abdominal aortic aneurysms
2. Hypertension: is associated with ascending aortic aneurysms.
3. weak vessel walls:
 trauma
 vasculitis
 congenital defects (Marfan syndrome, Ehlers-Danlos syndrome)
 Excessive connective tissue degradation
 absence of vasavasorum in abdominal aorta
4. infections:
 Mycotic aneurysms, (bacteremia from salmonella gastroenteritis)
 Tertiary syphilis, (A predilection of the spirochetes for the vasavasorum)
d) CLINICAL CONSEQUENCES:
given somewhere else*

Q-31 A 46-year-old man has the sudden onset of substernal chest pain with radiation to his left
arm. This persist for the next 6 hours, laboratory studies show a rise serum troponin and CK-
MB angiography reveals a thrombosis of the left anterior descending coronary artery,
a) What is the most probable diagnosis?
b) What is the tissue response to ischemia?

ANS:

KMU Past Papers Solved SEQs


Cardiovascular System 28

a) ACUTE MYOCARDIAL INFARCTION


b) TISSUE RESPONSE TO ISCHEMIA
1. Within seconds of vascular obstruction, aerobic glycolysis ceases, leading to a drop
in adenosine triphosphate (ATP) and accumulation of potentially noxious
metabolites (e.g., lactic acid) in the cardiac myocytes.
2. functional consequence is a rapid loss of contractility, which occurs within a
minute or so of the onset of ischemia. Ultrastructural changes (including
myofibrillar relaxation, glycogen depletion, cell and mitochondrial swelling) also
become rapidly apparent.
3. These early changes are potentially reversible.
4. ischemia lasting at least 20 to 40 minutes causes irreversible damage and myocyte
death leading to coagulation necrosis and vessel injury ensues subsequently
microvascular thrombosis occur with longer periods of ischemia,
5. Thus, if myocardial blood flow is restored before irreversible injury occurs, cell
viability can be preserved
6. However, some time reperfusion injury occurs
7. Despite timely reperfusion, in the postischemic state, myocardium remains in non-
contractile state called stunned myocardium this defect is caused by persistent
abnormalities in cellular biochemistry it is severe enough to produce transient but
reversible cardiac failure.
8. Myocardial ischemia also contributes to arrhythmias, causing electrical instability
(irritability) of ischemic regions of the heart sudden cardiac death in the setting of
myocardial ischemia most often (in 80% to 90% of cases) is due to ventricular
fibrillation caused by myocardial irritability.
9. Irreversible injury of ischemic myocytes first occurs in the subendocardial zone
10. This region is especially susceptible to ischemia because it is the last area to
receive blood delivered by the epicardial vessels, and also because it is exposed to
relatively high intramural pressures, which act to impede the inflow of blood.
11. With more prolonged ischemia, a wavefront of cell death moves through other
regions of the myocardium, and involve the entire wall thickness (transmural
infarct) within 3 to 6 hours;
Additional:
Atherosclerosis classification according to American heart association:
i) Type I: Initial lesions
ii) Type II: Fatty streaks
iii) Type III: Intermediate lesions
iv) Type IV: Atheromatous lesions
v) Type V: Fibrofatty lesions
vi) Type VI: Complicated lesions

KMU Past Papers Solved SEQs


Hematopoietic and Lymphoid Systems 29

Q-1
A. Define Thalassemia.
B. Give the laboratory diagnosis of ß — Thalassemia major.
C. Give preventive measures for Thalassemia major.
ANS:
a) THALASSEMIA
The thalassemias are hereditary disorders in which there is reduced synthesis of one or
more of the globin polypeptide chains which give quantitative abnormalities of
polypeptide globin chain synthesis.
b) LABORATORY DIAGNOSIS
1. Blood findings:
i) Anemia, usually severe and mild in minor thalassemia.
ii) Blood film shows severe microcytic hypochromic red cell morphology, marked
anisopoikilocytosis, basophilic stippling, presence of many target cells, tear
drop cells and normoblasts.
iii) Reticulocytosis is generally present, mild in minor thalassemia.
iv) MCV, MCH and MCHC are significantly reduced, slightly reduced in minor
thalassemia.
v) WBC count is often raised with some shift to left.
vi) Platelet count is usually normal but may be reduced in patients with massive
splenomegaly.
1. Osmotic fragility reveals decreased osmotic fragility.
2. Serum bilirubin (unconjugated) is generally raised, mild or normal in minor
thalassemia.
3. Hemoglobin electrophoresis shows presence of increased amounts of HbF, increased
amount of HbA2, and almost complete absence or presence of variable amounts of
HbA.
4. In minor thalassemia there is two-fold increase in HbA2 and a slight elevation in HbF.
5. Bone marrow aspirate examination shows normoblastic erythroid hyperplasia with
predominance of inter mediate and late normoblasts.
c) PREVENTIVE MEASURES
Making an antenatal diagnosis by chorionic villous biopsy or cells obtained by
amniocentesis
fetal DNA studied by PCR amplification technique for presence of genetic mutations of
thalassemias.

Q-2
a) Enumerate causes of Macrocytic anemia,
b) Name Laboratory investigations for a suspected case of Megaloblastic anemia.

ANS:
a) CAUSES OF MACROCYTIC ANEMIA
A. Vitamin b12 deficiency
1. Inadequate dietary intake e.g. strict vegetarians, breast-fed infants.

KMU Past Papers Solved SEQs


Hematopoietic and Lymphoid Systems 30

2. Malabsorption
i) Gastric causes: pernicious anemia, gastrectomy, congenital lack of intrinsic
factor.
ii) Intestinal causes: tropical sprue, ileal resection, Crohn’s disease, intestinal
blind loop
syndrome, fish-tapeworm infestation.
B. Folate deficiency
1. Inadequate dietary intake e.g. in alcoholics, teenagers, infants, old age, poverty.
2. Malabsorption e.g. in tropical sprue, coeliac disease, partial gastrectomy,
jejunal resection, Crohn’s disease.
3. Excess demand
i) Physiological: pregnancy, lactation, infancy.
ii) Pathological: malignancy, increased hematopoiesis, chronic exfoliative skin
disorders, tuberculosis, rheumatoid arthritis.
4. Excess urinary folate loss e.g. in active liver disease, congestive heart failure.
C. other causes
1. Alcoholic liver diseases.
2. Impaired metabolism e.g. inhibitors of dihydrofolate (DHF) reductase such as
methotrexate and pyrimethamine; alcohol, congenital enzyme deficiencies.
3. Unknown etiology e.g. in Di Guglielmo’s syndrome, congenital
dyserythropoietic anemia, refractory megaloblastic anemia.
b) LABORATORY INVESTIGATIONS OF MEGALOBLASTIC ANEMIA
A. General laboratory findings
1. Blood picture and red cell indices:
i) Hemoglobin is decreased.
ii) Macrocytosis. (egg shaped) macro-ovalocytes)
iii) Reticulocyte count is low to normal.
iv) Elevated mcv (above 120 fl) elevated MCH (above 50 pg) and normal or reduced
MCHC.
v) Hyper segmented neutrophils (having more than 5 nuclear lobes)
vi) Platelets are reduced large and misshapen
2. Bone marrow findings:
i) Hypercellular BM with a decreased myeloid-erythroid ratio.
ii) Erythroid hyperplasia with megaloblasts.
iii) Giant forms of metamyelocytes and band cells may be present in the marrow
iv) Megakaryocytes with bizarre multi-lobulated nuclei.
v) Marrow iron prussian blue staining shows increase iron granules in the erythroid
precursors.
3. Biochemical findings
i) Rise in serum unconjugated bilirubin and LDH
ii) Decreased heptoglobin
iii) The serum iron and ferritin may be normal or elevated.
B. Special tests
1. For vitamin b12 deficiency:
i) Microbiological assay using vitamin B12 sensitive microorganism (e. Gracilis)

KMU Past Papers Solved SEQs


Hematopoietic and Lymphoid Systems 31

ii) Radioassay assays of serum B12 by radioisotope dilution (RID) and


radioimmunoassay (RIA)
iii) Schilling test (24-hour urinary excretion test)
iv) Enzymatic assays show elevated methylmalonic acid and homocysteine.
v) Serum antibodies to intrinsic factor.
vi) Dramatic response to parenteral administration of vitamin B12
2. Tests for folate deficiency
i) Measurement of formiminoglutamic acid (FIGLU) urinary excretion (histidine
loading test)
ii) Serum and red cell folate assay.

Q-3
a) Classify Hodgkin's disease
b) Give a brief account of Burkitt's lymphoma
c) Give microscopic appearance of Nodular sclerosis

ANS:
a) CLASSIFICATION (RYE CLASSIFICATION)
1. Lymphocyte-predominance type
2. Nodular-sclerosis type
3. Mixed-cellularity type
4. Lymphocyte-depletion type
5. lymphocyte rich
(WHO classification)
1. Nodular lymphocyte-predominant HD (a new type)
2. Classic HD (includes all the 4 above subtypes in the Rye classification)

b) BURKITT’S LYMPHOMA
Burkitt’s lymphoma/leukemia is an uncommon Tumor in adults but comprises about
30% of childhood NHLs.
Burkitt’s leukemia corresponds to L3 ALL of FAB grouping and is uncommon. Three
subgroups of Burkitt’s lymphoma are recognized:
1. African endemic,
2. sporadic and
3. immunodeficiency-associated.
On M/E All three types of Burkitt’s lymphoma are similar. Tumor cells are intermediate in
size, non-cleaved, and homogeneous in size and shape. The nuclei are round to oval and
contain 2-5 nucleoli. The cytoplasm is basophilic and contains lipid vacuolation. The
Tumor cells have a very high mitotic rate, and therefore high cell death. This feature
accounts for presence of numerous macrophages in the background of this Tumor
containing phagocytosed Tumor debris giving it a ‘starry sky’ appearance. Burkitt’s
lymphoma is a high-grade Tumor and is a very rapidly progressive human Tumor.
c) NODULAR SCLEROSIS MORPHOLOGY
i) Bands of collagen: Variable amount of fibrous tissue is characteristically present in
the involved lymph nodes.

KMU Past Papers Solved SEQs


Hematopoietic and Lymphoid Systems 32

ii) Lacunar type RS cells: Characteristic lacunar type of RS cells with distinctive
pericellular halo are present a, pericellular space or lacuna in which it lies, which is
due to artefactual shrinkage of the cell cytoplasm.
iii) Other cells: eosinophils neutrophils and lymphocytes.

Q-4
a) What is Neutropenia? Enumerate its causes
b) Enumerate the causes of Eosinophilia
c) What are the laboratory findings of Chronic myeloid leukemia (CML)
ANS:
a) NEUTROPENIA
Definition absolute neutrophil count <1500 cells/mm3
Causes
1) Aplastic anemia
2) Immune destruction
•Examples systemic lupus erythematosus (SLE), and paroxysmal nocturnal
hemoglobinuria
3) Septic shock
4) Drugs
 Penicillin, cephalosporins, quinidine, and sulfonamides
5) Tick-borne diseases
 Ehrlichiosis, anaplasmosis, and babesiosis
6) Viral infections
 Hepatitis, infectious mononucleosis
7) Bacterial infections
 Typhoid fever, brucellosis, and tuberculosis
8) Fungal infections
 Systemic fungal infections (e.g., histoplasmosis)
9) Ionizing radiation
10)Certain rare hereditary, congenital or familial disorders

b) EOSINOPHILIA
Definition absolute eosinophil count >400 cells/mm3
Causes
1) Type I hypersensitivity reaction
 Examples are bronchial asthma, reaction to penicillin, hay fever, and atopic
dermatitis
2) Protozoal disease
 Dientamoeba fragilis (only protozoal infection with eosinophilia)
3) Invasive helminthic infection
a. Examples strongyloidiasis and hookworm infection
b. Pinworms and adult ascariasis are not accompanied by eosinophilia (noninvasive).
4) Polyarteritis nodosa, Churg-Strauss syndrome
5) Addison disease (cortisol deficiency)
6) Job syndrome (mutation affecting TH17 cells and patient suitable to muco-cutaneous
candidiasis)

KMU Past Papers Solved SEQs


Hematopoietic and Lymphoid Systems 33

7) Löeffler’s syndrome. (eosinophilic pneumonitis)


8) Pulmonary infiltration with eosinophilia (PIE) syndrome.
9) Tropical eosinophilia.
c) LAB DIAGNOSIS OF CML
GIVEN SOMEWHERE ELSE**

Q-5A 40 years old female presented in the OPD with complaints of easy fatigability, weakness and
weight loss. She also complained of dragging sensation in the abdomen. On examination she
had marked splenomegaly. On peripheral blood examination, TLC was 250,000/cm, platelet
count 450,000/cmm. Smear showed predominance of myelocytes, metamyelocytes and
neutrophils.
a) What is the provisional diagnosis?
b) Write the salient features of pathophysiology.
c) What other investigations would you do?
d) What are different phases of the disease?
e) What transformation occurs in this disorder?

ANS:
a) CHRONIC MYELOID LEUKEMIA
b) PATHOPHYSIOLOGY
Neoplastic clonal expansion of the pluripotential stem cell. These cell possesses the
balanced reciprocal translocation between chromosomes 9 and 22, forming Philadelphia
chromosome. The t(9;22) involves fusion of BCR (breakpoint cluster region) gene on
chromosome 22q11 with ABL (named after Abelson murine Leukemia virus) gene
located on chromosome 9q34. The fusion product so formed is termed “Ph chromosome
t(9;22) (q34;11), BCR/ABL
c) INVISTIGATIONS
A. Blood findings
1. Normocytic to macrocytic anemia
2. Marked leukocytosis
3. Blast cells present may it may be
4. < 10% chronic phase
> 10% accelerated phase
>20 % blast phase
5. Platelets count are raised
6. No auer rods are seen and there is basophilia
B. Imaging:
1. x- ray abdomen
2. ultrasound abdomen
C. Bone marrow findings
1. Hypercellular with myeloblasts < 10%
2. Increased myeloid erythroid ratio
3. Decrease erythrocytes and increased reticulin
D. Cytogenetics
1. Karyotyping for Philadelphia chromosome
2. BCR ABL fusion gene using PCR and FISH

KMU Past Papers Solved SEQs


Hematopoietic and Lymphoid Systems 34

E. Cytochemistry
1. Decreased leukocyte alkaline phosphatase (LAP) score (increase in
lukemoid reactions)
F. Other:
1. Increased serum B12 and uric acid
d) PHASES
1. Chronic phase of CML:
Begins as a myeloproliferative disorder and consists of excessive proliferation of
myeloid cells of intermediate grade (i.e. myelocytes and metamyelocytes) and
mature segmented neutrophils. Myeloblasts are LESS THAN 10% of cells in the
peripheral blood and bone marrow.
2. An accelerated phase of CML:
Is also described in which there is progressively rising leukocytosis associated
with thrombocytosis or thrombocytopenia and splenomegaly.
Myeloblasts are MORE THAN 10% of cells in the peripheral blood and bone
marrow.
3. Blastic phase or blast crisis:
It fulfills the definition of acute leukemia in having blood or marrow blasts MORE
THAN 20%. These blast cells may be myeloid, lymphoid, erythroid or
undifferentiated.
4. Spent phase or phase of extensive bone marrow fibrosis
e) TRANSFORMATION:
Transformation to acute leukemia
1. 70% to AML
2. 30% to ALL

Q-6
a) Classify Hodgkin lymphoma.
b) Draw and label the histological picture.
c) Write the salient features of etiology and pathogenesis.

ANS:
a) CLASSIFICATION (RYE CLASSIFICATION)
1. Lymphocyte-predominance type
2. Nodular-sclerosis type
3. Mixed-cellularity type
4. Lymphocyte-depletion type
5. lymphocyte rich
(WHO classification)
1. Nodular lymphocyte-predominant HD (a new type)
2. Classic HD (includes all the 4 above subtypes in the Rye classification)

b) HISTOLOGY OF HODGKIN LYMPHOMA


G/A Initially, the lymph nodes are discrete and separate from one another but later the
lymph nodes form a large matted mass due to infiltration into the surrounding connective

KMU Past Papers Solved SEQs


Hematopoietic and Lymphoid Systems 35

tissue. The sectioned surface of the involved lymph nodes or extra-nodal organ involved
appears grey-white and fishflesh-like.
M/E The findings are as under:
1. Nodular Lymphocyte-predominance type characterized by
proliferation of small lymphocytes admixed with a varying number of histiocytes
atypical RS cells, polyploid variant having polyploid, and twisted nucleus
(popcornlike).
2. Nodular-sclerosis:
Bands of collagen: Variable amount of fibrous tissue
Lacunar type RS cells: Characteristic lacunar type of RS cells with distinctive
pericellular halo are present.
3. Mixed-cellularity type:
proliferating lymphocytes, histiocytes, eosinophils, neutrophils and plasma cells.
Typical RS cells are frequent.
4. Lymphocyte-depletion:
Lymphocytes are depleted
With diffuse fibrosis and infiltration by atypical histiocytes (Hodgkin cells), and
numerous typical and atypical (pleomorphic) RS cells.
5. lymphocyte- rich:
c) ETIOLOGY AND PATHOGENESIS
Etiology: HIV and ebstine Barr virus (EBV) infection
Pathogenesis:
1. Genetic susceptibility underlies HL in children.
2. NF-κB (nuclear factor kappa-light chain-enhancer of activated B cells) is activated by
EBV or other factors.
3. Activation of the transcription factor NF-κB is common in classical HL.
4. Once activated, it turns on genes that promote proliferation of B cells.
5. RS cells secrete IL-5, IL-13 and TGF β. IL-5 stimulate eosinophils while IL-13 stimulate
RS cells.

Q-7A 25 years old female patient presented with marked pallor, weakness, tachycardia and
dyspnea on mild exertion. On physical examination she has angular stomatitis and
koilonychias. She gives past history of peptic ulceration.
a) Name the most likely diagnosis.
b) Name the laboratory investigations you would suggest.
c) Justify how these investigations will help you reach the diagnosis.
ANS:
a) IRON DEFICIENCY ANEMIA:
b) LAB INVESTIGATIONS WITH JUSTIFICATIONS:
1. Blood picture and red cell indices
i) decreased Hb
ii) hypochromic microcytic cells, anisocytosis, poikilocytosis Target cells.
iii) Reticulocyte normal or reduced.
iv) decreased MCV, MCH and MCHC
v) Leucocytes and DLC normal.
vi) Platelets Platelet, normal.

KMU Past Papers Solved SEQs


Hematopoietic and Lymphoid Systems 36

2. Bone marrow findings


i) erythroid hyperplasia (myeloid-erythroid ratio decreased).
ii) predominance of small polychromatic normoblasts (micro normoblasts) other
cells are normal.
iii) Marrow iron staining (Prussian blue reaction) on bone marrow aspirate smear
shows deficient reticuloendothelial iron stores.
3. Biochemical findings
i) The serum iron level is low
ii) Total iron binding capacity (TIBC) is high
iii) Serum ferritin is very low
iv) Red cell protoporphyrin is very low
v) Serum transferrin receptor protein is raised as determined by immunoassay.

Q-8
a) Enumerate transfusion reactions.
b) Discuss Rh. Incompatibility.

ANS:
a) TRANSFUSTION REACTIONS
A. Immunologic transfusion reactions
1. Hemolytic transfusion reactions Hemolytic transfusion reaction may be immediate
or delayed, intravascular or extravascular.
2. Transfusion related acute lung injury (TRALI) This is an uncommon reaction
resulting from transfusion of donor plasma containing high levels of anti-HLA
antibodies which bind to leucocytes of recipient. These leucocytes then aggregate
in pulmonary micromutation and release mediators of increased vascular
permeability resulting in acute pulmonary edema and signs and symptoms of
respiratory failure.
3. Other allergic reactions These are:
a. Febrile reaction
b. Anaphylactic shock
c. Allergic reactions
d. Graft-versus-host disease.
B. Nonimmune transfusion reactions these include:
1. Circulatory overload
2. Massive transfusion
3. Transmission of infection
4. Air embolism
5. Thrombophlebitis
6. Transfusion hemosiderosis
b) RH INCOMPATIBILITY
Rh-D incompatibility Rh incompatibility occurs when a Rh negative mother is sensitized
to Rh-positive blood. This results most often from a Rh-positive fetus by passage of Rh-
positive red cells across the placenta into the circulation of Rh-negative mother. The risk
of sensitization of a Rh negative woman married to Rh-positive man is small in first
pregnancy but increases during successive pregnancies if prophylactic anti-D

KMU Past Papers Solved SEQs


Hematopoietic and Lymphoid Systems 37

immunoglobulin is not given within 72 hours after the first delivery. The HDN due to Rh-
D incompatibility in its severest form may result in intrauterine death from hydrops
foxtails. Moderate disease produces a baby born with severe anemia and jaundice due to
unconjugated hyperbilirubinemia. When the level of unconjugated bilirubin exceeds 20
mg/dl, it may result in deposition of bile pigment in the basal ganglia of the CNS called
kernicterus and result in permanent brain damage.
LABORATORY FINDINGS
1. Cord blood shows variable degree of anemia, reticulocytosis, elevated serum
bilirubin and a positive direct Coombs’ test if the cord blood is Rh-D positive.
2. Mother’s blood is Rh-D negative with high plasma titer of anti-D.

Q-9A 60 years old female complaining of aches pains, on blood CP shows normocytic,
normochromic anemia, Hb 8.0gm/dl ESR 110mm at end of IS t hour. The peripheral blood
smear also shows Rouleau formation.
a) What is your provisional diagnosis?
b) What investigations would you do to confirm the diagnosis?
c) Write the salient features of etiology and pathogenesis.
d) What are the main clinic pathological features of this disease?

ANS:
a) MULTIPLE MYLOMA
b) INVESTIFATIONS
1. Blood:
Anemia, increased ESR, atypical plasma cells.
blood smear for type of paraproteins and Rouleau formation.
2. Urine examination for albumin, glucose, microscopy and for Bence-Jones proteins
3. Biochemical estimation:
Renal function tests (urea, creatinine, BUN),
liver function tests (bilirubin, ALT, AST, alkaline phosphatase, total proteins and
AG ratio),
blood glucose, lipid profile, calcium, phosphate.
4. Radiograph
Skull vertebral column, ribs, scapula, pelvis and femur for lytic bony lesions.
5. Ultrasound examination of abdomen
6. CT-scan of the pelvis and abdomen
7. Bone marrow examination:
Atypical plasma cells more than 10 % having multiple nuclei and cytoplasmic
granules (Russells bodies) containing immunoglobulins. These cells also lack cart
wheel chromatin pattern.
8. Serum and urine electrophoresis shows
M band (gamma globulin)
Bens jhones protein (light chain kappa and lambda)
c) ETIOLOGY AND PATHOGENESIS
1. Radiation exposure
2. Epidemiologic factors higher incidence in blacks.
3. Occupational exposure to petroleum products

KMU Past Papers Solved SEQs


Hematopoietic and Lymphoid Systems 38

4. Karyotypic abnormalities Several chromosomal alterations ie


i) Translocations t(11;14) t(4;14) (q13;q32) and (p16;q32).
ii) Deletion of 13q.
5. Oncogenes anti-oncogenes
i) Overexpression of MYC and RAS growth promoting oncogenes in some cases.
ii) Mutation in p53 and RB growth-suppressing anti-oncogene in some cases.

PATHOGENESIS:
1. Cell-surface adhesion molecules bind myeloma cells to bone marrow stromal cells and
extracellular matrix proteins.
2. This binding triggers production of several cytokines (IL-6, VEGF, TGF-b, TNF-a IL-1,
lymphotoxin, macrophage inhibitory factor-1a (MIP-1a) and RANK ligand) by
fibroblasts and macrophages of the marrow.
3. Myeloma (M) proteins production cell via Cyclin-D and p21.
4. IL-6 cytokine having antiapoptotic effects on Tumor cells.
5. The above cytokines produced by myeloma cells bring about bony destruction by
acting as osteoclast-activating factor (OAF).
d) CLINICOPATHOLOGICAL FEATURES
1. Bone pain is the most common symptom.
2. Susceptibility to infections
3. Renal failure
4. Anemia occurs in about 80% of patients
5. Bleeding tendencies
6. Hyperviscosity syndrome may produce headache, fatigue, visual disturbances and
hemorrhages.
7. Neurologic symptoms.
8. Biochemical abnormalities. These include the following:
i) hypercalcemia due to destruction of bone;
ii) hyperuricemia from necrosis of Tumor mass and from uremia related to renal
failure; and iii) increased b-2 microglobulins and other globulins in urine and
serum.
9. POEMS syndrome (polyneuropathy, organomegaly, endocrinopathy, multiple
myeloma and skin changes)

Q-10
a) Define Thalassemia,
b) Give the types of thalassemia,
c) How will you investigate a case of thalassemia in the lab?

ANS:
a) THALASSEMIA
The thalassemias are hereditary disorders in which there is reduced synthesis of one or
more of the globin polypeptide chains which give quantitative abnormalities of
polypeptide globin chain synthesis.
b) TYPES OF THALASSEMIA:
1. α-thalassaemias

KMU Past Papers Solved SEQs


39

i) Hydrops foetalis
ii) Hb-H disease
iii) α-Thalassaemia trait
2. β-thalassaemias
i) β -Thalassaemia major
ii) β -Thalassaemia intermedia
iii) β -Thalassaemia minor
c) LABORATORY DIAGNOSIS
Given somewhere else**

Q-11
a) Define Aplastic Anemia
b) Enumerate the causes of Aplastic Anemia
c) Give the salient features in pathogenesis of Aplastic Anemia

ANS:
a) APLASTIC ANEMIA
Aplastic anemia is defined as pancytopenia, simultaneous presence of anemia, leucopenia
and thrombocytopenia resulting from aplasia of the bone marrow.
b) CAUSES
A. Primary aplastic anemia
1. Fanconi’s anemia This has an autosomal recessive inheritance
2. Immune causes: suppression of hematopoietic stem cells immune system
B. Secondary aplastic anemia
Drugs:
1. Dose-related aplasia with methotrexate daunorubicin, busulfan nitroso urea,
anthracyclines
2. Idiosyncratic aplasia e.g. with chloramphenicol, sulfa drugs, oxyphenbutazone,
phenylbutazone, chlorpromazine, gold salts
3. Toxic chemicals: benzene derivatives, insecticides, arsenicals
4. Infections: viral hepatitis, Epstein-Barr virus infection, AIDS and other viral
illnesses.
5. Miscellaneous: SLE, and with therapeutic X-rays.
c) PATHOGENESIS
All cases appear to be sufficient reduction in the number of hematopoietic pluripotent
stem cells which results in decreased or total absence of these cells for division and

Q-12
a) Enumerate causes of iron deficiency Anemia
b) Briefly describe lab diagnosis of iron deficiency anemia

differentiation.
ANS:
a) CAUSES OF IRON DEFICIENCY ANEMIA
1. Increased blood loss:

KMU Past Papers Solved SEQs


Hematopoietic and Lymphoid Systems 40

a) Uterine e.g. excessive menstruation in reproductive years, repeated miscarriages,


at onset of menarche, post-menopausal uterine bleeding
b) Gastrointestinal e.g. peptic ulcer, Haemorrhoids hookworm infestation, cancer of
stomach and large bowel, Oesophageal varices, hiatus hernia, chronic aspirin
ingestion, ulcerative colitis, diverticulosis
c) Renal tract e.g. hematuria, hemoglobinuria
d) Nose e.g. repeated epistaxis
e) Lungs e.g. hemoptysis
2. Increased requirements
a) Spurts of growth in infancy, childhood and adolescence
b) Prematurity
c) Pregnancy and lactation
3. Inadequate dietary intake
a) Poor economic status
b) Anorexia e.g. in pregnancy
c) Elderly individuals due to poor dentition, apathy and financial constraints
4. Decreased absorption
a) Partial or total gastrectomy
b) Achlorhydria
c) Intestinal malabsorption such as in coeliac disease
b) LAB DIAGNOSIS
GIVEN SOMEWHERE ELSE**
Q-13
a) Enlist the causes of macrocytosis
b) Write the blood and bone marrow finding in megaloblastic anemia
ANS:
a) CAUSES OF MACROCYTOSIS
1. Vitamin b12 deficiency
2. Folate deficiency
3. Drugs which interfere with DNA synthesis
4. Acquired defects of hematopoietic system
5. Congenital enzyme deficiencies
6. Hemolysis
7. Liver disease
8. Chronic alcoholism
9. Hypothyroidism
10. Aplastic anemia
11. Myeloproliferative disorders
12. Reticulocytosis
b) LAB DIAGNOSIS OF MEGALOBLASTIC ANEMIA

KMU Past Papers Solved SEQs


Hematopoietic and Lymphoid Systems 41

GIVEN SOMEWHERE ELSE**

Q-14
a) Which leukemia is more common in children?
b) Write its subtypes according to FAB classification
c) Write the blood and bone marrow findings along with the cytochemistry, immunological
markers and the chromosomal abnormalities

ANS
a) ACUTE LYMPHOBLASTIC LEUKEMIA (ALL)
b) FAB CLASSIFICATION:
i) L1. Childhood-ALL (B-ALL, and T-ALL)
ii) L2. Adult-ALL (mostly T-ALL)
iii) L3. Burkitt type-ALL (B-ALL)
c) LAB DIAGNOSIS
i) Blood examination:
Anemia, neutropenia, thrombocytopenia, smear cells are found on peripheral
smear.
DLC show lymphoblast with rounded nuclei with coarse clumped chromatin,
increased nuclear cytoplasmic ratio and absence of granules and no Aur Rods.
ii) Bone marrow examination:
20 to 95% malignant precursor B cells and T cells, reduced megakaryocytes.
iii) Cytochemistry:
1. PAS positive (because lymphoblast contain cytoplasmic glycogen granules)
2. Acid phosphatase positive
3. MPO negative
4. Sudan black negative
5. Nonspecific esterase negative
iv) Immunological markers:
1. TdT positive both pre- B and pre- T cells
2. Pre- B cells markers: CD 19, CD 10, CD 9a
3. Pre- T cells markers: CD 1, CD 2, CD 3, CD 5, CD 7.
v) Chromosomal abnormalities/ cytogenetic abnormalities:
1. Cryptic (12:21) translocation TEL1 and AML1 fusion gene formation, (Good
prognosis)
2. 9:22 translocation, ABL and BCR fusion gene formation, (Bad prognosis)
3. Hyperdiploidy; more than 50 chromosomes per cell (Good prognosis)
PEARLS: Other Bad prognostic features
1. Male gender
2. Adult
3. Increased lymphoblasts
4. CNS involvement

KMU Past Papers Solved SEQs


Hematopoietic and Lymphoid Systems 42

Q-15
a) Define anemia
b) Enumerate common causes of iron deficiency anemia
c) What are the blood and bone marrow findings in iron deficiency anemia?

ANS:
a) ANEMIA
Anaemia is defined as reduced haemoglobin concentration in blood below the lower limit
of the normal range for the age and sex of the individual. In adults, the lower extreme of
the normal haemoglobin is taken as 13.0 g/dl for males and 11.5 g/dl for females.
b) CAUSES OF IRON DEFICIENCY ANEMIA
1. Increased blood loss:
a) Uterine e.g. excessive menstruation in reproductive years, repeated miscarriages,
at onset of menarche, post-menopausal uterine bleeding
b) Gastrointestinal e.g. peptic ulcer, haemorrhoids hookworm infestation, cancer of
stomach and large bowel, oesophageal varices, hiatus hernia, chronic aspirin
ingestion, ulcerative colitis, diverticulosis
c) Renal tract e.g. haematuria, haemoglobinuria
d) Nose e.g. repeated epistaxis
e) Lungs e.g. haemoptysis
2. Increased requirements
a) Spurts of growth in infancy, childhood and adolescence
b) Prematurity
c) Pregnancy and lactation
3. Inadequate dietary intake
a) Poor economic status
b) Anorexia e.g. in pregnancy
c) Elderly individuals due to poor dentition, apathy and financial constraints
4. Decreased absorption
a) Partial or total gastrectomy
b) Achlorhydria
c) Intestinal malabsorption such as in coeliac disease

c) BONE MARROW FINDINGS IN IRON DEFICIENCY ANEMIA:


Bone marrow findings
1. erythroid hyperplasia (myeloid-erythroid ratio decreased).
2. predominance of small polychromatic normoblasts (micro normoblasts) other cells
are normal.
3. Marrow iron Iron staining (Prussian blue reaction) on bone marrow aspirate smear
shows deficient reticuloendothelial iron stores.

KMU Past Papers Solved SEQs


Hematopoietic and Lymphoid Systems 43

Q-16 A 40-year-old female presented in the OPD with complaints of easy fatigability, weakness
and weight loss. She also complained of dragging sensation in the abdomen. On examination
she had marked splenomegaly. On peripheral blood examination 250,000/cm, platelet count
450,000/gm, and smear showed predominance of myelocytes, metamyelocytes, and
neutrophils.
a) What is the most provisional diagnosis?
b) Write the salient features of the pathophysiology?
c) What other investigations would you do?
d) What transformation occurs in this disorder?

ANS:
a) CHRONIC MYELOID LEUKEMIA
b) PATHOPHYSIOLOGY
Neoplastic clonal expansion of the pluripotential stem cell. These cell possesses the
balanced reciprocal translocation between chromosomes 9 and 22, forming Philadelphia
chromosome. The t(9;22) involves fusion of BCR (breakpoint cluster region) gene on
chromosome 22q11 with ABL (named after Abelson murine Leukemia virus) gene
located on chromosome 9q34. The fusion product so formed is termed “Ph chromosome
t(9;22) (q34;11), BCR/ABL
c) INVISTIGATIONS
GIVEN SOMEWHERE ELSE**
d) TRANSFORMATION
GIVEN SOMEWHERE ELSE**

Q-17 A 9 year old African boy has increasing pain and swelling on the right side of face for last 8
month. On examination there is a large non-tender mass involving the mandible. There is no
lymphadenopathy and no splenomegaly. Biopsy reveals intermediate sized malignant cells
with high mitotic index and increased histiocytic
(a) What is the most likely diagnosis? 1
(b) What are the different types? 1.5
(c) List the principal microscopic features. 4
(d) Write the immunological markers and chromosomal translocations seen & viral agent
involved in this disorder. 1+1.5+1

ANS:
a) BURKITT LYMPHOMA
b) TYPES
1. African Endemic
2. Sporadic.
3. Immunodeficiency associated
c) MORPHOLOGY:
1. Tumor cell have oval nuclei with 2-5 distnct nucleoli
2. Basophilic cytoplasm with lipid filled vacuoles
3. High mitotic figures and apoptosis

KMU Past Papers Solved SEQs


Hematopoietic and Lymphoid Systems 44

4. Tissue macrophages ingested with nuclear debris.


5. macrophages when surrounded by clear space give pattern called “starry sky”
d) IMMUNOLOGICAL MARKERS CHROMOSOMAL ABNORMALITY AND VIRUS INVOLVE
Pan-B cell markers: CD19 and 20
Germinal B cell markers: CD 10 and BCL6

Chromosome 8 and 14 translocations (MYC IgH fusion gene)


Chromosome 8 and 22 translocations
Chromosome 8 and 2 translocations

Ebstin bar virus (EBV)

Q-18
(a) Define thalassemia syndrome. 2
(b) Write down the pathophysiology of skeletal defects in Beta thalassemia. 2
(c) Give the lab diagnosis of thalassemia ma or (blood, BM, Hb Electrophoresis) 3
(d) Make a flow chart to explain the pathogenesis of Beta thalassemia major. 3

ANS:
a) THALASSEMIA SYNDROME
The thalassemias are hereditary disorders in which there is reduced synthesis of one or
more of the globin polypeptide chains which give quantitative abnormalities of
polypeptide globin chain synthesis.
b) PATHOPHYSIOLOGY:
Anemia  tissue hypoxia  increase erythropoietin  hyperplasia of erythroid
progenitors due to ineffective erythropoiesis  bone marrow expansion  fill
intramedullary space of skeleton and invade bone cortex
c) LABORATORY DIAGNOSIS
GIVEN SOMEWHERE ELSE**
PATHOGENESIS:
Most of β-thalassaemias arise from different types of mutations of b-globin gene resulting
from single base changes. Leads to either complete absence of β-globin chain synthesis or
partial synthesis of the β-globin chains.
Some of the important mechanism having effects on β-globin chain synthesis are:
i) Transcription defect
ii) Translation defect
iii) mRNA splicing defect
Thalassaemia major (Homozygous form) have following globin chain features;
i) β ° thalassaemia major characterised by complete absence of β -chain synthesis.
ii) β + thalassaemia major having incomplete suppression of β -chain synthesis.
These result in excessive formation of alternate hemoglobin, HbF (a2g2) and HbA2
(a2d2).
These form alpha 4 tetramers in RBC called alpha chain inclusions which are removed by
splenic macrophages,

KMU Past Papers Solved SEQs


Hematopoietic and Lymphoid Systems 45

RBCs with alpha chain inclusion undergo apoptosis in the bone marrow (ineffective
erythropoiesis)

Q-19
a) Define anemia 1
b) Give classification of anemia on the basis of red blood cell morphology, giving 3 examples
of each type 4
c) Give differential diagnosis of microcytic hypochromic anemia in tabulated form 5

ANS:
a) ANEMIA
Anemia is defined as reduced hemoglobin concentration in blood below the lower limit
of the normal range for the age and sex of the individual. In adults, the lower extreme of
the normal hemoglobin is taken as 13.0 g/dl for males and 11.5 g/dl for females.
b) CLASSIFICATION
1. Microcytic, hypochromic MCV, MCH, MCHC are all reduced
e.g. in iron deficiency anemia, sideroblastic anemia, thalassemia, anemia of chronic
disorders.
2. Normocytic, normochromic MCV, MCH, MCHC are all normal
3. e.g. blood loss anemia, hemolytic anemias, bone marrow failure, anemia of chronic
disorders.
4. Macrocytic MCV is raised
5. e.g. in megaloblastic anemia due to deficiency of vitamin B12 or folic acid.
c) DIFFERENTIAL DIAGNOSIS OF MICROCYTIC HYPOCHROMIC ANEMIA
1. Iron deficiency anemia,
2. Sideroblastic anemia,
3. Thalassemia,
4. Anemia of chronic disorders

Q-20 A Fifty eight -year old male patient presented with fever, weight loss and abdominal
distension for the last 6 months. His physical examination revealed generalized
lymphadenopathy and enlarged spleen, 8cm below the costal margin, Complete blood counts
revealed Hemoglobin = 12 g/dl, white blood cell count = 120 103/microliter and platelet
count"; 150 x103/microliter. His differential leukocyte count revealed 80% lymphoid cells.
a) What is the most likely diagnosis? 1
b) How will you investigate to further to confirm the diagnosis? 5
c) Write down the commonly used staging system of this disease 4

ANS:
a) CHRONIC LYMPHOCITIC LEUKEMIA:
b) INVESTIGATIONS:
i) Blood:
decreased hemoglobin, coombs test positive in 20% patients
DLC shows lymphocytes 90% or 80% ??
Smudge cells or basket cells in peripheral smear.
Reduced platelets count

KMU Past Papers Solved SEQs


Hematopoietic and Lymphoid Systems 46

ii) Bone marrow findings


Increased lymphocytes decreased myeloid and erythroid cells
iii) Lymph node biopsy
Diffuse proliferation of mature and well differentiated lymphocytes with no atypia
iv) Other tests
Erythrocyte rossete test is positive
v) Markers
CD 5, CD 19 CD 20 CD 23.
Surface immunoglobulin (light chain and heavy chain)
vi) Cytogenetic abnormality:
12 chromosome trisomy.

c) STAGING SYSTEM:
Binet classification
1. Stage A
a. Lymphocytosis
b. Less than three areas of lymphoid enlargement
c. No anemia
d. No thrombocytopenia
2. Stage B
a. Lymphocytosis
b. More than three area of lymphoid enlargement
c. No anemia
d. No thrombocytopenia
3. Stage C
a. Lymphoctosis
b. Regardless of area
c. Anemia
d. Thrombocytopenia

Q-21 A 45 years old male presents with fever, weakness and generalized body aches. On
examination there is massive splenomegaly. CBC shows a WBC count of 135000/mm3 with
2% myeloblast, 3% promyelocytes, 18% myelocytes along with stab mature neutrophils.
There is also thrombocytosis. Bone marrow examination shows myeloid hyperplasia.
a) What is the most likely diagnosis? 1
b) What further investigations would you do to confirm the diagnosis? 4
c) What are the different phases of the disease? 1.5
d) How would you differentiate the above condition from leukemoid reaction? 3.5

ANS:
a) CHRONIC MYELOID LEUKEMIA
b) INVESTIGATIONS:
GIVEN SOMEWHERE ELSE**
c) PHASES
1. Chronic phase: blasts < 10%
2. Accelerated phase: blasts > 10%

KMU Past Papers Solved SEQs


Hematopoietic and Lymphoid Systems 47

3. Blast phase: blasts > 20 %


4. Spent phase
d) In leukemoid reaction the NAP (neutrophil alkaline phosphatase) score is high
While in chronic myeloid leukemia NAP score is decreased.

Q-22
a) Define Anemia. What are the normal reference ranges Of the Hemoglobin for adult
males and females? 1+1
b) Enumerate the biochemical investigations for the diagnosis of iron deficiency
Anemia.3
c) Describe the peripheral blood and bone marrow morphology Of Megaloblastic
Anemias 5
ANS:
a) ANEMIA:
Anemia is defined as reduced hemoglobin concentration in blood below the lower limit of
the normal range for the age and sex of the individual.
Hb male 13.2 – 15.0 g/dl
Hb female 11.9 – 15 g/dl

b) LAB DIAGNOSIS:
GIVEN SOMEWHERE ELSE**
c) BLOOD AND BONE MARROW MORPHOLOGY OF MEGALOBLASTIC ANEMIA
A. Blood picture and red cell indices:
1. Hemoglobin is decreased.
2. Macrocytosis. (However, macro cytosis can also be seen in several other disorders
such as: hemolysis, liver disease, chronic alcoholism, hypothyroidism, aplastic
anemia, myelo proliferative disorders and reticulocytosis)
3. Reticulocyte count is low to normal.
4. Elevated MCV (above 120 fl) elevated MCH (above 50 pg) and normal or reduced
MCHC.
5. Hyper segmented neutrophils (having more than 5 nuclear lobes)
6. Platelets reduced
B. Bone marrow findings:
1. Hypercellular BM with a decreased myeloid-erythroid ratio.
2. Erythroid hyperplasia with Megaloblasts.
3. Giant forms of metamyelocytes and band cells may be present in the marrow.
4. Marrow iron Prussian blue staining shows increase iron granules in the erythroid
precursors.
Q-23
a) Define Thrombocytopenia.
b) What are the causes of thrombocytopenia?
c) Comments on pathogenesis involved in immune thrombocytopenic purpura (ITP)

ANS:
a) THROMBOCYTOPENIA

KMU Past Papers Solved SEQs


Hematopoietic and Lymphoid Systems 48

Thrombocytopenia is defined as a reduction in the peripheral blood platelet count below


the lower limit of normal i.e. below 150,000/µl.
b) CAUSES
A. Impaired platelet production:
1. Generalized bone marrow failure e.g. Aplastic anemia, Leukemia, myelofibrosis,
megaloblastic anemia,
2. marrow infiltrations (carcinomas, lymphomas, multiple myeloma, storage
diseases).
3. Selective suppression of platelet production e.g. Drugs (quinine, quinidine,
sulfonamides, PAS, rifampicin, anticancer drugs, thiazide diuretics), (heparin,
diclofenac, acyclovir), alcohol intake.
B. Accelerated platelet destruction
1. Immunologic thrombocytopenia e.g. ITP (acute and chronic), neonatal and post-
transfusion (isoimmune), drug-induced, secondary immune thrombocytopenia
(post-infection, SLE, AIDS, CLL, lymphoma).
2. Increased consumption e.g. DIC, TTP, giant hemangiomas, microangiopathic
hemolytic anemia.
C. Splenic sequestration
1. Splenomegaly
D. Dilutional loss
1. Massive transfusion of old stored blood to bleeding patients

c) PATHOGENESIS OF ITP
ACUTE ITP: This is a self-limited disorder, seen most frequently in children following
recovery from a viral illness (e.g. hepatitis C, infectious mononucleosis, CMV infection,
HIV infection) or an upper respiratory illness. The onset of acute ITP is sudden and
severe thrombocytopenia but recovery occurs within a few weeks to 6 months.
CHRONIC ITP: Chronic ITP occurs more commonly in adults, particularly in women of
child-bearing age (20-40 years) . There is association with SLE, AIDS and autoimmune
thyroiditis. There is formation of anti-platelet autoantibodies, Antibodies directed against
platelet membrane glycoproteins IIb/IIIa or Ib/IX complexes can be detected in these
patients. these platelet-associated IgG humoral antibodies synthesized mainly in the
spleen

Q-24
a) Name hereditary coagulation disorders.
b) Tabulate differences between hemophilia A and von-Willi brand disease.

ANS:
a) HEREDITARY COAGULATION DISORDERS
1. Classic hemophilia (hemophilia a)
2. Christmas disease (hemophilia b)
3. Von willebrand’s disease

KMU Past Papers Solved SEQs


Hematopoietic and Lymphoid Systems 49

b) DIFFERENCE BETWEEN HEMOPHILIA A AND VON WILLI BRAND DISEASE

Features Hemophilia A Von willi brand disease


Chromosomal X-linked recessive disorder autosomal dominant disorder
abnormality
sex Male are affected more Both are affected equally
Factor circulating vWF is reduced or factor VIII activity
responsible abnormal
Clinical bleeding for hours or days after spontaneous bleeding from
picture the injury mucous membranes and
excessive bleeding from wounds.
Lab findings Prothrombin time is usually Prolonged bleeding time
normal. Defective platelet aggregation
Activated partial with ristoceti
thromboplastin time APTT is
prolonged
Treatment Factor VIII replacement therapy ??

KMU Past Papers Solved SEQs


Lungs 50

Q-1
a) Classify Pneumonias.
b) Give the pathogenesis and complications of Lobar Pneumonia.

ANS:
a) CLASSIFICATION
A. On the basis of the anatomic region of the lung parenchyma involved,
1. Lobar pneumonia
2. Bronchopneumonia (or Lobular pneumonia)
3. Interstitial pneumonia.
B. Based on the clinical settings in which infection occurred
1. Community-acquire typical pneumonia
2. Community-acquire atypical pneumonia
3. Health care-associated pneumonia (including hospital-acquired pneumonia)
4. Ventilator-associated pneumonia
C. Based on etiology and pathogenesis,
1. Bacterial pneumonia B.
2. Viral pneumonia C.
3. fungal Pneumonias
4. Non infective pneumonias
a. Aspiration (inhalation) pneumonia
b. Hypostatic pneumonia
c. Lipid pneumonia

b) PATHOGENESIS
1. Micro-aspiration of oropharyngeal contents having microorganism
2. Inhalation of aerosol drops having microorganism
3. Bloodstream infection
4. These bacteria cause acute inflammation in the alveoli
5. There is acute inflammatory response to bacterial infection that lasts for 1 to 2
days this is called congestion.
6. Within few days the stage of red hepatization or early consolidation ensues, in
which the alveolar spaces are packed with neutrophils red cells and fibrin
7. When the red cells lysed lung become dark gray called gray hepatization or late
consolidation.
8. Resolution follow in uncomplicated cases when the exudates are enzymatically
digested, or organized by fibroblast growth.

COMPLICATIONS
1. Pleural effusion
2. Empyema
3. Lung abscess
4. Bacteremic dissemination leads to meningitis arthritis and infective endocarditis

KMU Past Papers Solved SEQs


Lungs 51

Q-2
a) Define pneumonia and name five bacteria causing community acquired acute
pneumonia.
b) Write down salient features of various stages of lobar pneumonia.
c) Name complications of acute bacterial pneumonia.

ANS:
a) DEFINITION:
Pneumonia is defined as acute inflammation of the lung parenchyma distal to the
terminal bronchioles (consisting of the respiratory bronchiole, alveolar ducts, alveolar
sacs and alveoli).
Bacteria causing pneumonia:
1. Streptococcus pneumonia
2. Haemophilus influenzae
3. Moraxella catarrhalis
4. Staphylococcus aureus
5. Legionella pneumophila
6. Klebsiella pneumonia
b) STAGES OF LOBAR PNEUMONIA:
1. Stage of congestion: the initial phase represents the early acute inflammatory
response to bacterial infection that lasts for 1 to 2 days.
G/A The affected lobe is enlarged, heavy, dark red and congested. Cut surface
exudes blood-stained frothy fluid.
M/E
i) Dilatation and congestion of the capillaries in the alveolar walls.
ii) Pale eosinophilic edema fluid in the air spaces.
iii) A few red cells and neutrophils in the intra-alveolar fluid.
iv) Numerous bacteria demonstrated in the alveolar fluid by Gram’s staining.
2. Red Hepatization: Early Consolidation: This phase lasts for 2 to 4 days.
G/A The affected lobe is red, firm and consolidated. The cut surface of the involved
lobe is airless, red-pink, dry, granular and has liver-like consistency. The stage of
red hepatization is accompanied by serofibrinous pleurisy.
M/E
i) The edema fluid of the preceding stage is replaced by strands of fibrin.
ii) There is marked cellular exudate of neutrophils and extravasation of red
cells.
iii) Many neutrophils show ingested bacteria.
iv) The alveolar septa are less prominent than in the first stage due to cellular
exudation.
3. Grey Hepatization: Late Consolidation: This phase lasts for 4 to 8 days.
G/A The affected lobe is firm and heavy. The cut surface is dry, granular and grey
in appearance with liver-like consistency.
M/E

KMU Past Papers Solved SEQs


Lungs 52

i) The fibrin strands are dense and more numerous.


ii) The cellular exudate of neutrophils is reduced due to disintegration of many
inflammatory cells as evidenced by their pyknotic nuclei. The red cells are
also fewer. The macrophages begin to appear in the exudate.
iii) The cellular exudate is often separated from the septal walls by a thin clear
space.
iv) The organisms are less numerous and appear as degenerated forms.
4. Resolution This stage begins by 8th to 9th day.
G/A The previously solid fibrinous constituent is liquefied by enzymatic action,
eventually restoring the normal aeration in the affected lobe. The cut surface is
grey-red or dirty brown and frothy, yellow, creamy fluid can be expressed on
pressing.
M/E
i) Macrophages are the predominant cells in the alveolar spaces, while
neutrophils diminish in number. Many of the macrophages contain engulfed
neutrophils and debris.
ii) Granular and fragmented strands of fibrin in the alveolar spaces are seen due
to progressive enzymatic digestion.
iii) Alveolar capillaries are engorged.
iv) There is progressive removal of fluid content as well as cellular exudate from
the air spaces, partly by expectoration but mainly by lymphatics.
c) COMPLICATIONS
1. Pleural effusion
2. Empyema
3. Lung abscess
4. Bacteremic dissemination leads to meningitis arthritis and infective endocarditis

Q-3
a) Classify Pneumonia. 4
b) Write down the steps in sequence which can result in inflammatory consolidation. 3
c) Briefly discuss the pathogenesis of Tuberculosis. 3

ANS:
a) CLASSIFICATION:
GIVEN SOMEWHERE ELSE**

b) INFLAMMATORY CONSOLIDATION
1. There is acute inflammatory response to bacterial infection that lasts for 1 to 2
days this is called congestion.
2. Within few days the stage of red hepatization or early consolidation ensues, in
which the alveolar spaces are packed with neutrophils red cells and fibrin
3. When the red cells lysed lung become dark gray called gray hepatization or late
consolidation.
4. Resolution follow in uncomplicated cases when the exudates are enzymatically
digested, or organized by fibroblast growth.

KMU Past Papers Solved SEQs


Lungs 53

c) PATHOGENESIS OF TUBERCULOSIS
1. Once a virulent strain of mycobacteria gains entry into the macrophage
endosomes through macrophage mannose receptor and complement receptors.
2. the organisms inhibit normal microbicidal responses by preventing the fusion of
the lysosomes with the phagocytic vacuole.
3. The prevention of phagolysosome formation allows unchecked mycobacterial
proliferation.
4. Thus “primary tuberculosis” in the non-sensitized patient is characterized by
bacillary proliferation within the pulmonary alveolar macrophages and air spaces,
resulting bacteremia and seeding of multiple sites.
5. In some people with polymorphisms of the NRAMP1 (natural resistance–
associated macrophage protein 1) gene, the disease may progress from this point
without development of an effective immune response. NRAMP1 is a
transmembrane ion transport protein found in endosomes and lysosomes that is
believed to contribute to microbial killing.
6. Processed mycobacterial antigens reach the draining lymph nodes and are
presented to CD4 T cells by dendritic cells and macrophages.
7. Under the influence of macrophage-secreted IL-12, CD4+ T cells of the TH1 subset
are generated that are capable of secreting IFN-γ.
8. IFN-γ is crucial in activating macrophages.
9. Activated macrophages, in turn release
(1) TNF, which is responsible for recruitment of monocytes, which in turn
undergo activation and differentiation into the “epithelioid histiocytes”
(2) expression of the inducible nitric oxide synthase (iNOS) gene, which results
in elevated nitric oxide levels at the site of infection, with excellent
antibacterial activity;
(3) generation of reactive oxygen species, which can have antibacterial activity.
10. Defects in any of the steps of a TH1 response result in poorly formed granulomas,
absence of resistance, and disease progression. Persons with inherited mutations
in any component of the TH1 pathway are extremely susceptible to infections with
mycobacteria.

Q-4
a) Classify Pneumonia.
b) Give the etiology of Bronchopneumonia.
c) Enumerate the fungal infections of lung

ANS:
b) CLASSIFICATION
GIVEN SOMEWHERE ELSE**
c) ETIOLOGY
1. staphylococci,
2. streptococci,
3. pneumococci,
4. Klebsiella pneumoniae,
5. Haemophilus influenzae,

KMU Past Papers Solved SEQs


Lungs 54

6. gram-negative bacilli like Pseudomonas and coliform bacteria.


d) FUNGAL INFECTINOS
1. Pneumocystis pneumonia (P. jirovecii)
2. Aspergillosis and aspergilloma (Aspergillus fumigatus)
3. Candidiasis or moniliasis (Candida albicans)
4. Mucormycosis or phycomycosis (Mucor and Rhizopus)
5. Histoplasmosis (Histo plasma capsulatum)
6. Cryptococcosis (Cryptococcus neoformans)
7. Coccidioidomycosis (Coccidioides immitis)
8. Blastomycosis (Blastomyces dermatitidis)

Q-5
a) Classify tumors of lung.
b) Enumerate the etiology and pathogenesis of carcinoma lung.
c) Give morphology of Squamous cell carcinoma lung.

ANS:
a) CLASSIFICATION OF LUNG TUMORS
I. Epithelial tumors
A. Benign
1. Papilloma
2. Adenoma
3. Hamartoma (coin lesion)
B. Malignant Bronchogenic carcinoma
1. Squamous cell (epidermoid) carcinoma
2. Small cell carcinoma
i) Pure
ii) Combined (with any other non-small cell carcinoma lung)
3. Adenocarcinoma
i) Acinar predominant
ii) Papillary predominant
iii) Lepidic predominant (formerly bronchiolo-alveolar carcinoma)
iv) Solid predominant with mucin formation
v) Micropapillary predominant
4. Large cell carcinoma
5. Adeno-squamous carcinoma
6. Other carcinomas
i) Pulmonary neuroendocrine tumor (carcinoid tumor)
ii) Bronchial gland carcinomas
a. Adenoid cystic carcinoma
b. Mucoepidermoid carcinoma
II. Soft tissue Tumors
(Fibroma, fibro sarcoma; leiomyoma, leiomyo-sarcoma; lipoma, chondroma,
haemangioma, lymphangioma, granular cell myoblastoma)

KMU Past Papers Solved SEQs


Lungs 55

III. Pleural Tumors


1. Benign mesothelioma
2. Malignant mesothelioma
IV. Miscellaneous Tumors
1. Carcinosarcoma
2. Pulmonary blastoma
3. Malignant melanoma
4. Malignant lymphoma
b) ETIOLOGY AND PATHOGENESIS
1) SMOKING: small cell and squamous cell carcinomas while adenocarcinoma mostly
occur in nonsmokers
Smokers have sequential epithelial changes ie squamous metaplasia dysplasia 
carcinoma in situ.
Carcinogens are: polycyclic aromatic hydrocarbons, nitrosamines and phenol
derivatives (tumor promoter)
2) OTHER FACTORS
i) Radiation exposure: to radon or patients receiving thoracic radiation.
ii) Atmospheric pollution
iii) Occupational causes: asbestos, nickel, beryllium, arsenic, metallic iron iron-oxide
and uranium mines
iv) Dietary factors: vitamin A deficiency.
v) Chronic scarring: old tuberculosis, asbestosis, chronic interstitial fibrosis, old
infarcts and in scleroderma
3) Molecular pathogenesis
i) Activation of growth-promoting oncogenes:
 Mutation in K-RAS oncogene
 Mutation in EGFR (epidermal growth factor receptor) oncogene in cases of
adenocarcinoma lung in non-smokers.
 Other mutations include EML4-ALK fusion gene, c MET tyrosine gene
amplification, BRAF, PIK3CA, HER-2, and MYC family, and overexpression of
bcl-2
ii) Inactivation of tumor-suppressor genes
 Inactivation of p53, Rb1 and p16 gene.
iii) Nicotine:
 Derivatives of nicotine in smoke unmask and expresses nicotine acetylcholine
receptors which activate the signaling pathway in tumor, blocking the
apoptosis.
iv) Inherited predisposition
a) Patients of Li-Fraumeni syndrome who inherit p53
b) Clinical cases of retinoblastoma having mutation in Rb gene
c) Mutations of cytochrome P450 system have been identified in lung cancer
patients; P450 metabolizes chemical carcinogen in tobacco smoke
C) MORPHOLOGY:
Grossly:
1. Arise centrally in major bronchi and spread to local hilar nodes, large lesions may
undergo central necrosis, giving rise to cavitation.

KMU Past Papers Solved SEQs


Lungs 56

2. Lesion often precede by squamous metaplasia or dysplasia and then transforms to


carcinoma in situ
3. In later stages tumor mass begins to obstruct the lumen of a major bronchus, often
producing distal atelectasis and infection.
4. There is surrounding invasion.
MICROSCOPICALLY
1. well differentiated, showing keratin pearls and intercellular bridges, moderately
differentiated and poorly differentiated having poor residual squamous cell features.
2. Occasionally, a variant of squamous cell carcinoma, spindle cell carcinoma, having
biphasic pattern of growth due to the presence of a component of squamous cell
carcinoma and the other sarcoma like spindle cell component, is found.

Q-6
(a) Write the histologic classification of malignant epithelial lung tumors. 4
(b) Compare and contrast squamous cell and adenocarcinoma of lung in a tabular form. 6

ANS:
a) HISTOLOGIC CLASSIFICATION OF MALIGNANT EPITHELIAL LUNG TUMORS
GIVEN SOMEWHERE ELSE**
b) SQUAMOUS CELL AND ADENOCARCINOMA OF LUNG COMPARISON

Features Squamous cell carcinoma Adenocarcinoma


Occurrence most commonly in men, often most common bronchogenic
with history of tobacco smoking carcinoma in women and is slow-
growing not related to smoking
Gene TP53 mutations or activation of activating mutations of the EGFR,
mutation the KRAS oncogene EML4-ALK tyrosine kinase fusion
genes, and c-MET tyrosine kinase
gene amplifications
Site arise centrally in major bronchi more peripherally located
Precursor squamous metaplasia or atypical adenomatous hyperplasia
lesion dysplasia in the bronchial (AAH) adenocarcinoma in situ (or)
epithelium, which then bronchioloalveolar carcinoma
transforms to carcinoma in situ,
Histology Large lesions may undergo acinar (gland-forming) papillary,
central necrosis, giving rise to mucinous (formerly mucinous
cavitation. Obstruction of major bronchioloalveolar carcinoma) and
bronchus, producing distal solid types.
atelectasis and infection Thyroid transcription factor 1
M/E showing keratin pearls positive
and intercellular bridges
Markers Parathyroid hormone related Mucin
peptide
Metastasis Later Earlier

KMU Past Papers Solved SEQs


Lungs 57

Q-7
a) Classify malignant epithelial lung tumors.
b) Write the salient features of etiology and pathogenesis of carcinoma of lung.
ANS:
a) CLASSIFICATION
GIVEN SOMEWHERE ELSE**
b) ETIOLOGY AND PATHOGENESIS
1) SMOKING: small cell and squamous cell carcinomas while adenocarcinoma mostly
occur in nonsmokers
Smokers have sequential epithelial changes ie squamous metaplasia dysplasia 
carcinoma in situ.
Carcinogens are: polycyclic aromatic hydrocarbons, nitrosamines and phenol
derivatives (tumor promoter)
2) OTHER FACTORS
i) Radiation exposure: to radon or patients receiving thoracic radiation.
ii) Atmospheric pollution
iii) Occupational causes: asbestos, nickel, beryllium, arsenic, metallic iron iron-oxide
and uranium mines
iv) Dietary factors: vitamin A deficiency.
v) Chronic scarring: old tuberculosis, asbestosis, chronic interstitial fibrosis, old
infarcts and in scleroderma
3) Molecular pathogenesis
i) Activation of growth-promoting oncogenes:
 Mutation in K-RAS oncogene
 Mutation in EGFR (epidermal growth factor receptor) oncogene in cases of
adenocarcinoma lung in non-smokers.
 Other mutations include EML4-ALK fusion gene, c MET tyrosine gene
amplification, BRAF, PIK3CA, HER-2, and MYC family, and overexpression of
bcl-2
ii) Inactivation of tumor-suppressor genes
 Inactivation of p53, Rb1 and p16 gene.
iii) Nicotine:
 Derivatives of nicotine in smoke unmask and expresses nicotine acetylcholine
receptors which activate the signaling pathway in tumor, blocking the
apoptosis.
iv) Inherited predisposition
a) Patients of Li-Fraumeni syndrome who inherit p53
b) Clinical cases of retinoblastoma having mutation in Rb gene
c) Mutations of cytochrome P450 system have been identified in lung cancer
patients; P450 metabolizes chemical carcinogen in tobacco smoke

Q-8
a) Name the histological types of lung cancer?
b) Briefly describe the features of small cell carcinoma

KMU Past Papers Solved SEQs


Lungs 58

ANS:
a) HISTOLOGICAL TYPES OF LUNG CANCER
GIVEN SOMEWHERE ELSE**
b) FEATURES OF SMALL CELL CARCINOMA
1. Small cell carcinoma Small cell carcinomas are frequently hilar or central in
location and early involvement of the hilar and mediastinal nodes.
2. Tumor have strong relation-ship to cigarette smoking and are highly malignant
tumors.
3. Necrosis is invariably present and may be extensive.
4. They are most often associated with ectopic hormone production which may
result in paraneoplastic syndromes
5. presence of neurosecretory granules in majority of tumor cells.
6. By immunohistochemistry, these tumor cells are positive for neuroendocrine
markers:
 chromogranin,
 neuron-specific enolase (NSE) and
 synaptophysin.
7. Small cell carcinomas have 2 subtypes:
i) Pure small cell carcinoma is composed of uniform, small (or oat-like) cells, larger
than lymphocytes with dense, round or oval nuclei having diffuse chromatin,
inconspicuous nucleoli and very sparse cytoplasm
ii) Combined small cell carcinoma is a tumor in which there is a definite component
of small cell carcinoma with component of other non-small lung carcinoma such
as squamous cell and/or adenocarcinoma.
Q-9
a) Classify lung tumors. 3
b) Write pathogenesis of adenocarcinoma of lung. 3
c) Give important morphological features of squamous cell carcinoma of lung. 4

ANS:
a) CLASSIFICATION
GIVEN SOMEWHERE ELSE**
b) PATHOGENESIS
activating mutations of the epidermal growth factor receptor (EGFR). K-RAS mutations
(in 30% of adenocarcinomas)
Other mutations occurring in 4% to 6% of adenocarcinomas are EML4-ALK tyrosine
kinase fusion genes and c-MET tyrosine kinase gene amplifications.
c) MORPHOLOGICAL FEATURES
GROSSLY:
1. Arise centrally in major bronchi and spread to local hilar nodes, large lesions may
undergo central necrosis, giving rise to cavitation.
2. Lesion often precede by squamous metaplasia or dysplasia and then transforms to
carcinoma in situ
3. In later stages tumor mass begins to obstruct the lumen of a major bronchus, often
producing distal atelectasis and infection.

KMU Past Papers Solved SEQs


Lungs 59

4. There is surrounding invasion.


MICROSCOPICALLY
1. well differentiated, showing keratin pearls and intercellular bridges, moderately
differentiated and poorly differentiated having poor residual squamous cell features.
2. Occasionally, a variant of squamous cell carcinoma, spindle cell carcinoma, having
biphasic pattern of growth due to the presence of a component of squamous cell
carcinoma and the other sarcoma like spindle cell component, is found.

Q-10
a) Define and enumerate chronic obstructive lung diseases.
b) Give the pathogenesis of emphysema.
c) Types of Emphysema

ANS:
a) COPD
Chronic obstructive pulmonary disease (COPD) are commonly used clinical terms for a
group of pathological conditions in which there is chronic, partial or complete,
obstruction to the airflow at any level from trachea to the smallest airways resulting in
functional disability of the lungs
1. Chronic bronchitis
2. Emphysema
3. Bronchiectasis
4. Small airways disease (bronchiolitis)
b) PATHOGENESIS OF EMPHYSEMA
1. Exposure to toxic substances such as tobacco smoke and inhaled pollutants induces
inflammation
2. There is accumulation of neutrophils, macrophages and lymphocytes in the lung.
3. These cells produce Elastases, cytokines (including IL-8) and oxidants  epithelial
injury and proteolysis of the extracellular matrix (ECM).
4. Elastin degradation products further increase the inflammation.
5. Decreased antioxidant and α1-antitrypsin (antielastases) activity epically in patients
with congenital α1-antitrypsin deficiency
6. with certain TGFB gene polymorphisms, mesenchymal cell response to TGF-β
signaling is reduced, which in turn results in inadequate repair of elastin injury
7. Matrix metalloproteinase MMP-9 gene polymorphisms and higher levels of both
MMP-9 and MMP-12 have been found in some emphysema patients.
8. In emphysema there is loss of both epithelial or endothelial cells and mesenchymal
cells, leading to lack of extracellular matrix, the scaffolding upon which epithelial cells
would have grown. Thus, emphysema can be thought of as resulting from insufficient
wound repair
c) TYPES OF EMPHYSEMA
1. Centriacinar (Centrilobular) Emphysema
2. Panacinar (Panlobular) Emphysema
3. Distal Acinar (Paraseptal) Emphysema
4. Irregular Emphysema

KMU Past Papers Solved SEQs


Lungs 60

5. Mixed (unclassified) emphysema

Q-11
a) Define Asthma
b) Enumerate the types of Asthma
c) Briefly mention the pathogenesis of atopic asthma.

ANS:
a) ASTHMA
Asthma is a chronic inflammatory disorder of airways that is characterized by increased
responsiveness of the tracheobronchial tree to a variety of stimuli resulting in
widespread spasmodic narrowing of the air passages which causes recurrent episodes of
wheezing, breathlessness, chest tightness, and cough, particularly at night and/or early in
the morning
b) TYPES
1. Extrinsic or Atopic or allergic Asthma
2. Intrinsic or Non-Atopic or idiosyncratic Asthma
3. Drug-Induced Asthma
4. Occupational Asthma
c) PATHOGENESIS OF ATOPIC ASTHMA
It is type I hypersensitivity reaction which leads to bronchial hyper-responsiveness to a
variety of stimuli.
A. SENSITIZATION OR TRIGGERING:
1. Allergin presented to T Helper Cell 2 (TH2)
2. Activated TH2 cell produce cytokines (IL-13 IL-4 IL-5)
3. IL-13 stimulates mucus production and also promotes IgE production by B cells
4. IL-4 stimulates IgE production
5. IL-5 activates eosinophils
6. IgE coats submucosal mast cells
B. REXPOSURE OF SAME ALLERGEN:
1. Allergen will cross link IgE on mast cell and release preformed mediators ie
histamine serotonin
C. EARLY PHASE:
1. bronchoconstriction, triggered by direct stimulation of sub-epithelial vagal
receptors.
2. increased mucus production
3. variable vasodilation.
D. THE LATE-PHASE:
1. inflammation, with activation of eosinophils, neutrophils, and T cells.
2. epithelial cells are activated to produce chemokines (including eotaxin) that
promote recruitment of more TH2 cells and eosinophils
E. AIRWAY REMODELING.
1. These changes include hypertrophy of bronchial smooth muscle and mucus
glands,

KMU Past Papers Solved SEQs


Lungs 61

2. and increased vascularity and deposition of sub-epithelial collagen,

Q-12
a) Define asthma and give its types.
b) Give a brief description of etiology and pathogenesis of Asthma.
ANS:
a) ASTHMA
Asthma is a chronic inflammatory disorder of airways that is characterized by increased
responsiveness of the tracheobronchial tree to a variety of stimuli resulting in
widespread spasmodic narrowing of the air passages which causes recurrent episodes of
wheezing, breathlessness, chest tightness, and cough, particularly at night and/or early in
the morning

b) ETIOLOGY AND PATHOGENESIS


ETIOLOGY:
1. dusts, pollen, animal dander, and foods initiate Extrinsic or Atopic or allergic
Asthma
2. Respiratory infections due to viruses (e.g., rhinovirus, parainfluenza virus) and
inhaled air pollutants (e.g., sulfur dioxide, ozone, nitrogen dioxide) are etiology of
Intrinsic or Non-Atopic or idiosyncratic Asthma
3. Aspirin cause Drug-Induced Asthma
4. fumes (epoxy resins, plastics), organic and chemical dusts (wood, cotton,
platinum), gases (toluene), and other chemicals are etiologic factor of
Occupational Asthma

Q-13
a) Define chronic bronchitis,
b) Enlist the types of chronic bronchitis,
c) Write salient features of its pathogenesis?

Pathogenesis: given somewhere else**


ANS:
a) CHRONIC BRONCHITIS
it is defined by the presence of a persistent productive cough for at least 3 consecutive
months in at least 2 consecutive years.
b) TYPES:
1. Simple chronic bronchitis
2. Chronic mucopurulent bronchitis
3. Chronic asthmatic bronchitis
4. Chronic obstructive bronchitis
c) PATHOGENISIS
1. The distinctive feature of chronic bronchitis is hypersecretion of mucus, beginning
in the large airways.

KMU Past Papers Solved SEQs


Lungs 62

2. Most important cause is cigarette smoking, (The transcription of the mucin gene
MUC5AC in bronchial epithelium and the production of neutrophil elastase are
increased as a consequence of exposure to tobacco smoke)
3. other air pollutants, such as sulfur dioxide and nitrogen dioxide, may contribute.
4. These environmental irritants induce hypertrophy of mucous glands in the
trachea and main bronchi, leading to a marked increase in mucin-secreting goblet
cells in the surface epithelium of smaller bronchi and bronchioles.
5. In addition, these irritants cause inflammation with infiltration of CD8+
lymphocytes, macrophages, and neutrophils.
6. airflow obstruction in chronic bronchitis is more peripheral and results from
a. small airway disease (chronic bronchiolitis),
b. coexistent emphysema.
7. local release of T cell cytokines such as IL-13 mediated mucus hypersecretion.
.
Q-14
a) Define Emphysema
b) Enlist the types
c) Write the salient features of its pathogenesis

ANS:
a) EMPHYSEMA
Emphysema is characterized by abnormal permanent enlargement of the air spaces
distal to the terminal bronchioles, accompanied by destruction of their walls without
significant fibrosis.
b) TYPES
1. Centriacinar (Centrilobular) Emphysema
2. Panacinar (Panlobular) Emphysema
3. Distal Acinar (Paraseptal) Emphysema
4. Irregular Emphysema
5. Mixed (unclassified) emphysema
c) PATHOGENESIS
1. Exposure to toxic substances such as tobacco smoke and inhaled pollutants induces
inflammation
2. There is accumulation of neutrophils, macrophages and lymphocytes in the lung.
3. These cells produce Elastases, cytokines (including IL-8) and oxidants  epithelial
injury and proteolysis of the extracellular matrix (ECM).
4. Elastin degradation products further increase the inflammation.
5. Decreased antioxidant and α1-antitrypsin (antielastases) activity epically in patients
with congenital α1-antitrypsin deficiency
6. with certain TGFB gene polymorphisms, mesenchymal cell response to TGF-β
signaling is reduced, which in turn results in inadequate repair of elastin injury
7. Matrix metalloproteinase MMP-9 gene polymorphisms and higher levels of both
MMP-9 and MMP-12 have been found in some emphysema patients.
8. In emphysema there is loss of both epithelial or endothelial cells and mesenchymal
cells, leading to lack of extracellular matrix, the scaffolding upon which epithelial cells

KMU Past Papers Solved SEQs


Lungs 63

would have grown. Thus, emphysema can be thought of as resulting from insufficient
wound repair

Q- 14 A 45-year-old man has smoked two packs of cigarettes per day for 20 years for the past 4
years he had chronic cough with mucoid expectoration
a) What is most likely diagnosis
b) Write the main feature of pathogenesis
c) Give brief account of its morphology

ANS:
a) CHRONIC BRONCHITIS
b) PATHOGENESIS
Given somewhere else**
c) MORPHOLOGY
 Gross
1. mucosal lining of the larger airways usually is hyperemic and swollen by edema
fluid.
2. covered by a layer of mucinous or mucopurulent secretions.
3. The smaller bronchi and bronchioles also may be filled with similar secretions.
 histology
1. the diagnostic feature of chronic bronchitis in the trachea and larger bronchi is
enlargement of the mucus-secreting glands
2. increase in size is assessed by the ratio of the thickness of the submucosal gland
layer to that of the bronchial wall (the Reid index— normally 0.4).
3. Inflammatory cells, largely mononuclear but sometimes admixed with neutrophils,
are frequently present in variable density in the bronchial mucosa.
4. Chronic bronchiolitis (small airway disease), characterized by goblet cell
metaplasia, mucous plugging, inflammation, and fibrosis, is also present.
5. In the most severe cases, there may be complete obliteration of the lumen as a
consequence of fibrosis (bronchiolitis obliterans) leads to luminal narrowing and
airway obstruction.

KMU Past Papers Solved SEQs


Gastrointestinal Tract, Liver and Pancreas 64

Q-1
a) Define an Ulcer.
b) List the causes & location of Peptic ulcer.
c) Briefly discuss the mechanism of Peptic ulcer by H-Pylori.

ANS:
a) DEFINITION:
An ulcer is a local defect, or excavation, of the surface of an organ or tissue that is
produced by necrosis of cells and sloughing (shedding) of necrotic and inflammatory
tissue.
b) CAUSES
1. H. pylori infection
2. NSAID
3. Zollinger-Ellison syndrome,
4. Cigarette smoking,
5. High-dose
6. Alcoholic cirrhosis,
7. Severe burns or trauma
8. Intracranial disease
9. Chronic obstructive pulmonary disease,
10. Chronic renal failure,
11. Chronic pancreatitis
12. Hyperparathyroidism.
13. Psychologic stress
14. People with blood group O
Location:
1. First portion of the duodenum, (most common)
2. Gastric antrum
3. Esophagus as a result of GERD
4. Jejunum in Zollinger-Ellison syndrome
c) PATHOGENESIS OF PEPTIC ULCER BY H-PYLORI.
H. pylori may invade the gastric mucosa,
Four features are linked to H. pylori virulence:
1. Flagella, which allow the bacteria to be motile in viscous mucus
2. Urease, which generates ammonia from endogenous urea, thereby elevating local
gastric pH around the organisms and protecting the bacteria from the acidic pH of the
stomach
Also form ammonium chloride and monochloramine which are toxic to mucosa.
3. Adhesins, which enhance bacterial adherence to surface foveolar cells bacterial
protease and phospholipase break down glycoprotein complexes.
4. Toxins, such as that encoded by cytotoxin-associated gene A (CagA) and vasulating
toxin A, that may be involved in epithelia injury and ulcer.
Over time, chronic antral H. pylori gastritis may progress to pan-gastritis, resulting in
multifocal atrophic gastritis, reduced acid secretion, intestinal metaplasia, and increased
risk of gastric adenocarcinoma in a subset of patients.
The bacteria interactions between the host immune system and the bacterium seem to be
critical.

KMU Past Papers Solved SEQs


Gastrointestinal Tract, Liver and Pancreas 65

Q-2
a) Define inflammatory bowel disease.
b) Tabulate differences between Crohn's disease and Ulcerative Colitis.

ANS:
a) INFLAMMATORY BOWEL DISEASE
Inflammatory bowel disease (IBD) is a chronic condition resulting from inappropriate
mucosal immune activation, includes two major entities, Crohn disease and ulcerative
colitis, usually have distinctive morphological appearance, different sites of gut are
involved by these two entities.
b) DIFFERENCES BETWEEN CROHN’S DISEASE AND ULCERATIVE COLITIS

KMU Past Papers Solved SEQs


Gastrointestinal Tract, Liver and Pancreas 66

Q-3
a) Classify gastric tumors according to WHO classification
b) Enumerate the causes of gastric carcinoma
c) Morphology of gastric carcinoma

ANS:
a) CLASSIFICATION
1. Benign Tumors
a) Epithelial Adenomas (adenomatous or neoplastic polyps)
b) Non-epithelial Gastrointestinal spindle cell (stromal) Tumors (GIST)
2. Malignant Tumors
a) Epithelial (90%)
(i) Adenocarcinoma
a. Papillary
b. Mucinus
c. Tubular
d. Signet ring (diffuse)
(ii) Adeno-squamous
b) Non-epithelial (2%)
(i) Leiomyosarcoma
(ii) Leiomyoblastoma (epithelioid leiomyoma)
c) Carcinoid Tumor (3%)
d) Lymphoma (4%)
b) CAUSES
1. H. pylori infection.
2. Epstein-Barr virus infection.
3. Dietary factors, ingestion of smoked foods, nitrates in foods and drinking water,
alcohol
4. Geographical factors Japan, Chile and Italy have the highest incidence while the
incidence is low in the US, UK and Canada
5. Racial factors.
6. Genetic factors, Individuals with blood group A have higher tendency to develop
gastric cancer (peptic ulcer blood group O). A germ line mutation in E-cadherin gene
and mutation in APC tumor suppressor gene.
7. Pre-malignant changes in the gastric mucosa
i. Hypo- or achlorhydria
ii. Gastric mucosa with intestinal metaplasia due to GERD and Barrett esophagus.
iii. Adenomatous (neoplastic) polyps of the stomach.
iv. Chronic gastric ulcer (ulcer-cancer)
v. Stump carcinoma in patients who have undergone partial gastrectomy.
c) MORPHOLOGY
Gastric adenocarcinomas are classified according to their location in the stomach as well
as gross and histologic morphology. (Lauren classification)
1. Intestinal-type cancers:
Grossly:
i) Tend to be bulky and are composed of glandular structures similar to
esophageal and colonic adenocarcinoma.

KMU Past Papers Solved SEQs


Gastrointestinal Tract, Liver and Pancreas 67

ii) Typically grow along broad cohesive fronts to form either an exophytic mass
or an ulcerated tumor.
Microscopically:
i) The neoplastic cells often contain apical mucin vacuoles, and abundant mucin
may be present in gland lumina.
2. Diffuse gastric cancers:
Grossly:
i) Infiltrative growth pattern
ii) Often evoke a desmoplastic reaction that stiffens the gastric wall
iii) Causes diffuse rugal flattening and a rigid, thickened wall that imparts a
“leather bottle” appearance termed linitis plastica.
Microscopically:
i) These composed of discohesive cells with large mucin vacuoles that expand the
cytoplasm and push the nucleus to the periphery, creating a signet ring cell
morphology
ii) These cells permeate the mucosa and stomach wall individually or in small
clusters.

Q-4
a) Classify tumors of salivary glands on histological basis
b) Give brief account of morphology of pleomorphic adenoma

ANS:
a) CLASSIFICATION:
A. BENIGN
1. Adenomas:
a. Pleomorphic adenoma 50%
b. Monomorphic adenoma
i) Warthin’s tumor 5%
ii) Oncocytomas (Oxyphil adenoma) 2%
iii) Basal cell adenoma 2%
iv) Cystadenoma 2%
2. Mesenchymal tumors (rare)
B. MALIGNANT
1. Mucoepidermoid carcinoma 15%
2. Acinic cell carcinoma 6%
3. Adenocarcinoma 6%
4. Adenoid cystic carcinoma 4%
5. Malignant mixed tumor 3%
a. Carcinoma in pleomorphic adenoma
b. Carcinosarcoma
c. Metastasizing mixed salivary tumor
b) MORPHOLOGIC FEATURES
A. Grossly:
1. Circumscribed, pseudo-encapsulated, rounded multi-lobulated, firm mass
2. The cut surface is grey-white and bluish translucent chondroid, occasionally may
show small cystic spaces.
B. Microscopically,

KMU Past Papers Solved SEQs


Gastrointestinal Tract, Liver and Pancreas 68

characterized by pleomorphic or ‘mixed’ appearance having epithelial elements and


stromal element
a. Epithelial component may form ducts, acini, tubules, sheets and strands of cells of
ductal or myoepithelial origin.
1. The ductal cells are cuboidal or columnar, while myoepithelial cells may be
polygonal or spindle-shaped resembling smooth muscle cells.
2. No epithelial dysplasia or mitotic activity is evident
3. PAS-positive epithelial mucin is found in the lumina of duct-like structures is.
4. Focal areas of squamous metaplasia and keratinization may be present.
5. Immuno-histochemically the tumor cells are immunoreactive for epithelial
(cytokeratin, EMA, CEA) as well as myoepithelial (actin, vimentin and S-100)
antibodies.
b. Stromal elements
1. Myxoid, mucoid and chondroid matrix, which simulates cartilage
(pseudocartilage).
2. True cartilage and even bone may also be observed
3. Based on immunohistochemistry stromal elements are modified myoepithelial
cells as seen by S-100 immunostain positivity

Q-5
a) Enumerate the precancerous lesions of the oral cavity.
b) List the risk factors for squamous cell carcinoma of oral cavity.
c) Give a brief account of gross morphology of squamous cell carcinoma of oral cavity.

ANS:
a) PRECANCEROUS LESION OF ORAL CAVITY:
1. Leukoplakia
 Hyperkeratotic leukoplakia
 Dysplastic leukoplakia
2. Lichen planus
3. Erythroplakia
4. Melanosis and mucosal hyperpigmentation.
b) ORAL CAVITY SQUAMOUS CELL CARCINOMA RISK FACTORS:
a. Oral cavity tumors
1. Smoking
2. Tobacco chewing
3. Alcohol
4. Dietary deficiency vitamin B and iron
5. Dental sepsis, ill-fitting dentures, sharp teeth.
6. Poor oral hygiene.
7. Exposure to sunlight
8. Exposure to radiation.
b. Tonsillar crypts, base of the tongue or oropharynx tumors.
1. human papillomavirus (HPV), particularly HPV-16.
c) MORPHOLOGY:
a. Gross:

KMU Past Papers Solved SEQs


Gastrointestinal Tract, Liver and Pancreas 69

1. Most common locations are the ventral surface of the tongue, floor of the mouth,
lower lip, soft palate, and gingiva
2. Squamous cell carcinoma of oral cavity may be
i) Ulcerative type— have irregular and indurated or rolled borders
ii) Papillary or verrucous type—is soft and wart-like growth.
iii) Nodular type—appears as a firm, slow growing submucosal nodule.
iv) Scirrhous type—is characterized by infiltration into deeper structures.
b. Histology:
1. Squamous cell carcinoma ranges from well-differentiated keratinizing carcinoma
to highly-undifferentiated neoplasm.
2. The degree of histologic differentiation, as determined by the relative degree of
keratinization,
3. Numerous nests and islands of malignant keratinocytes invading the underlying
connective tissue stroma.
Q-6An elderly obese female of 68 years presented with left lower quadrant discomfort, fatigue,
weakness occult bleeding (melena), iron deficiency anemia, and disturbed bowel habits.
a) What is the most likely diagnosis?
b) How will you classify the tumors of this region?
c) Give a brief account on the morphological appearance of this tumor,

ANS:
a) LEFT-SIDED COLORECTAL ADENOCARCINOMAS
b) CLASSIFICATION OF COLON TUMORS:
1. Polyps:
i. Non-neoplastic polyps
a. Hyperplastic (metaplastic) polyps
b. Hamartomatous polyps
(i) Peutz-Jeghers polyps
(ii) Juvenile polyps
c. Inflammatory polyps (Pseudopolyps)
d. Lymphoid polyps
ii. Neoplastic polyps (Adenomas)
a. Tubular adenoma
b. Villous adenoma
c. Tubulovillous adenoma
iii. Familial polyposis syndromes
a. Familial adenomatosis polyposis
b. Hereditary Nonpolyposis Colorectal Cancer (lynch syndrome)
2. Benign tumors:
i. Leiomyomas,
ii. leiomyoblastoma,
iii. neurilemmoma,
iv. lipoma and vascular tumors
3. Malignant tumors:
i. Carcinoma
a. Adenocarcinoma
 Left side
 Right side

KMU Past Papers Solved SEQs


Gastrointestinal Tract, Liver and Pancreas 70

b. Mucinous adenocarcinoma,
c. signet-ring cell carcinoma,
d. adenosquamous carcinoma
ii. Other malignant tumors
a. Leiomyosarcoma,
b. malignant lymphoma,
c. carcinoid tumors
c) MORPHOLOGY:
c. Distribution:
1. Rectum, 60%
2. Sigmoid and descending colon (25%),
3. Caecum and ileocaecal valve (10%);
4. Ascending colon, hepatic and splenic flexures (5%);
5. Uncommon transverse colon.
d. Gross:
1. Right-sided colonic growths:
tend to be large, cauli flower-like, soft and friable masses projecting in the form of
exophytic or polypoid masses.
2. Left-sided colonic growths,
have napkin ring configuration i.e. they encircle the bowel wall circumferentially
with increased fibrous tissue forming annular ring,
constrictions and luminal narrowing are present
have central ulceration on the surface with slightly elevated margins
e. Histology:
1. The appearance of right and left-sided growths is similar.
2. Most tumors are composed of tall columnar cells that resemble dysplastic
epithelium found in adenomas.
3. The invasive component of these tumors elicits a strong stromal desmoplastic
response, which is responsible for their characteristic firm consistency.
4. Some poorly differentiated tumors form few glands
5. Others may produce abundant mucin that accumulates within the intestinal wall
6. Tumors also may be composed of signet ring cells that are similar to those in
gastric cancer

Q-7
a) Define celiac disease.
b) What is the pathogenesis of this disorder?
c) Enumerate the histological characteristics of this disease.

ANS:
a) CELIAC DISEASE:
Celiac disease, also known as celiac sprue or gluten-sensitive enteropathy, is an immune-
mediated enteropathy triggered by the ingestion of gluten-containing cereals, such as
wheat, rye, or barley, in genetically predisposed persons
b) PATHOGENESIS
1. Celiac disease is an intestinal immune reaction to gluten, the major storage protein of
wheat and similar grains.

KMU Past Papers Solved SEQs


Gastrointestinal Tract, Liver and Pancreas 71

2. Gluten is digested by luminal and brush border enzymes into amino acids and
peptides, including a 33–amino acid gliadin peptide
3. Gliadin is de-amidated by tissue transglutaminase
4. These de-amidated gliadin is then interact with HLA-DQ2 or HLA-DQ8 on antigen-
presenting cells.
5. These APC present gliadin to CD4+ T cells.
6. These T cells produce cytokines that are likely to contribute to the tissue damage and
characteristic mucosal histopathology like loss of villous surface
7. B cell are also activated and produce
1) anti-tissue transglutaminase,
2) anti-deamidated gliadin,
3) and, anti-endomysial antibodies,
8. There is accumulation of CD8+ cells and may play role in causing tissue damage.
9. deamidated gliadin peptides induce epithelial cells to produce the cytokine IL-15,
10. these IL-5 activation and proliferation of CD8+ intraepithelial lymphocytes and
express NKG2D.
11. These lymphocytes become cytotoxic and kill enterocytes that having surface MIC-A,
that is recognized by NKG2D.
12. immune mechanisms may increase the movement of gliadin peptides across the
epithelium, which are deamidated by tissue transglutaminase, thus perpetuating the
cycle of disease.
c) HISTOLOGICAL CHARACTERISTICS:
The histopathologic picture is characterized by
1. Increased numbers of intraepithelial CD8+ T lymphocytes,
2. Intraepithelial lymphocytosis,
3. Crypt hyperplasia, and villous atrophy
4. This loss of mucosal and brush border surface area.
5. Increased crypt mitotic activity because of increased rates of epithelial turnover
6. Increased numbers of plasma cells, mast cells, and eosinophils, especially within the
upper part of the lamina propria.

Q-8
a) Define Barrett’s esophagus. How it is diagnosed?
b) Enumerate risk factors for squamous cell carcinoma of esophagus.
c) What are the complications of Barrett’s esophagus?

ANS:
a) BARRET’S ESOPHAGUS:
Barrett esophagus is a complication of chronic GERD that is characterized by intestinal
metaplasia within the esophageal squamous mucosa, in which stratified squamous
epithelium of the lower esophagus is replaced by columnar epithelium having goblet
cells.
Diagnosis:
1. Esophageal endoscopy shows velvety mucosa
2. Biopsy
b) RISK FACTORS OF SQUAMOUS CELL CARCINOMA OF ESOPHAGUS:
1. Alcohol
2. Tobacco
3. Poverty,
4. Caustic esophageal injury,

KMU Past Papers Solved SEQs


Gastrointestinal Tract, Liver and Pancreas 72

5. Achalasia,
6. Plummer-Vinson syndrome,
7. Frequent consumption of very hot beverages,
8. Previous radiation therapy to the mediastinum.
9. Nutritional deficiencies such as iron
10. Polycyclic hydrocarbons,
11. Nitrosamines,
12. Fungus-contaminated foods
13. High risk HPV infection.
c) COMPLICATION OF BARRET’S ESOPHAGUS:
1. Ulceration of esophagus
2. Stricture formation
3. Distal adenocarcinoma

Q-9
a) List the most common malignant tumors of stomach.
b) Name the diseases which predispose to gastric carcinoma
c) What are the macroscopic features of gastric carcinoma?
ANS:
a) COMMON MALIGNANT TUMORS OF STOMACH
1. Gastric adenocarcinoma
2. Leiomyosarcoma
3. Malignant lymphoma (MALTOMA)
4. Carcinoid Tumor
b) DISEASE WHICH PREDISPOSE TO GASTRIC CARCINOMA:
1. Chronic gastritis, most commonly due to H. Pylori infection
2. Atrophic gastritis with Hypochlorhydria or achlorhydria and intestinal metaplasia of
gastric mucosa.
3. Chronic gastric ulcer (ulcer-cancer), and its association with achlorhydria.
4. Gastric adenomas
5. Stump carcinoma in patients who have undergone partial gastrectomy.
6. Menetrier disease
7. Barrett’s esophagus
c) MACROSCOPIC FEATURES OF GASTRIC CARCINOMA:
Site:
Most common locations lesser curvature, pylorus and antrum.
Other less common locations are the body, cardia and fundus
Appearance:
1. Ulcerative carcinoma  The Tumor appears as a flat, infiltrating and ulcerative
growth with irregular necrotic base and raised margin.
2. Fungating (Polypoid) carcinoma  cauliflower growth projecting into the lumen.
3. Scirrhous carcinoma (Linitis plastica)  stomach is thickened due to extensive
desmoplasia giving the appearance as ‘leather-bottle stomach’ or ‘linitis plastic
4. Colloid (Mucoid) carcinoma
5. Ulcer-cancer
The Lauren classification that separates gastric cancers into intestinal and diffuse types
1. Intestinal: exophytic mass or an ulcerated tumor

KMU Past Papers Solved SEQs


Gastrointestinal Tract, Liver and Pancreas 73

2. Diffuse: gastric wall and may cause diffuse rugal flattening and a rigid, thickened wall
that imparts a “leather bottle” appearance termed linitis plastica

Q-10 A 65 years old man visits a dentist for white, irregular raised patch on the lateral border of
the tongue for more than 8 months and slowly increasing in size. The patient is a chronic
smoker on examination, the white lesion cannot be scrapped.
a) Name three white lesions of oral cavity,
b) How will the dentist establish final diagnosis?
c) Differentiate between leukoplakia and erythroplakia,

ANS:
a) WHITE LESIONS OF ORAL CAVITY:
1. Leukoplakia
2. Lichen planus
3. Candidiasis
4. Lupus erythematosus
5. White spongy nevus
b) DIAGNOSIS:
Always Biopsy these lesion because of the high risk for progression to oral squamous cell
carcinoma.
c) DIFFERENCES BETWEEN LEUKOPLAKIA AND ERYTHROPLAKIA

Features Leukoplakia Erythroplakia


Color and shape white patch or plaque Red patch or plaque
Grossly Homogenous Homogenous
Nodular Spekled
Erosive Erosive
Dysplasia Less more
Keratinization More Less
Malignant 1 to 17 % 17 time more than
potential leukoplakia

Q-11 A 60-year-old male present with abdominal discomfort nausea easy fatigability loss 10 kg
weight over a period of 6 month. On endoscopic examination an ulcerated exophytic mass in
stomach antrum along the lesser curvature is seen.
a) What is your probable diagnosis?
b) How will you establish final diagnosis? Enlist its histological types.
c) Enumerate the major factor that are associated with increased incidences of the diagnosed
disease.

ANS:
a) STOMACH ADENOCARCINOMA (INTESTINAL TYPE)
b) DIAGNOSIS:
1. Gastric endoscopy
2. Biopsy
3. CT-scan
Histological types
1. Intestinal: exophytic mass or an ulcerated tumor

KMU Past Papers Solved SEQs


Gastrointestinal Tract, Liver and Pancreas 74

2. Diffuse: gastric wall and may cause diffuse rugal flattening and a rigid, thickened wall
that imparts a “leather bottle” appearance termed linitis plastica
c) RISK FACTORS
A) Environmental
1. H. pylori infection.
2. Epstein-Barr virus infection.
3. ingestion of smoked foods,
4. nitrates in foods and drinking water,
5. alcohol
B) Genetic factors:
1. Individuals with blood group A
2. A germ line mutation in E-cadherin gene and
3. mutation in adenomatous polyposis coli (APC) tumor suppressor gene.
C) Predisposing condition
1. Chronic gastritis, most commonly due to H. pylori infection
2. atrophic gastritis with Hypochlorhydria or achlorhydria and intestinal metaplasia
of gastric mucosa.
3. Chronic gastric ulcer (ulcer-cancer), and its association with achlorhydria.
4. Gastric adenomas
5. Stump carcinoma in patients who have undergone partial gastrectomy.
6. Menetrier disease
7. Barrett’s esophagus
8. GERD
9. Obesity

Q-12 A 70-year male presented with anemia weight loss and constipation. Endoscopic
examination reveals mass in descending colon. Biopsy from the mass reveals adenocarcinoma
of the colon.
a) Write salient gross features of right and left side colon cancers
b) Enumerate adenoma-carcinoma sequence of colon cancer (APC/B-catenin pathway)

ANS:
a) GROSS FEATURES OF:
RIGHT SIDE: Tumors in the right side (proximal colon) often grow as polypoid, exophytic
masses that extend along one wall of the large-caliber cecum and ascending colon;
these tumors rarely cause obstruction
LIFT SIDE: Carcinomas in the (left side) distal colon tend to be annular lesions that
produce “napkin ring” constrictions and luminal narrowing to the point of obstruction.
Both forms grow into the bowel wall over time and may be palpable as firm masses.
b) THE APC/Β-CATENIN PATHWAY.
1. Germline or somatic mutations of APC gene on chromosome 5: (Hyper-proliferative
epithelium on histology)
1st copy of APC gene gets mutated.
2. Loss of DNA Methylation: (Early adenoma)
2nd copies of the APC gene are functionally inactivated.
APC is a key negative regulator of β-catenin, a component of the WNT signaling
pathway
The APC protein normally binds to and promotes degradation of β-catenin. With loss
of APC function, β-catenin accumulates and translocates to the nucleus, where it

KMU Past Papers Solved SEQs


Gastrointestinal Tract, Liver and Pancreas 75

activates the transcription of genes, such as those encoding MYC and cyclin D1, which
promote proliferation.
3. Activating Point mutations in Proto-oncogene in KRAS: (intermediate Adenoma)
Which promote growth and prevent apoptosis.
4. mutations in other tumor suppressor genes such as SMAD2 and SMAD4: (late
adenoma)
which are effectors of TGF-β signaling and loss of these genes may allow unrestrained
cell growth.
5. Loss tumor suppressor gene TP53 due to chromosomal deletions: (Carcinoma)
tumor suppressor genes may be silenced by methylation
6. Expression of telomerase also increases as lesions become more advanced. (Further
advanced Carcinoma)
PEARLS:
The microsatellite instability pathway
1. Germline or somatic mutations of mismatch repair genes MLH1, MSH2 MSH6, PMS1,
PMS2
2. Alteration of second allele of these genes mutation or methylation
3. accumulate in microsatellite repeats in noncoding regions.
4. when microsatellite sequences are located in the coding or promoter regions of genes
it causes subsequent mutation in these genes which are as type II TGF-β receptor and
the pro-apoptotic protein BAX, oncogene BRAF, TCF-4 and IGF2R,

Q-13
a) Name the ulcerative lesions of gastrointestinal tract
b) Give a brief account of how the Peptic ulcer is produced.

ANS:
a) ULCERATIVE LESIONS IN GIT
1. Ulcers in the Oral Cavity:
i) Aphthous Stomatitis:
ii) Trauma, Cancer, and Infections like Syphilis and Tuberculosis, pancytopenia
iii) Behçet’s disease
iv) Herpes virus infections
v) Vitamin B and C Deficiency
vi) Xerostomia sjogran syndrome
2. Ulcers of the Esophagus
i) Esophagitis
ii) Esophageal varices
iii) Mallory-Weiss Tear:
iv) Esophageal carcinoma
3. Stomach and Duodenum
i) Peptic Ulcers
ii) Gastritis
iii) Stomach Cancer
4. Intestinal and Anal Ulcers:
i) Ulcerative colitis
ii) Hiv,
iii) Tuberculosis,

KMU Past Papers Solved SEQs


Gastrointestinal Tract, Liver and Pancreas 76

iv) Cytomegalovirus
v) Typhoid
vi) Cancer
vii) Anal fissure
viii) Ischemic bowl disease
ix) Radiation
5. Ulcers at Multiple Sites in the Digestive Tract:
i) Crohn’s Disease
b) PATHOGENESIS OF PEPTIC ULCER:
1. NSAID-induced ulcers are caused by direct chemical irritation as well as
cyclooxygenase inhibition, which prevents prostaglandin synthesis. This eliminates
the protective effects of prostaglandins, which include enhanced bicarbonate
secretion and increased vascular perfusion.
2. Cushing ulcers in Lesions associated with intracranial injury are thought to be caused
by direct stimulation of vagal nuclei, which causes gastric acid hypersecretion.
3. Curling ulcers, occurring in the proximal duodenum and associated with severe burns
or trauma
4. Systemic acidosis, a frequent finding in critically ill patients, also may contribute to
mucosal injury by lowering the intracellular pH of mucosal cells.
5. Stress induced ulcer due to Hypoxia and reduced blood flow caused by stress-induced
splanchnic vasoconstriction also contribute to acute ulcer.
6. Peptic ulcer in H. pylori infection through these virulence factors
1) Flagella, which allow the bacteria to be motile in viscous mucus
2) Urease, which generates ammonia from endogenous urea, thereby elevating
local gastric pH around the organisms and protecting the bacteria from the
acidic pH of the stomach
3) Also form ammonium chloride and monochloramine which are toxic to
mucosa.
4) Adhesins, which enhance bacterial adherence to surface foveolar cells bacterial
protease and phospholipase break down glycoprotein complexes.
5) Toxins, such as that encoded by cytotoxin-associated gene A (CagA) and
vasulating toxin A, that may cancer development by poorly defined mechanism
7. Zollinger-Ellison syndrome in which there is uncontrolled release of gastrin by a
tumor and the resulting massive acid production, characterized by multiple peptic
ulcerations in the stomach, duodenum, and even jejunum

Q-14 A young emotionally stressed female presents to medical OPD with complaints of
intermittent Attacks of mild diarrhea, abnormal pain and fever followed by asymptomatic
period lasting for weeks and months. Positive “string sign” is seen. Colonoscopy reveals
patchy, deep intestinal ulcers.
a) What is the most likely diagnosis? 1
b) How would you differentiate the diagnosed disease from other form of inflammatory
Bowel Disease (IBD)? 6
c) What are the main factors that contribute to the above diagnose disease? 3

ANS:
a) CROHN DISEASE
b) CROHN DISEASE AND ULCERATIVE COLITIS DIFFERENCES
**Given above some where

KMU Past Papers Solved SEQs


Gastrointestinal Tract, Liver and Pancreas 77

c) RISK FACTORS:
1. Genetic: monozygotic twin has 50% concordance rate
2. Intestinal microbiota
3. Intestinal epithelial defects
4. Mucosal immune response
5. Smoking
6. Physical or emotional stress
7. Specific dietary items

Q-15
a) Simply enumerate the ulcerative lesions of the oral cavity
b) How aphthous ulcer differ from the herpetic ulcer with reference to site of occurrence

ANS:
a) ULCERATIVE LESION OF ORAL CAVITY:
1. Aphthous ulcer
2. Behçet’s disease
3. Herpes virus infections
4. Vitamin B and C Deficiency
5. Trauma, Cancer, and Infections like Syphilis and Tuberculosis

b) APTHOUS ULCER AND HERPETIC ULCER DIFFERENCES

Aphthos ulcer Herpetic ulcer


1 Immune disorder exact etiology HSV 1 or HSV 2 infection
unknown
2 Frequently occur Rarely occur
3 Involve movable mucosa ie soft Can present anywhere ie lips nasal
palate and cheek mucosa orifices buccal mucosa gingiva hard
palate soft palate
4 Associated with IBC bahcet disease No such association
5 No recurrent infections Recurrent infection because virus is
latent in trigeminal ganglion
6 Constitutional symptoms are absent Fever malaise is present
ie fever malaise
7 Cervical lymphadenopathy is absent Cervical lymphadenopathy is present
8 No vesicles Vesicles are present
9 Non-keratinized mucosa Keratinized mucosa

Q-16 A 7-month baby presented with weight loss and diarrhea. On examination the child was
irritable and Abdomen was distended. The child was recently started on wheat diet.
a) What is your diagnosis? 2
b) Write the pathogenesis of the disease. 5
c) What lab tests will confirm the diagnosis? 3

ANS:
a) CELIAC DISEASE
b) PATHOGENESIS:

KMU Past Papers Solved SEQs


Gastrointestinal Tract, Liver and Pancreas 78

1. Celiac disease is an intestinal immune reaction to gluten, the major storage protein of
wheat and similar grains.
2. Gluten is digested by luminal and brush border enzymes into amino acids and
peptides, including a 33–amino acid gliadin peptide
3. Gliadin is de-amidated by tissue transglutaminase
4. These de-amidated gliadin is then interact with HLA-DQ2 or HLA-DQ8 on antigen-
presenting cells.
5. These APC present gliadin to CD4+ T cells.
6. These T cells produce cytokines that are likely to contribute to the tissue damage and
characteristic mucosal histopathology like loss of villous surface
7. B cell are also activated and produce
 anti-tissue transglutaminase,
 anti-deamidated gliadin,
 and, anti-endomysial antibodies,
8. There is accumulation of CD8+ cells and may play role in causing tissue damage.
9. deamidated gliadin peptides induce epithelial cells to produce the cytokine IL-15,
10. these IL-5 activation and proliferation of CD8+ intraepithelial lymphocytes and
express NKG2D.
11. These lymphocytes become cytotoxic and kill enterocytes that having surface MIC-A,
that is recognized by NKG2D.
12. immune mechanisms may increase the movement of gliadin peptides across the
epithelium, which are deamidated by tissue transglutaminase, thus perpetuating the
cycle of disease.
c) LAB DIAGNOSIS:
1. Serum specific antibodies
i. Anti-tissue transglutaminase IgA and IgG
ii. Anti endomysial IgA antibody
iii. Anti-gliadin IgA and IgG antibodies
2. Serum total IgA level
3. Endoscopic biopsy
4. Karyotyping for HLA DQ2 and HLA DQ8. Its absence will exclude the disease (high
negative predictive value)

Q-17 A 71-year-old male presents with dysphagia and is found to have a 5-cm mass that is
located in the middle third of the esophagus and extends into adjacent lung tissue. Mediastinal
lymph nodes are also enlarged.
a) What is the most likely diagnosis? 1
b) What is the TNM stage of disease? 3
c) Give the morphological features of the diagnosed lesion. 6

ANS:
a) SQUAMOUS CELL CARCINOMA OF ESOPHAGUS:
b) TNM stage 3
c) MORPHOLOGICAL FEATURES
a. Locations:
1. Upper third (~15%)
2. Middle third (~50%)
3. Lower third (~35%)
b. Gross:

KMU Past Papers Solved SEQs


Gastrointestinal Tract, Liver and Pancreas 79

1. Polypoid fungating type, cauliflower-like friable mass protruding into the lumen.
2. Ulcerating type, a necrotic ulcer with everted edges
3. Diffuse infiltrating type, annular, stenosing narrowing of the lumen due to
infiltration into the wall of esophagus.
4. These cancers may invade surrounding structures including the respiratory tree,
causing pneumonia; the aorta, causing catastrophic exsanguination; or the
mediastinum and pericardium
c. Microscopic:
1. Squamous cell carcinoma begins as an in situ lesion in the form of squamous
dysplasia.
2. majority of the squamous cell carcinomas of the esophagus are well-differentiated
or moderately differentiated
3. Prickle cells, keratin formation and epithelial pearls are commonly seen.
4. However, non-keratinizing and anaplastic growth patterns can also occur.
5. histologic variants include verrucous squamous cell carcinoma (An exophytic,
slow-growing, extremely well-differentiated variant), spindle cell carcinoma, and
basaloid squamous cell carcinoma

Q-18
a) Enumerate gall bladder stones
b) Give its etiology and pathogenesis
c) Enumerate complications of gall stones

ANS:
a) GALL BLADDER STONES:
1. Cholesterol stones, (containing crystalline cholesterol monohydrate 80% of stones,
radiopaque due to calcium carbonate otherwise RADIOLUCENT)
2. Pigment stones, (made of bilirubin calcium salts)
i. Black stones (small in size, fragile to the touch, and numerous, found in sterile
gallbladder bile, radiopaque)
ii. Brown stones (single or few in number and to have a soft, greasy, found in
infected intra or extrahepatic ducts, radiolucent)

b) ETIOLOGY AND PATHOGENESIS:


ETHOLOGY:
1. Increased cholesterol synthesis and hepatic uptake  increase biliary cholesterol
secretion.
2. Reduced bile acid pool
3. Increased deoxycholic acid
4. Mutation in MDR3 gene  decrease lecithin
5. Decreased cholecystokinin receptors in gall bladder  decreased motility
6. Estrogenic influences (pregnancy, oral contraceptives)  increase cholesterol uptake
and synthesis
7. Hypocholesterolemic agents  increase biliary cholesterol secretion
8. Chronic hemolytic anemias  Increased level of unconjugated bilirubin
9. Alcoholic cirrhosis
10. Chronic biliary infections such as clonorchis sinensis and ascaris lumbricoides
Pathogenesis:

KMU Past Papers Solved SEQs


Gastrointestinal Tract, Liver and Pancreas 80

1. Supersaturation of bile with cholesterol due to


i) Increase cholesterol synthesis and production
ii) Reduced bile acid pool
iii) Increased conversion of cholic acid to deoxycholic acid
iv) Decreased lecithin due to MDR3 gene mutation
2. Nucleation of cholesterol monohydrate in mucin gel layer of gall bladder, due to
i) Increased pro nucleating factors (mucin and non-mucin glycoproteins)
ii) Decreased anti nucleating factors (apolipoproteins AI and AII)
3. Mucin hypersecretion and consequent entrapment of cholesterol crystal.
4. Gall bladder hypomobility resulting in stasis of biliary sludge and lithogenesis.
5. Pigmented gall stones formation is due to
Chronic hemolysis resulting in increased level of unconjugated bilirubin in bile
i) Alcoholic cirrhosis
ii) Chronic biliary infections by parasites such as clonorchis sinensis and ascaris
lumbricoide
c) COMPLICATIONS OF GALL BLADDER STONES:
1. Cholecystitis
2. Choledocholithiasis: Patients with gallstone in the common bile duct develop
obstructive jaundice. Fever may develop due to bacterial ascending cholangitis.
3. Mucocele: distension of the gallbladder by clear, watery mucinous secretion resulting
from impacted stones in the neck of the gallbladder.
4. empyema: When it gets infected and puss is accumulated.
5. Biliary fistula: between one part of the biliary system and the bowel, and rarely
between the gallbladder and the skin.
6. Perforation
7. Gallstone ileus: gallstones in the intestine may cause intestinal obstruction.
8. Pancreatitis: Obstructive cholecystasis may result in acute pancreatitis.
9. Gallbladder cancer

Q-19
a) Define jaundice.
b) Classify Jaundice in infancy and childhood.
c) Enumerate the complications of Hepatitis C (HCV).

ANS:
a) JAUNDICE:
Jaundice is yellow discoloration of skin and sclera (icterus), occurs when systemic
retention of bilirubin produces serum levels above 2.0 mg/dL (hyperbilirubinemia),
normal level in adults is below 1.2 mg/dL.
b) NEONATAL JAUNDICE CLASSIFICATION
A. Unconjugated hyperbilirubinemia
1) Non-hemolytic
Gilbert’s syndrome
Crigler-Najjar syndrome (type I and II)
2) Hemolytic
Physiologic and prematurity jaundice
Congenital hemolytic disorders
B. Conjugated hyperbilirubinemia
1) Dubin-Johnson syndrome,

KMU Past Papers Solved SEQs


Gastrointestinal Tract, Liver and Pancreas 81

2) Rotor’s syndrome
3) Benign recurrent intrahepatic cholestasis
4) Progressive familial intrahepatic cholestasis
5) Neonatal hepatitis
6) Biliary atresia
7) Reye’s syndrome
c) COMPLICATION OF HEPATITIS C
1. Chronic hepatitis
2. Cirrhosis
3. Liver failure
4. Hepatic encephalopathy
5. Hematemesis from esophageal varices
6. Hepatocellular carcinoma

Q-20
a) Tabulate differences between Hepatitis A, B and C viral diseases.
b) Describe liver function tests (LFTs).

ANS:
a) DIFFERENCES BETWEEN HEPATITIS A, B AND C VIRAL DISEASES:
Features Hepatitis A Hepatitis B Hepatitis C
1. Agent HAV HBV HCV
2. Genome RNA, ss, linear DNA, ss/ds RNA, ss, linear
circular
3. Spread Faeco-oral Parenteral, close contact Parenteral, close
contact
4. Incubation 15-45 days 30-180 days 20-90 days
period
5. Antigen HAV HBsAg HCV RNA
HBcAg C 100-3
HBeAg C 33c
HBxAg NS5
6. Antibodies anti-HAV anti-HBs anti-HBc anti- anti-HCV
HBe
7. Severity Mild Severe Moderate
8. Chronic state None Ocasional Common
9. Prognosis Excellent (self-limiting) Worse with age Moderate

b) LIVER FUNCTION TESTS


1. Tests for manufacture and excretion of bile
i) Serum bilirubin (percent conjugated bilirubin) (conjugated bilirubin/total
bilirubin x 100)
a. CB < 20% -------------------------------------hemolytic disease, Gilbert’s disease
b. CB 20-50% ------------------------------------Viral hepatitis
c. CB > 50% -------------------------------------primary biliary cirrhosis, Dubin Johnson
and Rotor syndrome, bile duct obstruction
ii) Bilirubin in faces ------------------------------ -Absent in biliary obstruction
iii) Bilirubin in urine --------------------------------Conjugated bilirubinuria in hepatitis

KMU Past Papers Solved SEQs


Gastrointestinal Tract, Liver and Pancreas 82

iv) Urobilinogen
v) Bile salt ------------------------------------------Increase in cholestasis
2. Serum enzyme assays
i) Alkaline phosphatase (ALP)---------------------Highest in biliary obstruction
ii) Gamma Glutamyl transpeptidase (GGT)--------Specific for hepatobiliary diseases
iii) Aspartate transaminase (ALT) ----------------- ALT > AST in viral hepatitis
iv) Alanine transaminase (AST)-------------------- AST > ALT in alcoholic hepatitis
v) Lactate dehydrogenase (LDH)----------------- Increased in tumors involving the liver
3. Tests for metabolic functions
i) Serum proteins ------------------------------ Hypoalbuminemia in hepatocellular
diseases
ii) Immunoglobulins ---------------------------- Nonspecific alterations in IgA, IgG and
IgM
iii) Clotting factors --------------------------- PT, APTT is prolonged in hepatocellular
diseases
iv) Plasma ammonia ---------------------------- Increased in acute fulminant hepatitis,
cirrhosis hepatic encephalopathy
v) Blood glucose and GTT----------------------decrease in hepatic necrosis
4. Immunologic tests
i) Mitochondrial antibodies ------------------ In primary biliary cirrhosis
ii) Hepatitis B anti bodies --------------------- In Hepatitis B infection
iii) Amoeba antibodies ------------------------ Amoebic liver abscess
5. Ancillary diagnostic tests
i) Ultrasound
ii) FNAC
iii) Percutaneous lever biopsy

Q-21 An old man of 50 years has the following LFT report


i. Serum bilirubin 10mg/dl.
ii. Serum SGPT 55U/L
iii. Serum AST 50 U/L
iv. Serum alkaline phosphatase 1200 U/L
v. Serum Albumin 2 Gm/dl
a) What is the most likely diagnosis?
b) Give the causes related to your diagnosis.
c) What other investigations will you advise for this patient?

ANS:
a) CHOLESTERIC LIVER DISEASE
b) CAUSES OF CHOLESTASIS
A) Intrahepatic cholestasis
1. Drugs like oral contraceptives pills and anabolic steroids
2. Neonatal hepatitis
3. Pregnancy induced
4. Cholestasis of sepsis
5. Bacterial cholangitis
6. Primary biliary cirrhosis
7. Primary sclerosing cholangitis

KMU Past Papers Solved SEQs


Gastrointestinal Tract, Liver and Pancreas 83

B) Extrahepatic cholestasis
1. Blockage of CBD
2. Cholelithiasis
3. Extra hepatic biliary atresia
4. Primary sclerosing cholangitis
5. Carcinoma of head of pancreas
c) INVESTIGATIONS:
1. Serum anti-mitochondrial antibodies (AMA)
2. Serum anti-Nuclear antibodies (ANA)
3. Ultrasound abdomen
4. Endoscopic retrograde cholangio-pancreato-graphy (ERCP)
5. Hepatobiliary imino-di-acetic acid (HIDA) radio-nucleotide scan
6. Plain X-ray
7. Helical CT scan--- show C sign in cancer of head of pancreas
8. FNAC liver
9. Percutaneous liver biopsy
10. CT guided percutaneous pancreas biopsy

Q-22 A 10-year-old child is brought to casualty for pain abdomen fever and vomiting of 3 days
duration. Relevant lab investigation reveal serum ALT level of 750IU/ml and positive igM anti
HAV.
a) name the disease that child is suffering form, its causative agent and incubation period.
b) Name the lesion caused by this agent.
c) What is the usual course of this disease and what is its mortality rate?
d) Write on sentence comment on this chronicity and carrier stage.
e) How is it transmitted and what is the clinical significance of igG and HAC?

ANS:
a) Hepatitis A
Hepatitis A virus
2-6 weeks
b) Acute hepatitis
Fulminant hepatitis rarely
c) Benign self-limiting disease with mortality rate of 0.1%
d) HAV doesn’t cause chronic hepatitis or carrier state
e) Transmitted through feco-oral route
igG antibody persists beyond convalescence and give primary defense against reinfection.

Q-23
a) Define Cholelithiasis
b) What are the risk factors for gall stones?
c) What are the four defects involved in the formation of cholesterol gall stones?

ANS:
a) CHOLELITHIASIS: mean gall stones, which commonly contain cholesterol, bile pigment
and calcium salts in varying proportions and are formed from constituents of the bile (viz.
cholesterol, bile pigments and calcium salts)
b) RISK FACTORS

KMU Past Papers Solved SEQs


Gastrointestinal Tract, Liver and Pancreas 84

1.Geography: prevalent in almost the entire Western world.


2.Genetic factors
3.Age: The incidence increases above the age of 40
4.Sex: Gallstones are twice more frequent in women than in men.
5.Obesity
6.Diet: Deficiency of dietary fiber content is linked to higher prevalence of gallstones
7.Drugs
8.Pregnancy
9.Acquired disorders: Crohn’s disease, ileal resection, ileal bypass surgery spinal cord
injury.
c) PATHOGENESIS:
1. Supersaturation of bile with cholesterol due to:
i) Increase cholesterol synthesis and production
ii) Reduced bile acid pool
iii) Increased conversion of cholic acid to deoxycholic acid
iv) Decreased lecithin due to MDR3 gene mutation
2. Nucleation of cholesterol monohydrate in mucin gel layer of gall bladder, due to
i) Increased pro nucleating factors (mucin and non-mucin glycoproteins)
ii) Decreased anti nucleating factors (apolipoproteins AI and AII)
3. Mucin hypersecretion and consequent entrapment of cholesterol crystal.
4. Gall bladder hypomobility resulting in stasis of biliary sludge and lithogenesis.
5. Pigmented gall stones formation is due to
i) Chronic hemolysis resulting in increased level of unconjugated bilirubin in bile
ii) Alcoholic cirrhosis
iii) Chronic biliary infections by parasites such as clonorchis sinensis and ascaris
lumbricoide

Q-24
a) Classify Hepatitis on the basis of etiology and pathogenesis,
b) How will you diagnose acute HBV in the lab?
c) What is the sequelae of chronic liver disease caused by HCV?

ANS:
a) HEPATITIS CLASSIFICATION
1. Viral hepatitis
2. Autoimmune hepatitis
3. Toxin induce hepatitis
4. Drug induced hepatitis
b) LAB DIAGNOSIS OF HBV
A. Serum enzyme assays
ALT and AST
B. Serum antibodies and antigens:
1. HBsAg (hepat itis B surface antigen) appears early in about 6 weeks of
infection. It usually disappears in 3-6 months. Its persistence for more than 6
months implies a carrier state. HBsAg may also be demonstrated in the cell
membrane of hepatocytes of carriers and chronic hepatitis patients by Orcein
staining (orange positivity)

KMU Past Papers Solved SEQs


Gastrointestinal Tract, Liver and Pancreas 85

2. Anti-HBs appears late, about 3 months after the onset. AntiHBs response may
be both IgM and IgG type
3. HBeAg present (3-6 weeks) during an acute attack.
4. Anti-HBe appears after disappearance of HBeAg. Seroconversion from HBeAg
to anti-HBe during acute stage of illness is a prognostic sign for resolution of
infection.
5. HBcAg HBcAg derived from core protein cannot be detected in the blood. But
HBcAg can be demonst rated in the nuclei of hepatocytes in carrier state and in
chronic hepatitis
6. Anti-HBc can be detected in the serum of acute hepatitis B patients during pre-
icteric stage. initially it is IgM class antibody (indicate recent acute infection)
which persists for 4-6 months and is followed later by IgG anti-HBc (indicate
infection in the remote past
7. HBV-DNA Detection of HBV-DNA by molecular hybridization using the
Southern blot technique is the most sensitive index of hepatitis B infection. It is
present in pre-symptomatic phase and transiently during early acute stage.
c) SEQUELAE OF CHRONIC LIVER DISEASE CAUSED BY HCV
1. Spontaneous remission of disease
2. Having disease without progression
3. Progression to cirrhosis which ultimately leads to
i) Liver failure
ii) Hepatic encephalopathy
iii) Hematemesis from esophageal varices
iv) Hepatocellular carcinoma
v) Death

Q-25 Some of the army jawans in their camp in flood affected area have developed loss of
appetite and vomiting. On examination there is tenderness in the right upper quadrant of the
abdomen. The urine shows the presence of bile pigments.
a) What is your diagnosis?
b) What relevant test will you perform on this patient?
c) How will you correlate the lab tests with pathogenesis?

ANS:
a) HEPATITIS A
b) TESTS:
1. Serum ALT
2. Serum anti HAV IgM
3. Serum anti HAV IgG
c) CORRELATION
1. Increase ALT show acute disease
2. Anti HAV IgM appear in the blood at the onset of symptoms,

KMU Past Papers Solved SEQs


Gastrointestinal Tract, Liver and Pancreas 86

3. Anti HAV IgG persist beyond convalescence and is the primary defense against
reinfection
Q-26
a) Define Cirrhosis?
b) What are the features of Cirrhosis?
c) What are the complications of Cirrhosis?
ANS:
a) CIRRHOSIS
Cirrhosis is defined as a diffuse process characterized by fibrosis and the conversion of
normal liver architecture into structurally abnormal nodules
b) FEATURES OF CIRRHOSIS:
1. It almost involves the entire liver and the normal lobular architecture of hepatic
parenchyma is disorganized.
2. Fibrous septa in the form of delicate bands or broad scars around multiple adjacent
lobules.
3. There is formation Parenchymal nodules, (micronodules— less than 3 mm in diameter
and macronodules —over 1 cm), encircled by these fibrous bands.
4. It occurs after hepatocellular necrosis of varying etiology.
c) COMPLICATIONS OF CIRRHOSIS:
1. hepatic failure
i) coagulation defects
ii) hypoalbuminemia
iii) hepatic encephalopathy
2. portal hypertension
i. splenomegaly
ii. esophageal varices
iii. hemorrhoids
3. ascites
i. bacterial peritonitis
4. hepatorenal syndrome
5. hyper-estrenism
i. gynecomastia
ii. infertility
iii. testicular atrophy
6. hepatocellular carcinoma

Q-27
a) What are different types of acute cholecystitis?
b) What could be the possible course of acute cholecystitis?
c) Just enumerate the complications of acute cholecystitis

ANS:
a) TYPES OF ACUTE CHOLECYCTITIS:
1. Acute calculous cholecystitis
2. Acucte acalculous cholecystitis
b) POSSIBLE COURSE OF ACUTE CHOLECYSTITIS:
1. Obstruction results in distension of the gallbladder in case of acute calculous
cholecystitis

KMU Past Papers Solved SEQs


Gastrointestinal Tract, Liver and Pancreas 87

2. Followed by acute inflammation which is due to


i) The action of phospholipases derived from the mucosa hydrolyzes biliary
lecithin to lysolecithin, which is toxic to the mucosa
ii) The normally protective glycoprotein mucous layer is disrupted, exposing the
mucosal epithelium to the direct detergent action of bile salts.
iii) Prostaglandins released within the wall of the distended gallbladder contribute
to mucosal and mural inflammation.
3. Distention and increased intraluminal pressure also may compromise blood flow
to the mucosa leads to hypoxic cell injury.
4. Later, secondary bacterial infection, chiefly by E. coli and Streptococcus faecalis,
supervenes
c) COMPLICATION OF ACUTE CHOLECYCYTIS:
1. Bacterial superinfection with cholangitis or sepsis
2. Gallbladder perforation and local abscess formation
3. Gallbladder rupture with diffuse peritonitis
4. Biliary enteric (cholecystenteric) fistula, with drainage of bile into adjacent
organs, entry of air
5. and bacteria into the biliary tree, and potentially gallstone-induced intestinal
obstruction (ileus)

Q-28 A biopsy report of a 55 years old lady, admitted to hospital for complaint of abdominal
distension and pallor, reveals loss of normal lobular architecture with replacement of liver
parenchyma by haphazardly regenerating nodules surrounded by coarse fibrous septa
a) Name the disease, the lady is most probably suffering from. 2
b) Enlist only 05 important and common causes of this disease. 2.5
c) Enumerate only five complications of this disease 2.5
d) What is the most important complication of ascites in this disease? 1
e) name only four poor prognostic signs of this disease 2

ANS:
a) LIVER CIRRHOSIS
b) CAUSES:
1. Alcoholic liver disease
2. Viral hepatitis B and C
3. Autoimmune disease
i) Primary biliary cirrhosis
ii) Autoimmune hepatitis
4. Metabolic diseases
i) Hemochromatosis
ii) Wilson disease
iii) Alpha antitrypsin deficiency
iv) Galactosemia
5. Non-alcoholic stetho-hepatitis
c) Complication of cirrhosis:
1. Hepatic failure
i) Coagulation defects
ii) Hypoalbuminemia
iii) Hepatic encephalopathy
2. Portal hypertension

KMU Past Papers Solved SEQs


Gastrointestinal Tract, Liver and Pancreas 88

i) Splenomegaly
ii) Esophageal varices
iii) Hemorrhoids
3. Ascites
i. Bacterial peritonitis
4. Hepatorenal syndrome
5. Hyper-estrenism
i) Gynecomastia
ii) Infertility
iii) Testicular atrophy
6. Hepatocellular carcinoma
d) Bacterial peritonitis
e) Prognostic signs
1. Hepatic encephalopathy
2. Hepatorenal syndrome
3. Ascites
4. Coagulation defects

Q-29
a) Define cirrhosis 2
b) Classify cirrhosis on the basis of size of nodules 2
c) Briefly discuss the laboratory diagnosis of Hepatitis B infection and the sequence of rise
and fall of serological markers of Hepatitis B virus. 6
ANS:
a) CIRRHOSIS:
Cirrhosis is defined as a diffuse process characterized by fibrosis and the conversion of
normal liver architecture into structurally abnormal nodules
b) CIRRHOSIS CLASSIFICATION:
1. MICRO-NODULAR CIRRHOSIS: in which the nodules are less than 3 mm in diameter.
The micro-nodular cirrhosis includes alcoholic cirrhosis
2. MACRO-NODULAR CIRRHOSIS in this type, the nodules are generally larger than 1
cm in diameter. macro-nodular cirrhosis corresponds to post-necrotic or post
hepatitis cirrhosis.
3. MIXED CIRRHOSIS in mixed type, some parts of the liver show micro-nodular
appearance while other parts show macro-nodular pattern.
c) LABORATORY DIAGNOSIS OF HEPATITIS B INFECTION:
1. HBsAg appear before the onset of symptoms and peak during overt disease and then
decline and undetectable in 3 to 6 months
2. Anti HBs antibody rise few week or several months after the disappearance of HBsAg,
and persist lifelong conferring the immunity
(time taken form disappearance of HBsAg to the rise in Anti HBs is called windows
period in this time serum IgM Anti HBc is diagnostic)
3. HBeAg, HBV-DNA and DNA polymerase appear after HBsAg, it signifies active viral
replication
4. IgM anti HBc appear before the onset of symptoms, over a period of months IgM is
replaced by IgG anti HBc
5. Serum aminotransferases rise in acute disease.

KMU Past Papers Solved SEQs


Gastrointestinal Tract, Liver and Pancreas 89

Q-30
a) Name four Cholestatic liver diseases. 2
b) What are the others three associated conditions with primary biliary cirrhosis? 1.5
c) Write down the 3 associated conditions with sclerosing cholangitis? 1.5
d) Tabulate the differences between Hepatitis A, B & C viruses. 5

ANS:
a) CHOLESTATIC LIVER DISEASE
1. Neonatal cholestasis
2. Cholestasis of sepsis
3. Primary biliary cirrhosis
4. Primary sclerosing cholangitis
b) ASSOCIATED CONDITION WITH PRIMARY BILIARY CIRRHOSIS:
1. Sjogren syndrome
2. Scleroderma
3. Thyroid disease
c) ASSOCIATED CONDITION WITH PRIMARY SCLEROSING CHOLANGITIS
1. Inflammatory bowel disease (ulcerative colitis mainly)
2. Pancreatitis
3. Idiopathic fibrosing disease (retroperitoneal fibrosis)
d) GIVEN SOME WHERE ABOVE**

Q-31
a) What are the etiological factors in acute pancreatitis?
b) What are the salient features of acute pancreatitis?
c) What are the Bio chemical findings in acute pancreatitis?

ANS:
a) ETIOLOGICAL FACTORS
i) Metabolic
1. Alcoholism
2. Hyperlipoproteinemia hypertriglyceridemia
3. Hypercalcemia, hyperparathyroidism
4. Drugs (e.g., azathioprine anticonvulsant)
ii) Genetic
1. Mutations in the cationic trypsinogen (PRSS1) and trypsin inhibitor (SPINK1)
genes
iii) Mechanical
1. Trauma
2. Gallstones
3. Iatrogenic injury
4. Pancreatic cancer
5. Pancreatic divism
6. Biliary sludge
7. Parasites ie ascaris lumbrecoids
8. Perioperative injury Endoscopic procedures with dye injection
iv) Vascular
1. Shock

KMU Past Papers Solved SEQs


Gastrointestinal Tract, Liver and Pancreas 90

2. Atheroembolism
3. Polyarteritis nodosa
v) Infectious
1. Mumps
2. Coxsackievirus
b) SALIENT FEATURES ??
c) BIOMARKERS/ LABORATORY DIAGNOSIS:
1. Serum amylase
2. Urine amylase
3. Serum lipase
4. Serum immunoreactive trypsin (SIT)
5. Decreased fecal elastase
6. Neutrophilic leukocytosis
7. Hypocalcemia
8. Hyperglycemia
9. CT scan MRI and x ray

Q-32
a) Define acute Pancreatitis.
b) How will you diagnose in the laboratory?
c) Enumerate complications of acute Pancreatitis,

ANS:
a) ACUTE PANCREATITIS:
Acute pancreatitis is a reversible inflammatory disorder that varies in severity, ranging
from focal edema and fat necrosis to widespread hemorrhagic parenchymal necrosis
b) LABORATORY DIAGNOSIS:
Given somewhere else
c) COMPLICATIONS:
1. Pancreatic necrosis
2. Pancreatic pseudocyst
3. Pancreatic abscess
4. Pancreatic ascites
5. Chronic pancreatitis
Clinical complications
1. Shock
2. Respiratory distress syndrome
3. Acute tubular necrosis ATN
4. Disseminated intravascular coagulation (DIC)

KMU Past Papers Solved SEQs


Kidney Male Female Genital System and Lower Urinary Tract 91

Q-1
a) Classify tumors of the Ovary. 4
b) Give morphology of serous tumor of Ovary. 5
c) Enumerate the investigations for the diagnosis of serous tumor. 1

ANS:
a) CLASSIFY OVARIAN TUMORS.
1. Tumors of surface epithelium (common epithelial tumors) (60-70%)
A. Serous tumors
i) Serous cystadenoma
ii) Borderline serous tumor
iii) Serous cystadenocarcinoma
B. Mucinous tumors
i) Mucinous cystadenoma
ii) Borderline mucinous tumor
iii) Mucinous cystadenocarcinoma
C. Endometrioid tumors
D. Clear cell (mesonephroid) tumors
E. Brenner tumors
2. Germ cell tumors (15-20%)
A. Teratomas
i) Benign (mature, adult) teratoma
 Benign cystic teratoma (dermoid cyst)
 Benign solid teratoma
ii) Malignant (immature) teratoma
iii) Monodermal or specialized teratoma
 Struma ovarii
 Carcinoid tumor
B. Dysgerminoma
C. Endodermal sinus (yolk sac) tumor
D. Choriocarcinoma
E. Others (embryonal carcinoma, polyembryoma, mixed germ cell tumors)
3. Sex cord-stromal tumors (5-10%)
A. Granulosa-theca cell tumors
i) Granulosa cell tumor
ii) Thecoma
iii) Fibroma
B. Sertoli-Leydig cell tumors (Androblastoma, arrhenoblastoma)
C. Gynandroblastoma
4. Miscellaneous tumors
A. Lipid cell tumors
B. Gonadoblastoma
5. Metastatic tumors (5%)
A. Krukenberg tumor
B. Mullerian tumor

b) MORPHOLOGY OF SEROUS TUMOR OF OVARY.

A. Gross appearance:

KMU Past Papers Solved SEQs


Kidney
Kidney Male
Male Female
Female Genital
Genital System
System and
and Lower
Lower Urinary
Urinary Tract
Tract 92

1. Most serous tumors are large, spherical to ovoid, cystic structures with smooth and
glistering serosal covering. By contrast, the surface of the cystadenocarcinoma has
nodular irregularities.
2. small cystic tumors may have a single cavity, but larger ones have multiloculated
masses.
3. The cystic spaces usually are filled with a clear serous fluid.
4. papillary projections are present which Protrude into the cystic cavities (more
prominent in malignant tumors)
B. MICROSCOPIC APPEARANCE:
1. SEROUS CYSTADENOMA Clearly Benign tumors contain a single layer of tall
columnar epithelial often are ciliated, that line the cyst or cysts and Papillary
Projection. but does not invade fibro-vascular stromal stalk of papillae.
2. SEROUS CYSTADENOCARCINOMA Clearly malignant carcinoma, there is
anaplasia of the lining cells appears, multilayering, loss of basal polarity and nests
or undifferentiated sheets of malignant cells does invade the fibrovascular
stromal stalk of papillae. Psammoma bodies are common in the tips of papillae.
malignant serous tumors spread to periaortic lymph nodes;
3. BORDER LINE TUMORS: Tumors of low malignant potential Between clearly
benign and frank carcinoma forms, having stratification (2-3 layers) of the
epithelial cells, less cytologic atypia maintained basal polarity of nuclei, moderate
nuclear abnormalities, and some mitotic activity. typically, little or no stromal
invasion.
c) INVESTIGATIONS FOR THE DIAGNOSIS OF SEROUS TUMOR
the protein CA-125, is elevated in the sera of 75% to 90% of women with epithelial
ovarian cancer.

Q-2
a) Classify Testicular tumors. 3
b) Describe the morphology of Seminoma testes. 7
ANS:
a) CLASSIFICATION OF TESTICULAR TUMORS.
all testicular tumors are divided into 3 groups: germ cell tumors, sex cord-stromal tumors and
mixed forms.
I. Germ Cell Tumors
C. Seminoma
D. Spermatocytic seminoma
E. Embryonal carcinoma
F. Yolk sac tumor (Syn. endodermal sinus tumor, orchioblastoma, infantile type
embryonal carcinoma)
G. Polyembryoma
H. Choriocarcinoma
I. Teratomas
i) Mature
ii) Immature
iii) With malignant transformation
J. Mixed forms

KMU Past Papers Solved SEQs


Kidney Male Female Genital System and Lower Urinary Tract 93

II. Sex Cord-Stromal Tumors


1. Leydig cell tumor
2. Sertoli cell tumor (Androblastoma)
3. Granulosa cell tumor
4. Mixed forms
III. Combined Germ Cell-Sex Cord-Stromal Tumors
1. Gonadoblastoma
IV. Other Tumors
1. Malignant lymphoma (5%)

b) MORPHOLOGY OF SEMINOMA TESTES.


Grossly,
1. the involved testis is enlarged up to 10 times, the tumor rarely invades the tunica.
(potato tumor)
2. Cut section of the affected testis shows homogeneous, grey-white lobulated
appearance
3. Large tumors may contain foci of coagulation necrosis, usually without hemorrhage
Microscopically,
1. Tumor cells
The seminoma cells generally lie in cords, sheets or columns forming lobules.
The tumor cells are uniform in size with clear cytoplasm and well-defined cell
borders.
The cytoplasm contains glycogen that stains positively with PAS reaction.
The nuclei are centrally located, large, hyperchromatic and usually contain 1-2
prominent nucleoli.
Tumor giant cells may be present.
Mitotic figures are infrequent. However, anaplastic seminomas have increased mitotic
activity and have aggressive behavior.
2. Stroma
The stroma of seminoma is delicate fibrous tissue which divides the tumor into
lobules. The stroma shows a characteristic lymphocytic infiltration, indicative of
immunologic response of the host to the tumor.
The tumors show granulomatous reaction in the stroma.
Sometime syncytiotrophoblasts are present that are the source of the minimally
elevated serum HCG concentrations encountered in some males with pure seminoma.

Q-3
a) Define nephritic syndrome. 2
b) Enumerate primary glomerular diseases. 4
c) Briefly give the pathogenesis of anti-glomerular basement disease. 4

ANS:
a) NEPHROTIC SYNDROME:

KMU Past Papers Solved SEQs


Kidney Male Female Genital System and Lower Urinary Tract 94

DEFINITION glomerular injury is primarily due to neutrophils and is dominated by the


acute onset of usually grossly visible hematuria (red blood cells and red cell casts in
urine), proteinuria of mild to moderate degree, azotemia, edema, hypertension and
oliguria.
b) PRIMARY GLOMERULAR DISEASES:
1. Minimal change disease
2. Focal segmental glomerulosclerosis
3. Membranous nephropathy
4. Acute post infectious GN
5. Membranoproliferative GN
6. IgA nephropathy.
c) PATHOGENESIS OF ANTI-GLOMERULAR BASEMENT DISEASE:
In this type of injury, antibodies are directed against fixed antigens in the GBM A
conformational change in the α3 chain of the type IV collagen of the GBM appears to be
key in inciting autoimmunity. anti-GBM antibodies (mostly IgG) and complement (mainly
C3) deposit along the glomerular basement membrane.

Q-4
a) Define nephrotic syndrome. 2
b) Enumerate glomerular changes in diabetes mellitus. 3
c) Define pyelonephritis
d) Enumerate type of pyelonephritis and causes separately.
e) Microscopic features of chronic pyelonephritis. 5

ANS:
a) NEPHROTIC SYNDROME:
is a glomerular syndrome characterized by heavy proteinuria (excretion of greater than
3.5 g of protein/day in adults), hypoalbuminemia, severe edema, hyperlipidemia, and
lipiduria (lipid in the urine) and hypercoagulability.

b) GLOMERULAR CHANGES IN DIABETES MELLITUS.


1. Diffuse glomerulosclerosis: The pathologic changes consist of thickening of the
GBM and diffuse increase in mesangial matrix with mild proliferation of mesangial
cells. Various exudative lesions such as capsular hyaline drops and fibrin caps
present
2. Nodular glomerulosclerosis (Kimmelstiel-Wilson disease): The pathologic changes
consist of one or more nodules in a few or many glomeruli. Nodule is an ovoid or
spherical, laminated, hyaline, acellular mass located within a lobule of the
glomerulus. As the nodular lesions enlarge, they compress the glomerular
capillaries and obliterate the glomerular tuft resulting in renal ischemia occurs
leading to tubular atrophy and interstitial fibrosis and results in grossly small,
contracted kidney.
3. Osmotic damage to glomerular capillary endothelial cells
4. Hyper filtration damage to the mesangium
5. Diabetic microangiopathy which increased deposition of type IV collagen in the
GBM, tubular cell basement membranes, and mesangium
c) CHRONIC PYELONEPHRITIS:
it is interstitial inflammation and scarring of the renal parenchyma are associated grossly
visible scarring and deformity of the pelvicalyceal system

KMU Past Papers Solved SEQs


Kidney Male Female Genital System and Lower Urinary Tract 95

d) TYPES AND CAUSES.


1. ACUTE
Urinary tract infections
2. CHRONIC OBSTRUCTIVE
Calculi
Unilateral obstructive lesion of the ureter
Congenital anomelies of Urethera
3. CHRONIC REFLUX ASSOCIATED
Vesicoureteral reflex
Intrarenal reflex
Spinal cord injury
Neurogenic bladder dysfunction secondary to diabetes.
e) MICROSCOPIC FEATURES OF CHRONIC PYELONEPHRITIS
a. GROSSLY,
The kidneys are usually small and contracted
The surface of the kidney is irregularly scarred and show characteristic U-shaped
depressions on the cortical surface.
There is generally blunting and dilatation of calyces (calyectasis) and dilated
pelvis of the kidney
b. MICROSCOPICALLY,
i) Interstitium
There is chronic interstitial inflammatory reaction, chiefly composed of
lymphocytes, plasma cells and macrophages with interstitial fibrosis.
When there is collection of foamy macrophages admixed with other
inflammatory cells and giant cells  Xanthogranulomatous pyelonephritis
ii) Tubules
The tubules show varying degree of atrophy and dilatation.
Thyroidization of tubules is present (Many of the dilated tubules contain
(eosinophilic) pink to blue, glassy-appearing PAS-positive casts, known as
colloid casts, that suggest the appearance of thyroid tissue).
A few tubules may contain neutrophils.
iii) Pelvicalyceal system
The renal pelvis and calyces are dilated.
The walls show chronic inflammation Lymphoid follicles with germinal
centers and fibrosis
The lining epithelium may undergo squamous metaplastic change.
iv) Blood vessels
show obliterative endarteritis and hypertensive hyaline arteriolosclerosis.
v) Glomeruli
periglomerular fibrosis, hyalinization of glomeruli and Glomerulosclerosis is
present, later is develops as a secondary process caused by nephron loss

Q-5
a) Classify Renal tumors 2
b) Give pathogenesis of carcinoma kidney 4
c) Morphology of Wilm's tumor 4

ANS:
a) RENAL TUMORS CLASSIFICATION

KMU Past Papers Solved SEQs


Kidney Male Female Genital System and Lower Urinary Tract 96

1. Benign
i) Oncocytoma
ii) Adenoma
iii) Angiomyolipomas
iv) Cortical papillary adenoma
2. Malignant
i) Renal cell carcinoma (Synonyms: Adenocarcinoma, Hypernephroma, Grawitz
Tumor)
a. Clear Cell Carcinomas (most common 65%)
b. Papillary Renal Cell Carcinomas (10% to 15%)
c. Chromophobe Renal Carcinomas (5%)
d. granular cell
e. sarcomatoid and
f. collecting duct type
ii) Wilms’ tumor (nephroblastoma)
iii) Transitional Cell Carcinoma
b) PATHOGENESIS OF CARCINOMA KIDNEY
1. Genetic factors Heredity
Clear cell carcinoma:
i) Patients have germline mutations of tumor suppressor VHL gene (a tumor
suppressor gene) located on chromosome 3p (band 3p25) and loss of second
allele by somatic mutation or hypermethylation, The product VHL gene causes
the degradation of hypoxia-induced factors (HIFs), and in the absence of VHL,
HIFs are stabilized and contribute to carcinogenesis by stimulating the
expression of VEGF, an important angiogenic factor, as well as a number of
other genes that drive tumor cell growth.
ii) Cytogenetic abnormality in chromosome 3.
iii) loss of function mutation in genes which epigenetic processes ie histones
methylation.
Papillary Renal Cell Carcinomas: MET proto-oncogene, located on chromosomal 7
(sub-band 7q31). The MET gene is a tyrosine kinase receptor for the growth factor
called hepatocyte growth factor. Activity of gene increase due to activation mutation
or duplications of chromosome 7. This leads to abnormal growth in proximal tubular
epithelium.
Chromophobe RCC: there is losses of entire chromosomes, including chromosomes 1,
2, 6, 10, 13, 17, and 21. Thus, they show extreme hypodiploidy.
2. Tobacco is the major risk factor for RCC, whether chewed or smoked and accounts for
20-30% cases of RCC. Cigarette smokers have two-fold higher risk of developing RCC.
3. Cystic diseases of the kidneys increase the risk of development of RCC. Patients on
long-term dialysis develop acquired cystic disease which may evolve into RCC and
adenomas. Adult polycystic kidney disease and multicystic nephroma is associated
with higher occurrence of papillary RCC.
4. Other risk factors
i) Hypertension
ii) Exposure to asbestos, cadmium, heavy metals and petrochemical products.
iii) In women, obesity and estrogen therapy.
iv) Analgesic nephropathy.
v) chronic renal failure
vi) Tuberous sclerosis.
c) MORPHOLOGY OF WILM'S TUMOR

KMU Past Papers Solved SEQs


Kidney Male Female Genital System and Lower Urinary Tract 97

GROSSLY Wilms tumor typically is a large, solitary, well-circumscribed mass, on cut


section, the tumor shows appear soft, fish flesh-like grey-white to cream-yellow tumor
with foci of necrosis and hemorrhages and grossly identifiable myxomatous or
cartilaginous elements
On microscopic examination,
1. The classic tri-phasic combination of blastemal, stromal, and epithelial cell types is
observed in most lesions,
i) blastemal component: Sheets of small blue cells, with few distinctive features
ii) Epithelial differentiation: abortive tubules and poorly-formed glomerular
structures are Present
iii) Stromal cells: usually fibrocytic or myxoid in nature. Rarely, other
heterologous elements are identified, including squamous or mucinous
epithelium, smooth muscle, adipose tissue, cartilage, and osteoid and
neurogenic tissue.
2. Approximately 5% of tumors contain foci of anaplasia.
3. Nephrogenic rests: precursor lesions of Wilms tumors present in the renal
parenchyma adjacent to the tumor.
Nephrogenic rests may be;
i) Hyperplastic rests, expansile masses that resemble Wilms tumors.
ii) Sclerotic rests consisting predominantly of fibrous tissue with occasional admixed
immature tubules of glomeruli.

Q-6
a) What is Dysfunctional uterine bleeding 2
b) Enumerate causes of Dysfunctional uterine bleeding 5
c) Give morphological features of endometrial hyperplasia 3

ANS:
A) DYSFUNCTIONAL UTERINE BLEEDING
Dysfunctional uterine bleeding (DUB) may be defined as excessive bleeding occurring during
or between menstrual periods without a causative uterine lesion such as tumor, polyp,
infection, hyperplasia, trauma, blood dyscrasia or pregnancy.
B) CAUSES OF DYSFUNCTIONAL UTERINE BLEEDING
1. ANOVULATORY CYCLES: due to
i) hypothalamic-pituitary axis, adrenal, or thyroid dysfunction;
ii) functional ovarian lesions producing excess estrogen;
iii) malnutrition, obesity, or debilitating disease
iv) severe physical or emotional stress.
v) ovulatory failure results in an excess of estrogen relative to progesterone.
So there is a proliferative phase that is not followed by the normal secretory phase.
The endometrial glands develop disorderly while the endometrial stroma, which
requires progesterone for growth, may be scarce.
This combination of abnormalities makes the endometrium prone to breakdown and
abnormal bleeding.
2. Inadequate luteal phase.
The corpus luteum regress prematurely leads to decreased progesterone. The
endometrium fails to show the expected secretory changes.
3. Contraceptive-induced bleeding.

KMU Past Papers Solved SEQs


Kidney Male Female Genital System and Lower Urinary Tract 98

4. ENDOMETRIAL DISORDERS: chronic endometritis, endometrial polyps, and


submucosal leiomyomas
5. COMPLICATIONS OF PREGNANCY (abortion, trophoblastic disease, ectopic
pregnancy)
b) MORPHOLOGICAL FEATURES OF ENDOMETRIAL HYPERPLASIA
a) Simple hyperplasia without atypia: (cystic glandular hyperplasia, CGH)
1. Characterized by the presence of varying sized glands, many of which are large
and cystically dilated and are lined by atrophic epithelium.
2. Mitoses are scanty and there is no atypia.
3. The stroma between the glands is sparsely cellular and edematous
b) Complex hyperplasia without atypia:
1. The glands are increased in number, exhibit variation in size and are irregular in
shape.
2. The glands are lined by multiple layers of tall columnar epithelial cells with large
nuclei which have not lost basal polarity
3. There is no significant atypia.
4. The glandular epithelium form papillary in-folds into the stroma
5. The stroma is generally dense, cellular and compact
c) Complex hyperplasia with atypia:
1. atypical cells in the hyperplastic epithelium which may be mild, moderate or
severe.
2. The cytologic features are loss of polarity, large size, irregular and hyperchromatic
nuclei, prominent nucleoli, and altered nucleocytoplasmic ratio.
3. this form is endometrial intraepithelial neoplasia (EIN) or carcinoma in situ for
this form of endometrial hyperplasia.

Q-7
a) Briefly give the pathogenesis of carcinoma cervix 4
b) Give an account of staging of carcinoma cervix 2
c) Give morphology of Carcinoma cervix 4

ANS:
a) PATHOGENESIS OF CARCINOMA CERVIX:
HPV, the causative agent of cervical neoplasia,
Most HPV infections cause acute and chronic inflammatory response.
But some infections persist, and progress to precancerous epithelial change termed
cervical intraepithelial neoplasia (CIN), CIN usually starts as low-grade dysplasia (CIN I)
to moderate (CIN II) and then severe dysplasia (CIN III) over time
CIN transform into invasive cervical carcinomas in many years.
Important risk factors related to HPV exposure include
1. Early age at first intercourse
2. Multiple sexual partners
3. Male partner with multiple previous sexual partners
4. Persistent infection by high-risk strains of papillomavirus
HPV infect immature squamous cells of the basal layer of the transformation zone.
HPV-infected squamous cells express of two potent oncoproteins encoded in the HPV
genome called E6 and E7.
The E6 and E7 proteins bind and inactivate tumor suppressors gene (p53 and Rb).

KMU Past Papers Solved SEQs


Kidney Male Female Genital System and Lower Urinary Tract 99

Activate cyclin E and promote growth and increased susceptibility to additional


mutations, suppression of DNA damage induced apoptosis that may eventually lead to
carcinogenesis.
HPV can be classified as high-risk or low-risk
i) High-risk type HPV, most commonly of types 16 and 18 (in 70% cases), and less
often types 31, 33, 52 and 58, found in cervical cancer.
ii) Low-risk type HPV types 6 and 11 are found most frequently in condylomas.
high-risk HPV integrate into the host genome
Low risk does not integrate into the host genome, remaining instead as free episomal
viral DNA.

HPV is not sufficient to drive the neoplastic process. Some other factors such as immune
and hormonal status, co-infection with other sexually transmitted agents, cigarette
smoking women, users of oral contraceptives, and HIV infection.
Mutations in the tumor suppressor gene LKB1 were identified in more than 20% of
cervical cancers. The LKB1 protein is a kinase that phosphorylates and activates AMPK, a
metabolic sensor. AMPK in turn regulates cell growth through the mTOR complex.

b) STAGING OF CARCINOMA CERVIX


A. Stage 0: CIS
B. Stage I.
Ia. invasion not deeper than 3 mm not wider than 7 mm
Ib. invasion deeper than 3 mm and wider than 7 mm
C. Stage II. Extend beyond cervix, involve upper third of vagina but not beyond pelvic
wall
D. Stage III. Extend beyond pelvic wall involve lower third of vagina
E. Stage IV. Involve bladder and rectum
c) MORPHOLOGY OF CERVICAL CARCINOMA.
A. Grossly:
1. Invasive carcinomas show grossly conspicuous exophytic tumors encircling the
cervix and penetrating into the underlying stroma produce a barrel cervix,
2. The diagnosis can be suspected clinically on the basis of Schiller’s test done on
bedside
B. Histologically:
1. cellular atypia found in dysplasia or carcinoma in situ found in squamo-columnar
junction or transitional zone.
2. Dysplastic cells are distributed in the layers of squamous epithelium for varying
thickness,
3. Dysplasia is graded as mild, moderate and severe dysplasia, and carcinoma in situ
4. In mild dysplasia (CIN-1), the abnormal cells extend up to one-third thickness from
the basal to the surface layer
5. In moderate dysplasia (CIN-2) up to two-thirds
6. In severe dysplasia (CIN-3), these cells extend more than one third thickness of
epithelium
7. In carcinoma in situ (included in CIN-3), the entire thickness from the basement
membrane to the surface shows dysplastic cells.
8. The individual dysplastic or abnormal cells have features like crowding of cells,
pleomorphism, high nucleocytoplasmic ratio, coarse and irregular nuclear
chromatin, numerous mitoses and scattered dyskaryotic cells.

KMU Past Papers Solved SEQs


Kidney Male Female Genital System and Lower Urinary Tract 100

Q-8
a) Enumerate common forms of renal cell carcinoma. 3
b) Write the salient features of morphology (gross and microscopic) clear cell carcinoma 3+4

ANS:
a) FORMS OF RENAL CELL CARCINOMA.
1. Clear cell carcinoma
2. Papillary renal cell carcinoma
3. Chromophobe renal cell carcinoma
4. Granular cell type RCC
5. Sarcomatoid type RCC
6. Collecting duct type RCC

b) MORPHOLOGY
Grossly,
RCC commonly arises from the poles of the kidney as a solitary and unilateral tumor,
The tumor is generally large, golden yellow and circumscribed. They may arise anywhere in
the cortex
Cut section
renal cell carcinomas are yellow to orange to gray-white and commonly shows large areas of
ischemic necrosis, cystic change and foci of hemorrhages.
The margins of the tumor are well defined.
Sometime small satellite nodules are found, As the tumor enlarges, it may fungate through
the walls of the collecting system, extending through the calyces and pelvis as far as the
ureter. presence of tumor thrombus in the renal vein which may extend into the vena cava
and even into the right side of the heart
Occasionally, direct invasion into the perinephric fat and adrenal gland may be seen.
Histologically,
clear cells
The clear cytoplasm of tumor cells is due to removal of glycogen and lipid from the
cytoplasm during processing of tissues.
the tumor cells of clear cell renal cell carcinoma may appear almost vacuolated or may be
solid The classic vacuolated (lipid-laden) or clear cells, with small and round nuclei, are
demarcated only by their cell membranes.
granular cells,
resembling the tubular epithelium, which have small, round, regular nuclei enclosed within
granular pink cytoplasm.
Some tumors are highly anaplastic, with numerous mitotic figures and markedly enlarged,
hyperchromatic, pleomorphic nuclei.
The cells may form abortive tubules or may cluster in cords or disorganized masses
The stroma is usually scant but highly vascularized.

Q-9
a) What are the risk factors for cervical cancer? 3
b) Give the Pathogenesis of cancer of cervix due to human papilloma virus. 3
c) Morphology of cervical carcinoma. 4
ANS:

KMU Past Papers Solved SEQs


Kidney Male Female Genital System and Lower Urinary Tract 101

a) RISK FACTORS FOR CERVICAL CANCER


1. Early age at first intercourse
2. Multiple sexual partners
3. Male partner with multiple previous sexual partners
4. Persistent infection by high-risk strains of papillomavirus
5. Mutations in the tumor suppressor gene LKB1 were identified in more than 20% of
cervical cancers.
6. other factors
i) Immunodeficiency and hormonal abnormality,
ii) Co-infection with other sexually transmitted agents,
iii) Cigarette smoking women,
iv) Users of oral contraceptives,
v) HIV infection.
vi) Homosexual
vii) Increase parity
b) PATHOGENESIS OF CANCER OF CERVIX DUE TO HUMAN PAPILLOMA VIRUS.
1. High-risk type HPV, most commonly of types 16 and 18 and less often types 31, 33,
52 and 58 infect immature squamous cells of the basal layer of the transformation
zone.
2. HPV integrate into the host genome
3. HPV-infected squamous cells express of two potent oncoproteins encoded in the HPV
genome called E6 and E7.
4. The E6 and E7 proteins bind and inactivate tumor suppressors gene (p53 and Rb).
5. cyclin E which is held inactivated by tumor suppressors gene get activated and
promote growth and increased susceptibility to additional mutations, suppression of
DNA damage induced apoptosis that may eventually lead to carcinogenesis.
c) MORPHOLOGY OF CERVICAL CARCINOMA.
Given somewhere else**

Q-10
a) Give the gross and microscopic features of Wilm's tumor (Nephroblastoma). 3+4
b) What are the different syndromes associated with this tumor and their genetic abnormality?
3
ANS:
a) MORPHOLOGY OF WILM'S TUMOR
GROSSLY Wilms tumor typically is a large, solitary, well-circumscribed mass, on cut
section, the tumor shows appear soft, fish flesh-like grey-white to cream-yellow tumor
with foci of necrosis and hemorrhages and grossly identifiable myxomatous or
cartilaginous elements
On microscopic examination,
1. The classic tri-phasic combination of blastemal, stromal, and epithelial cell types is
observed in most lesions,
i) Blastemal component: Sheets of small blue cells, with few distinctive features
ii) Epithelial differentiation: abortive tubules and poorly-formed glomerular
structures are Present
iii) Stromal cells: usually fibrocytic or myxoid in nature. Rarely, other heterologous
elements are identified, including squamous or mucinous epithelium, smooth
muscle, adipose tissue, cartilage, and osteoid and neurogenic tissue.

KMU Past Papers Solved SEQs


Kidney Male Female Genital System and Lower Urinary Tract 102

2. Approximately 5% of tumors contain foci of anaplasia.


3. Nephrogenic rests: precursor lesions of Wilms tumors present in the renal
parenchyma adjacent to the tumor.
Nephrogenic rests may be;
i) Hyperplastic rests, expansile masses that resemble Wilms tumors.
ii) Sclerotic rests consisting predominantly of fibrous tissue with occasional admixed
immature tubules of glomeruli.
b) SYNDROMES ASSOCIATED WITH THIS TUMOR AND THEIR GENETIC ABNORMALITY.
1. WAGR syndrome (Wilms tumor, aniridia, genital abnormalities, and mental
retardation) genetic abnormality is deletion of Wilms tumor 1 gene (WT1), located on
11p13.
2. Denys-Drash syndrome (DDS) genetic abnormality is negative inactivating mutation
in a critical region of the Wilms tumor 1 gene WT1, located on 11p13. The syndrome
is characterized by gonadal dysgenesis and renal abnormalities.
3. Beckwith-Wiedemann syndrome (BWS) with genetic abnormality in locus that is
involved in these patients is in band p15.5 of chromosome 11 distal to the WT1 locus
called WT2. Patients having this syndrome exhibit enlargement of individual body
organs (e.g., tongue, kidneys, or liver) or entire body segments (hemihypertrophy);
enlargement of adrenal cortical cells (adrenal cytomegaly)

Q-11
a) Classify testicular germ cell tumors. 4
b) Give a brief account of gross and microscopic appearance of seminoma testis 2+4

ANS:
a) CLASSIFICATION:
I. GERM CELL TUMORS
1. Seminoma
2. Spermatocytic seminoma
3. Embryonal carcinoma
4. Yolk sac tumor (Syn. endodermal sinus tumor, orchioblastoma, infantile type
embryonal carcinoma)
5. Polyembryoma
6. Choriocarcinoma
7. Teratomas
iv) Mature
v) Immature
vi) With malignant transformation
8. Mixed forms
b) MORPHOLOGY OF SEMINOMA TESTES.
GIVEN SOMEWHERE ELSE**

Q-12
a) Write the staging of carcinoma bladder. 3
b) Write Grading System Classification of carcinoma bladder. 3
c) Give an account of the risk factors and morphology of carcinoma bladder. 4

ANS:

KMU Past Papers Solved SEQs


Kidney Male Female Genital System and Lower Urinary Tract 103

a) STAGING OF CARCINOMA BLADDER:


1. Stage 0: Carcinoma confined to the mucosa.
2. Stage A: Carcinoma invades the lamina propria but not the muscularis.
3. Stage B1: Carcinoma invades the superfcial muscle layer.
4. Stage B2: Carcinoma invades the deep muscle layer.
5. Stage C: Carcinoma invades the perivesical tissues.
6. Stage D1: Carcinoma shows regional metastases.
7. Stage D2: Carcinoma shows distant metastases
b) GRADING SYSTEM CLASSIFICATION OF TUMORS
1. Papilloma
2. papillary urothelial neoplasm of low malignant potential (PUNLMP)
3. low-grade papillary urothelial carcinoma
4. high-grade papillary urothelial carcinoma
c) RISK FACTORS:
1. Smoking Tobacco smoking
2. Industrial occupations Workers in industries that produce aniline dyes, rubber,
plastic, textiles, and cable whose carcinogens are Beta naphthylamine and benzene.
3. Schistosomiasis in endemic countries Egypt and Sudan, it induces local irritant effect
4. Dietary factors Certain carcinogenic metabolites of tryptophan.
5. Local lesions such as ectopia vesicae (extrophied bladder), vesical diverticulum,
leukoplakia of the bladder mucosa and urinary diversion in defunct ionalised bladder
6. Drugs Immunosuppressive therapy with cyclophosphamide and patients having
analgesic-abuse (phenacetin-) nephropathy.
7. Prior irraditation: prior irradiation for some pelvic cancers
Morphology:
1. urothelial tumors may be single or multiple.
2. About 90% of the tumors are papillary (non-invasive or invasive), whereas the
remaining 10% are flat indurated (non-invasive or invasive)
3. Most common location in the bladder is lateral walls, followed by posterior wall and
region of trigone.
4. The papillary tumors have a broad or narrow pedicle.
5. The non-papillary tumors are bulkier with ulcerated surface

Q-13
a) Define and classify Glomerulonephritis. 1+4
b) How will you diagnose Nephrotic syndrome in the lab? 5
ANS:
a) GLOMERULONEPHRITIS:
the term refers to several kidney diseases as is not strictly a single disease it may present
with hematuria and/or proteinuria or as a nephrotic syndrome a nephritic syndrome, or
chronic kidney disease.
Classification:
A. Primary glomerular diseases
1. Minimal-change disease
2. Focal segmental glomerulosclerosis
3. Membranous nephropathy

KMU Past Papers Solved SEQs


Kidney Male Female Genital System and Lower Urinary Tract 104

4. Acute postinfectious GN
5. Membranoproliferative GN
6. IgA nephropathy
B. Glomerulopathies secondary to systemic diseases
1. Lupus nephritis (systemic lupus erythematosus)
2. Diabetic nephropathy
3. Amyloidosis
4. GN secondary to multiple myeloma
5. Goodpasture syndrome
6. Microscopic polyangiitis
7. Wegener granulomatosis
8. Henoch-Schönlein purpura
9. Bacterial endocarditis–related GN
10. Thrombotic microangiopathy
C. Hereditary disorders
1. Alport syndrome
2. Fabry disease
3. Podocyte/slit-diaphragm protein mutations
b) LABORATORY DIAGNOSIS OF NEPHROTIC SYNDROME:
1. proteinuria >3.5 g/24 hours.
2. Hypoalbuminemia, with plasma albumin levels less than 3 g/dL
3. Generalized pitting edema and ascites due to hypoalbuminemia cause a decrease in
plasma oncotic pressure.
4. Hypertension in some types due to sodium retention
5. Hypercoagulable state due to loss of antithrombin III,
6. Hypercholesterolemia,
7. Hypogammaglobulinemia due to the loss of γ-globulins in the urine
8. Fatty casts with Maltese crosses and oval fat bodies,

Q-14
a) Enumerate the causes of acute renal failure. 4
b) Give a brief account of renal function tests. 6
ANS:
a) CAUSES OF ACUTE RENAL FAILURE:
1. Pre-renal causes Pre-renal diseases
i) renal arterial obstruction causing reduced perfusion of the kidneys.
ii) causes include inadequate cardiac output
iii) hypotension
iv) hypovolemia
2. Intra-renal causes Intra-renal disease
i) These include vascular disease of the arteries and arterioles within the kidney,
ii) diseases of glomeruli,
iii) acute tubular necrosis due to ischemia,
iv) nephrotoxin,
v) acute tubulointerstitial nephritis
vi) pyelonephritis.
3. Post-renal causes Post-renal disease

KMU Past Papers Solved SEQs


Kidney Male Female Genital System and Lower Urinary Tract 105

This may be caused by a mass within the lumen or from wall of the tract, or from
external compression anywhere along the lower urinary tract ureter, bladder neck or
urethra.
b) RENAL FUNCTION TESTS
Given somewhere else**

Q-15 A seven years old boy develops low grade fever and colored urine 2-3 weeks after
recovering from sore throat. He has developed moderate hypertension. On urine examination,
there is mild proteinuria, red cells casts.
a) What is the most likely diagnosis? 1
b) How the condition has developed? 6
c) What other conditions can produce such scenario? (Name three conditions) 3

ANS:
a) POST STREPTOCOCCAL INFECTION

b) PATHOGENESIS:
When there is initial infection localized to the pharynx or skin by certain “nephritogenic”
strains of β-hemolytic streptococci, there is glomerular deposition of immune complexes
resulting in proliferation of and damage to glomerular cells and infiltration of leukocytes,
especially neutrophils. tissue injury is primarily caused by complement activation by the
classical pathway
The relevant antigens probably are streptococcal proteins. Include streptococcal exotoxin
B and streptococcal GAPDH. Both activate the alternative complement pathway and have
affinity for glomerular proteins and plasmin.

c) OTHER CONDITIONS:
These include certain pneumococcal and staphylococcal infections as well as several
common viral diseases such as mumps, measles, chickenpox, and hepatitis B and C and
SLE.

Q-16
a) What is benign prostatic hyperplasia and give its morphology.
b) Give the pathogenesis of prostate adenocarcinoma?
ANS:
a) BENIGN PROSTATIC HYPERPLASIA:
BPH is characterized by proliferation of both stromal and epithelial elements, with
resultant enlargement of the gland and, in some cases, urinary obstruction.

Morphology:
Gross:
1. BPH always occurs in the inner, transitional zone of the prostate.
2. The affected prostate is enlarged, typically weighing between 60 and 100 g, and
contains many well-circumscribed nodules that bulge from the cut surface.

KMU Past Papers Solved SEQs


Kidney Male Female Genital System and Lower Urinary Tract 106

3. The nodules may appear solid or contain cystic spaces, the latter corresponding to
dilated glandular elements.
4. The urethra is usually compressed by the hyperplastic nodules, often to a narrow
slit.
5. In some cases, hyperplastic glandular and stromal elements lying just under the
epithelium of the proximal prostatic urethra may project into the bladder lumen as
a pedunculated mass, producing a ball-valve type of urethral obstruction.
Microscopically
1. the hyperplastic nodules are composed of variable proportions of proliferating
glandular elements and fibromuscular stroma.
2. The hyperplastic glands are lined by tall, columnar epithelial cells and a peripheral
layer of flattened basal cells
3. The glandular Lumina often contain inspissated, proteinaceous secretory material
known as corpora amylacea.
b) PATHOGENESIS OF PROSTATE ADENOCARCINOMA
1. Androgens:
2. Heredity also contributes, as there is an increased risk among first-degree relatives
of patients with prostate cancer.
3. Incidence of prostatic cancer is uncommon in Asians and highest among blacks
and is also high in Scandinavian countries.
4. Persons have MYC oncogene on chromosome 8q24 that appears to account for
some of the increased incidence of prostate cancer
5. Environment
6. somatic mutations are gene rearrangements that create fusion genes consisting of
the androgen-regulated promoter of the TMPRSS2 gene and the coding sequence
of ETS family transcription factors (the most common being ERG).
7. TMPRSS2-ETS fusion genes occur in approximately 40% to 50% of prostate
cancers; it is possible that unregulated increased expression of ETS transcription
factors interfere with prostatic epithelial cell differentiation.
8. Other mutations commonly lead to activation of the oncogenic PI3K/AKT signaling
pathway; of these, the most common are mutations that inactivate the tumor
suppressor gene PTEN, which acts as a brake on PI3K activity.

Q-17
a) classify tumors of ovary
b) enumerate surface epithelial tumors of ovary and note on cystic adenoma

ANS:
a) CLASSIFY OVARIAN TUMORS.
Given somewhere else**
b) SURFACE EPITHELIAL TUMORS OF OVARY:
1. Serous tumors
i) Serous cystadenoma
ii) Borderline serous tumor
iii) Serous cystadenocarcinoma
2. Mucinous tumors
i) Mucinous cystadenoma
ii) Borderline mucinous tumor

KMU Past Papers Solved SEQs


Kidney Male Female Genital System and Lower Urinary Tract 107

iii) Mucinous cystadenocarcinoma


3. Endometrioid tumors
4. Clear cell (mesonephroid) tumors
5. Brenner tumors
Serous cyst adenoma:
They are called serous tumors because of the presence of clear, watery, serous fluid in
these predominantly cystic tumors. About 60% of serous tumors are benign, Serous
tumors occur most commonly in 2nd to 5th decades of life. Histogenesis of the serous
tumors is by metaplasia from the surface (coelomic) epithelium or mesothelium which
differentiates along tubal-type of epithelium.
Morphologically it is a Clearly Benign tumors contain a single layer of tall columnar
epithelial cells often ciliated, that line the cyst or cysts and Papillary Projection the
epithelium does not invade fibro-vascular stromal stalk of papillae.

Q-18
a) Classify Testicular Tumors? 4
b) Briefly describe gross appearance of "classical" or "typical" seminoma. 3
c) Write three identification points of microscopic picture of seminoma. 3

ANS:
a) CLASSIFICATION OF TESTICULAR TUMORS.
Given somewhere else**
b) GROSS APPEARANCE OF CLASSICAL OR TYPICAL SEMINOMA
1. the involved testis is enlarged up to 10 times.
2. the tumor rarely invades the tunica.
3. Cut section of the affected testis shows homogeneous, grey-white lobulated
appearance.
4. Large tumors may contain foci of coagulation necrosis, usually without hemorrhage.

c) MICROSCOPIC PICTURE OF SEMINOMA.


1. Tumor cells
The seminoma cells generally lie in cords, sheets or columns forming lobules.
The tumor cells are uniform in size with clear cytoplasm and well-defined cell borders.
The cytoplasm contains glycogen that stains positively with PAS reaction.
The nuclei are centrally located, large, hyperchromatic and usually contain 1-2
prominent nucleoli.
Tumor giant cells may be present.
Mitotic figures are infrequent. However, anaplastic seminomas have increased mitotic
activity and have aggressive behavior.
2. Stroma
The stroma of seminoma is delicate fibrous tissue which divides the tumor into
lobules. The stroma shows a characteristic lymphocytic infiltration, indicative of
immunologic response of the host to the tumor.
The tumors show granulomatous reaction in the stroma.
Sometime syncytiotrophoblasts are present that are the source of the minimally
elevated serum HCG concentrations encountered in some males with pure seminoma.

KMU Past Papers Solved SEQs


Kidney Male Female Genital System and Lower Urinary Tract 108

Q-19
a) What is nephrotic syndrome?
b) Enumerate the causes of nephrotic syndrome?

ANS:
a) NEPHROTIC SYNDROME:
Nephrotic syndrome is a constellation of features in different diseases having varying
pathogenesis; it is characterized by findings of massive proteinuria, hypoalbuminemia,
edema, hyperlipidemia, lipiduria, and hypercoagulability.
b) CAUSES:
A. Primary glomerulonephritis
1. Minimal change disease (most common in children)
2. Membranous GN (most common in adults)
3. Membranoproliferative GN
4. Focal segmental glomerulosclerosis
5. Focal and diff use proliferative GN
6. IgA nephropathy
B. Systemic diseases
1. Diabetes mellitus
2. Amyloidosis
3. SLE
C. Systemic infections
1. Viral infections (HBV, HCV, HIV)
2. Bacterial infections (bacterial endocarditis, syphilis, leprosy)
3. Protozoa and parasites (P. falciparum malaria, filariasis)
D. Hypersensitivity reactions
1. D rugs (heavy metal compounds like gold and mercury, other drugs like
penicillamine, trimethadione and tolbutamide, heroin addiction)
2. Bee stings, snake bite, poison
E. Malignancy
1. Carcinomas
2. Myeloma
3. Hodgkin’s disease
F. PREGNANCY Toxemia of pregnancy
G. Circulatory disturbances
1. Renal vein thrombosis
2. Constrictive pericarditis
H. Hereditary diseases
1. Alport’s disease
2. Fabry’s disease
3. Nail-patella syndrome

Q-20
a) List the most important germ cell tumors of the ovary.
b) What are Immature Teratomas?
c) Briefly describe Dermoid Cyst or Mature Teratoma.

Ans:
a) IMPORTANT GERM CELL TUMORS OF THE OVARY:

KMU Past Papers Solved SEQs


Kidney Male Female Genital System and Lower Urinary Tract 109

1. Teratoma
2. Dysgerminoma
3. Endodermal sinus tumor
4. Choriocarcinoma
b) IMMATURE MALIGNANT TERATOMAS:
gross appearance:
Found early in life, the mean age at clinical detection being 18 years.
They differ strikingly from benign mature teratoma insofar as they often are bulky,
predominantly solid on cut section, and punctuated by areas of necrosis; uncommonly,
cystic foci are present that contain sebaceous secretion, hair, and other features similar to
those of mature teratoma.
MICROSCOPIC EXAMINATION:
The distinguishing feature is presence of immature elements or minimally differentiated
cartilage, bone, muscle, nerve, or other tissues.
Particularly ominous are foci of neuroepithelial differentiation, in view of the propensity
of such foci to be aggressive and metastasize widely.
Immature teratomas are both graded and staged in an effort to predict their behavior.
Grade I, stage I tumors often can be cured with appropriate therapy, whereas those of
higher grade and stage are associated with a more guarded outlook.

c) BENIGN (MATURE) CYSTIC TERATOMAS:


1. Almost all benign (mature) cystic teratoma are marked by the presence of mature
tissues derived from all three germ cell layers
i) Ectoderm
ii) Endoderm
iii) mesoderm.
2. Usually these tumors contain cysts lined by epidermis replete with adnexal
appendages hence the common designation dermoid cysts.
3. Most are discovered in young women as ovarian masses or are found incidentally
on abdominal radiographs or scans because they contain foci of calcification
produced by tooth-like structures contained within the tumor.
About 90% are unilateral, with the right side more commonly affected.
4. For unknown reasons, these neoplasms sometimes produce infertility and are
prone to undergo torsion (in 10% to 15% of cases), which constitutes an acute
surgical emergency.
5. A rare, but fascinating, paraneoplastic complication is limbic encephalitis, which
may develop in women with teratoma containing mature neural tissue and often
remits with tumor resection.
6. about 1% of cases, malignant transformation, usually to a squamous cell
carcinoma, is seen.
Morphology:
1. They often are filled with sebaceous secretion and matted hair that, when
removed, reveal a hair-bearing epidermal lining.
2. Sometimes there is a nodular projection from which teeth protrude.
3. Occasionally, foci of bone and cartilage, nests of bronchial or gastrointestinal
epithelium, and other tissues also are present.

KMU Past Papers Solved SEQs


Kidney Male Female Genital System and Lower Urinary Tract 110

Q-21
a) Classify renal tumors 4
b) Give the gross and microscopic features of clear cell renal cell carcinoma 6

ANS:
a) RENAL TUMORS CLASSIFICATION
Given somewhere else**
b) MORPHOLOGY
Given somewhere else**

Q-22
a) Classify rapidly progressive Glomerulonephritis 2
b) Describe the pathogenesis of anti GBM antibody-induced disease. 3
c) Describe morphological features of progressive Glomerular nephritis 5
ANS:
a) CLASSIFICATION:
Rapidly progressive glomerulonephritis
1) Anti-Glomerular Basement Membrane Antibody–Mediated Crescentic
Glomerulonephritis (12%) (in systemic diseases)
2) Post-infectious RPGN (immune-complex type) (44%)
3) Pauci-immune crescentic Glomerulonephritis (44%)
b) PATHOGENESIS:
In this type of injury, antibodies are directed against fixed antigens in the GBM A
conformational change in the α3 chain of the type IV collagen of the GBM appears to be
key in inciting autoimmunity.
c) MORPHOLOGY
A. GROSSLY The kidneys are enlarged and pale, often with petechial hemorrhages on
the cortical surfaces.
B. Light microscopy
1. glomeruli show segmental necrosis and GBM breaks,
2. Crescents formation result from proliferation of the parietal epithelial cells in
response to plasma proteins and fibrin in Bowman’s space and by migration of
monocytes/macrophages into Bowman’s space
3. Fibrin deposition is present alongside crescents.
4. Fibrin thrombi are frequently present in the glomerular tufts.
5. Besides the crescents, glomerular tufts may show increased cellularity as a result
of proliferation of endothelial and mesangial cells and leucocytic infiltration.
6. The uninvolved portion of the glomerulus shows no proliferation. In contrast to
immune complex mediated GN there is proliferation.
C. IMMUNOFLUORESCENCE MICROSCOPY
1. linear pattern of RPGN in Goodpasture’s syndrome (type I RPGN), show strong
staining of linear IgG and C3 deposits along the GBM.
2. Granular pattern of post-infectious RPGN (type II RPGN) consisting of IgG and C3
along the capillary wall.

KMU Past Papers Solved SEQs


Kidney Male Female Genital System and Lower Urinary Tract 111

3. Scanty or no deposits of immunoglobulin and C3 in pauci-immune GN (type III


RPGN).
D. ELECTRON MICROSCOPY
1. may show distinct ruptures in the GBM.
2. Post-infectious RPGN cases show electron-dense subepithelial granular deposits
3. RPGN in Good pasture’s syndrome show characteristic linear deposits along the

Q-23
a) Give a pathologic classification of testicular tumors 4
b) Name the two histological seminoma 4
c) Which germ cell tumor of testis is common in infancy and child hood? 2

GBM
ANS:
a) CLASSIFICATION:
Given somewhere else**
b) SEMINOMA ARE:
1. Classic seminoma
2. Spermetocytic seminoma
c) TUMORS COMMON IN CHILDHOOD AND INFANCY:
Yolk sac tumor and seminoma.

Q-24
(a) Enumerate the primary Glomerular diseases 1.5
(b) What is Nephrotic syndrome? 2
(c) Give the microscopic appearance of membrane glomerulonephritis under the following
headings
I. Light microscopy 4
II. Electron microscopy 1
III. Immunofluorescence microscopy 0.5

ANS:
a) PRIMARY GLOMERULAR DISEASES:
1) Minimal change disease
2) Focal segmental glomerulosclerosis
3) Membranous nephropathy
4) Acute post infectious GN
5) Membranoproliferative GN
6) IgA nephropathy
b) NEPHROTIC SYNDROME:
Nephrotic syndrome is a constellation of features in different diseases having varying
pathogenesis; it is characterized by findings of massive proteinuria, hypoalbuminemia,
edema, hyperlipidemia, lipiduria, and hypercoagulability.
c) MORPHOLOGY:
A. LIGHT MICROSCOPY
1. In early stages of disease, the glomeruli may appear normal
2. In later stages it shows diffuse thickening of the capillary wall,
3. thickened basement membrane, producing ‘duplication’ of GBM
4. There is no cellular proliferation in the glomerular tufts.

KMU Past Papers Solved SEQs


Kidney Male Female Genital System and Lower Urinary Tract 112

B. ELECTRON MICROSCOPY reveals


1. Sub-epithelial deposits present in spike and dome pattern.
2. As the disease progresses, these spikes close over the deposits,
incorporating them into the GBM.
3. The basement membrane material protrudes between deposits as ‘spikes’
4. Podocytes show effacement of foot processes.
5. Later in the disease, the incorporated deposits may be broken down and
eventually disappear, leaving cavities within the GBM.
6. Continued deposition of basement membrane matrix leads to progressive
thickening of basement membranes and the glomeruli can become
sclerosed.
C. IMMUNOFLUORESCENCE MICROSCOPY shows typical granular deposits of
immunoglobulins and complement along the GBM

Q-25
a) Define choriocarcinoma 1
b) Write down the gross and histologic morphology of choriocarcinoma 9

ANS:
a) CHORIOCARCINOMA:
Choriocarcinoma, a very aggressive malignant tumor, arises either from gestational
chorionic epithelium or, less frequently, from totipotential cells within the gonads (as a
germ cell tumor).

Metastasizes early and widely.


Primary focus may degenerate, leaving only metastases
In contrast with gestational tumors ovarian primaries are resistant to chemotherapy

b) MORPHOLOGY
Grossly:
1. Choriocarcinomas usually appear as hemorrhagic, necrotic uterine masses.
2. Sometimes the necrosis is so extensive that little viable tumor remains.
3. Indeed, the primary lesion may “self-destruct,” and only the metastases tell the
story.
Microscopically:
1. Very early, the tumor insinuates itself into the myometrium and into vessels.
2. In contrast with hydatidiform moles and invasive moles, chorionic villi are not
formed

KMU Past Papers Solved SEQs


Kidney Male Female Genital System and Lower Urinary Tract 113

3. the tumor is composed of anaplastic cuboidal cytotrophoblasts and


syncytiotrophoblasts
ANS:

Q-26 A 4-year-old girl presented with generalized edema and lab result show proteinuria in
excess of 3.5 gm/day with hypoalbumenimia. Patient improved dramatically with
corticosteroid administration
a) What is the most likely diagnosis? 1
b) Write two manifestations of Nephrotic and Nephritic syndrome. 4
c) Give light microscopic, ultra-structural and immune fluorescence findings. 5

a) MINIMAL CHANGE DISEASE


b) MENIFESTATIONS
A. Nephrotic syndrome:
1. proteinuria (excretion of greater than 3.5 g of protein/day in adults),
2. hypoalbuminemia,
3. severe edema,
4. hyperlipidemia,
5. lipiduria (lipid in the urine)
6. hypercoagulability.
B. Nephritic syndrome:
1. grossly visible hematuria (red blood cells and red cell casts in urine),
2. proteinuria of mild to moderate degree,
3. azotemia,
4. edema,
5. hypertension
6. oliguria.

c) MORPHOLOGY
A. LIGHT MICROSCOPE,
1. silver methenamine stained glomerulus appear normal, (minimal change disease
or nil lesion).
2. slight increase in the mesangial matrix
3. The cells of the proximal convoluted tubules often are heavily laden with protein
droplets and lipids, but this feature is secondary to tubular reabsorption of the
lipoproteins passing through the diseased glomeruli (lipoid nephrosis).
B. ELECTRON MICROSCOPE,
1. The GBM appears normal.
2. uniform and diffuse effacement (flattening) of the foot processes of the podocytes
3. The cytoplasm of the podocytes appears flattened over the external aspect of the
GBM,
4. There is also epithelial cell vacuolization, microvillus formation, and occasional
focal detachments. (foot process disease or podocytopathy)
C. IMMUNOFLUORESCENCE MICROSCOPY shows no deposits, complement or
immunoglobulins (nil deposits disease)

KMU Past Papers Solved SEQs


Kidney Male Female Genital System and Lower Urinary Tract 114

Q-27 A 30-year-old female presented with pelvic pain for more than eight (8) months.
Sonography reveals left ovarian mass of 9 cm in diameter. Cystectomy specimen reveals a
unilocular mass, filled by straw colored fluid. Histological evaluation show the cyst is lined by
columnar to cuboidal epithelium showing stratification at places. There is no nuclear atypia or
stromal invasion.
a) What is the most likely diagnosis? 2
b) How would you histologically differentiate between other types of the tumor Diagnosed? 5
c) Name other Cystic Ovarian Tumors. 3

ANS:
a) SURFACE EPITHELIAL TUMOR (TUMORS OF LOW MALIGNANT POTENTIAL OR
BORDER LINE TUMOR)
b) DIFFERENTIATION BETWEEN OTHER TYPES:
1. SEROUS CYSTADENOMA Clearly Benign tumors contain a single layer of tall columnar
epithelial often are ciliated, that line the cyst or cysts and Papillary Projection. but
does not invade fibro-vascular stromal stalk of papillae.
2. BORDER LINE TUMORS Tumors of low malignant potential Between clearly benign
and frank carcinoma forms, having stratification (2-3 layers) of the epithelial cells,
less cytologic atypia maintained basal polarity of nuclei, moderate nuclear
abnormalities, and some mitotic activity. typically, little or no stromal invasion.
3. SEROUS CYSTADENOCARCINOMA Clearly malignant carcinoma, there is anaplasia of
the lining cells appears, multilayering, loss of basal polarity and nests or
undifferentiated sheets of malignant cells does invade the fibrovascular stromal stalk
of papillae. Psammoma bodies are common in the tips of papillae.
c) OTHER CYSTIC OVARIAN TUMORS.
1. Serous tumor
2. Mucinous tumor
3. Endometrioid tumor
4. Brenner tumor
5. Benign (Mature) Cystic Teratomas
6. Granulosa-theca cell

Q-28 An orchidectomv specimen of a 45 years old male was received in pathology lab. On cut
section there was a circumscribed grayish white fleshy homogenous mass devoid of any
hemorrhage or necrosis. Microscopic examination of the representative sections showed sheets
of large polygonal cells having Clear Cytoplasm. Nuclei were large having centrally placed 1 to
2 distinct nucleoli. Sparse lymphocytic infiltration was also present.
a) What is the most probable diagnosis? 1
b) Why neoplastic cells in this case have clear cytoplasm? Name special stains
for its demonstration. 2+1
c) Name two immunohistochemical markers for identifying these tumor cells. 2
d) Name two sex cord stromal tumors of testis. 2
e) Name the substances released from germ cell tumors. 2

ANS:
a) CLASSICAL OR TYPICAL SEMINOMA

KMU Past Papers Solved SEQs


Kidney Male Female Genital System and Lower Urinary Tract 115

b) The cytoplasm contains glycogen that’s why and the special stains used is periodic Acid-
Schiff (PAS)
c) human chorionic gonadotropin (HCG) and alpha-foeto protein (AFP).
d) SEX CORD STROMAL TUMOR:
1. Leydig cell tumor
2. Sertoli cell tumor (Androblastoma)
e) SUBSTANCES RELEASED FROM GERM CELL TUMORS
1. human chorionic gonadotropin (HCG)
2. alpha-foeto protein (AFP).
3. carcinoembryonic antigen (CEA),
4. human placental lactogen (HPL),
5. placental alkaline phosphatase,
6. testosterone,
7. estrogen
8. luteinizing hormone

Q-29
a) Define endometriosis and give its six common sites of occurrences. 1+3
b) Give brief account of major theories of development of endometriosis. 2
c) Enlist eight risk factors for cervical squamous cell carcinoma 4

ANS:
a) ENDOMETRIOSIS:
Endometriosis is defined by the presence of endometrial glands and stroma in a location
outside the endomyometrium.
Common sites (in descending order of frequency):
A. Pelvic structures
1. ovaries,
2. uterine ligaments
3. rectovaginal septum
4. pouch of Douglas
5. fallopian tubes
B. Distant areas
1. laparotomy scars
2. umbilicus,
3. vagina,
4. vulva,
5. append ix
6. hernia sacs.
C. Distant sites
1. lymph nodes,
2. lungs,
3. heart,
4. skeletal muscle,
5. bones
b) THEORIES:
1. Transplantation or regurgitation theory is based on the assumption that ectopic
endometrial tissue is trans planted from the uterus to an abnormal location by way of
fallopian tubes due to regurgitation of menstrual blood.

KMU Past Papers Solved SEQs


Kidney Male Female Genital System and Lower Urinary Tract 116

2. Metaplastic theory suggests that ectopic endometrium develops in situ from local
tissues by metaplasia of the coelomic epithelium (from which endometrium
originates)
3. Vascular or lymphatic dissemination explains the development of endometrial tissue
at extra-pelvic sites by these routes.
c) RISK FACTORS FOR CERVICAL CANCER:
Given somewhere else**

Q-30
a) enumerate causes of hematuria
b) what is ethipathogenesis of membranoproliferative glomerulonephritis
c) what is morphology of membraneproliferative glomerulonephritis

ANS:
a) CAUSES OF HEMATURIA:
A. Renal causes:
1. IgA nephropathy
2. Alport syndrome
3. Post streptococcal glumeronephrits
4. Hereditary Nephritis
5. Rapidly progressive glomerulonephritis
6. Polycystic kidney disease
7. Renal cell carcinoma
B. Post renal causes:
1. Urinary tract infections
2. Nephrolithiasis
3. BPH
4. Hemorrhagic cystitis
C. Other causes:
1. Sickle cell disease
2. Paroxysmal nocturnal hemoglobinuria

b) ETIOPATHOGENESIS OF MEMBRANOPROLIFERATIVE GLOMERULONEPHRITIS


Etiology of MPGN is unknown though in some cases there is evidence of preceding
streptococcal infection.
A. TYPE I OR CLASSIC FORM
1. type I MPGN may be caused by circulating immune complexes
2. associated with systemic immunecomplex diseases (e.g. SLE, mixed cryoglob
ulinaemia, Sjögren’s syndr ome),
3. chronic infections (e.g. bacterial endocarditis, HIV, hepatitis B and C)
4. malignancies (e.g. lymphomas and Leukemia).
B. TYPE II OR DENSE DEPOSIT
1. excessive complement activation. (alternate pathway disease)
2. Type II MPGN is an autoimmune disease in which patients have IgG
autoantibody termed C3 nephritic factor, lead to uncontrolled cleavage of C3 and
activation of the alternative complement pathway.
3. Mutations in the gene encoding the complement regulatory protein factor H or
autoantibodies to factor H have been described in some patients.
4. Type II cases have an association with partial lipodystrophy,

KMU Past Papers Solved SEQs


Kidney Male Female Genital System and Lower Urinary Tract 117

C. TYPE III is rare


1. associated with systemic diseases or drugs.
c) MORPHOLOGY:
A. Grossly, the kidneys are usually pale in appearance and firm in consistency.
B. BY LIGHT MICROSCOPY,
i) Glomeruli Glomeruli
2. They are enlarged with accentuated lobular pattern.
3. mesangial cellular proliferation and increase in mesangial matrix.
4. The GBM is considerably thickened with silver stains it shows two basement
membranes with a clear zone between them.
5. This is commonly referred to as ‘double contour’, splitting, or ‘tram track’
appearance.
ii) Tubules Tubular cells
1. show vacuolation and hyaline droplets.
C. By electron microscopy and immunofluorescence microscopy,
TYPE I:
2. It shows electron-dense deposits in sub-endothelial location conforming to
immune-complex character of the disease
3. These deposits reveal positive fluorescence for C3 and slightly fainter staining
for IgG.
TYPE II:
1. dense amorphous deposits within the lamina densa of the GBM and in the
mesangium.
2. Immunofluorescence studies reveal the universal presence of C3 and
properdin in the deposits but the immunoglobulins are usually absent.
TYPE III:
1. electron-dense deposits within the GBM as well as in sub-endothelial and sub-
epithelial regions of the GBM.
2. Immunofluorescence studies show the presence of C3, IgG and IgM.

Q-31
a) Define proteinuria and describe its laboratory diagnosis?
b) Enumerate renal function tests

ANS:
a) PROTEINURIA:
means the presence of an excess of serum proteins in the urine.
b) LAB DIAGNOSIS:
1. Dipstick test
2. Urine protein electrophoresis
3. Liquid crystal method
4. Protein creatinine ratio

KMU Past Papers Solved SEQs


Kidney Male Female Genital System and Lower Urinary Tract 118

c) GIVEN SOMEWHERE ELSE**

Q-32 A 65-year-old male present with pain in the flank and palpable mass eight weeks duration
laboratory investigation reveal microscopic hematuria and serum calcium level of 12.5 mg/dl
a) Name of tumor mentioned in the scenario
b) Name different types of this tumor
c) Which type is the most common and name the gene undergoing mutation in this type?
d) How will you explain the raised serum calcium level?
e) What is its characteristic tendency during spread?
f) Which change is a very obvious frequently occurring microscopic feature of it?

ANS:
a) RENAL CELL CARCINOMA
b) TYPES:
1. Clear Cell Carcinomas (most common 65%)
2. Papillary Renal Cell Carcinomas (10% to 15%)
3. Chromophobe Renal Carcinomas (5%)
4. granular cell
5. sarcomatoid and
6. collecting duct type
c) COMMON TYPE AND MUTATION:
clear cell carcinoma having germline mutations of tumor suppressor VHL gene (a tumor
suppressor gene) located on chromosome 3p (band 3p25)
d) HYPERCALCEMIA:
some tumor produce hormone like substances e.g para thyroid hormone like protein
which cause hypercalcemia
e) TUMOR SPREAD
1. As the tumor enlarges, it may fungate through the walls of the collecting system,
2. extending through the calyces and pelvis as far as the ureter.
3. the tumor invades the renal vein and grows as a solid column within this vessel,
sometime extend as far as the inferior vena cava and even into the right side of the
heart.
4. Occasionally, direct invasion into the perinephric fat and adrenal gland may be seen.
f) MICROSCOPIC FEATURE:
The classic vacuolated (lipid-laden), or clear cells are demarcated only by their cell
membranes.

KMU Past Papers Solved SEQs


Endocrine System, Breast, Bones, Joints and CNS 119

Q-1
a) Classify breast tumor.
b) Give risk factors for carcinoma of Breast.
c) Give an account of clinical staging of breast carcinoma

ANS:
a) CLASSIFICATION OF BREAST TUMORS
1. Benign tumors
i) fibroadenoma,
ii) phyllodes Tumor (cystosarcoma phyllodes) and
iii) intraductal papilloma.
2. Malignant tumors
A) Non-invasive (in situ) carcinoma
1. Intraductal carcinoma
2. Lobular carcinoma in situ
B) Invasive carcinoma
1. Infiltrating (invasive) duct carcinoma-NOS (not otherwise specified) (80%)
2. Infiltrating (invasive) lobular carcinoma (10%)
3. Tubular (cribriform) carcinoma (6%)
4. Medullary carcinoma (2%)
5. Colloid (mucinous) carcinoma (2%)
6. Other types: Papillary carcinoma, adenoid cystic (invasive cribriform)
carcinoma, secretory (juvenile) carcinoma, inflammatory carcinoma,
metaplastic carcinoma.
C) Paget’s disease of the nipple
b) RISK FACTORS OF BREAST CARCINOMA
1. Geographic: The incidence of breast cancer is about 4-6 times higher in developed and
low in developing countries
2. Age: especially after menopause, peaking at roughly 80 years of age
3. Family history First-degree relatives (mother, sister, daughter) of women with breast
cancer, especially Premenopausal and bilateral cancer, have higher risk of
development of breast cancer.
4. Genetics:
Autosomal dominant BRCA1 and BRCA2 association
Li fraumeni multi-cancer syndrome; P53 suppressor gene mutation
RAS oncogene and RB1 gene mutation
5. Menstrual and obstetric history: early menarche, nulliparity, late age of first childbirth
and delayed menopause.
6. Preexisting lesions: Lobular carcinoma in situ and Fibrocystic change with atypical
epithelial hyperplasia
7. Endometrial cancer
8. Hormones: Exogenous estrogens in HRT Oral contraceptives
9. Diet: Consumption of large amounts of animal fats, high calorie foods.
10. Obesity: excessive estrogen produced by adipose tissue.
11. Cigarette smoking.
12. Alcohol consumption
13. Breast augmentation surgery.
14. High breast density.

KMU Past Papers Solved SEQs


Endocrine System, Breast, Bones, Joints and CNS 120

15. Exposure to ionizing radiation during breast development


c) STAGING OF BREAST CARCINOMA:
1. Stage TIS: In situ carcinoma (in situ lobular, intraductal, Paget’s disease of the nipple
without palpable lump)
2. Stage I: Tumor 2 cm or less in diameter No nodal spread
3. Stage II: Tumor > 2 cm and < 5 cm in diameter Regional lymph nodes involved
4. Stage III A: Tumor > 5 cm in diameter Regional lymph nodes involved on same side
5. Stage III B: Tumor > 5 cm in diameter Supraclavicular and infraclavicular lymph
nodes involved
6. Stage IV: Tumor of any size with or without regional spread but with distant
metastasis

Q-2A 25-year-old women presented with the left breast mass which was non-tender, firm and
well circumscribed.
a) Enumerate the likely causes for lump in this case.
b) Discuss the relevant investigation to reach the diagnosis.
c) Write the microscopic appearance of fibroadenoma.

ANS:
a) CAUSES OF FIBRO-ADENOMA
1. increase in estrogen.
2. menstrual cycle
3. during pregnancy
b) INVESTIGATIONS
1. Mammography (bilateral)
2. FNA of breast lump
3. Core needle biopsy of the lesion.
4. CT scan of the abdomen and chest.
5. Bone scan.
c) MICROSCOPIC FEATURE OF FIBROADENOMA:
The arrangements between fibrous overgrowth and ducts may produce two types of
patterns
1. Intracanalicular pattern:
stroma compresses the ducts so that they are reduced to slit-like clefts lined by ductal
epithelium
may appear as cords of epithelial elements surrounding masses of fibrous stroma.
2. Pericanalicular pattern:
encircling masses of fibrous stroma around the patent or dilated ducts.
The fibrous stroma may be quite cellular, areas of hyalinized collagen or loose and
myxomatous.
As in normal breasts these glandular spaces are lined by luminal and myoepithelial
cells with well-defined intact basement membrane
A. Variants:
1. tubular adenoma: predominantly composed of closely packed ductular or acinar
proliferation with scanty fibrous tissue element
2. lactating adenoma: an adenoma composed of acini with secretory activity, seen
mostly during pregnancy

KMU Past Papers Solved SEQs


Endocrine System, Breast, Bones, Joints and CNS 121

3. Juvenile fibroadenoma: larger and rapidly growing mass seen in adolescent girls.
Q-3
a) Enlist the benign tumors of Breast
b) Describe the gross and microscopic features of Fibro-adenoma?
c) Describe the etiology of carcinoma breast.

ANS:
a) BENIGN TUMORS
1. fibroadenoma,
2. phyllodes Tumor (cystosarcoma phyllodes) and
3. intraductal papilloma.
b) MORPHOLOGY OF FIBROADENOMA:
A. Grossly
1. solitary, firm in consistency well encapsulated, spherical or discoid mass.
2. The cut sur face is firm, grey white or tan-white color, slightly myxoid and may
show yellow-pink specks or slit like spaces formed by compressed ducts.
3. Occasionally, multiple fibroadenomas may form part of fibrocystic disease and is
termed fibroadenomatosis.
4. Less commonly, a fibroadenoma may be fairly large in size, up to 15 cm in
diameter, and is called giant fibroadenoma
B. Histology:
The arrangements between fibrous overgrowth and ducts may produce two types of
patterns
1. Intracanalicular pattern:
stroma compresses the ducts so that they are reduced to slit-like clefts lined by
ductal epithelium
may appear as cords of epithelial elements surrounding masses of fibrous stroma.
2. Pericanalicular pattern:
encircling masses of fibrous stroma around the patent or dilated ducts.
The fibrous stroma may be quite cellular, areas of hyalinized collagen or loose and
myxomatous.
As in normal breasts these glandular spaces are lined by luminal and
myoepithelial cells with well-defined intact basement membrane
 Variants:
1. tubular adenoma: predominantly composed of closely packed ductular or
acinar proliferation with scanty fibrous tissue element
2. lactating adenoma: an adenoma composed of acini with secretory activity, seen
mostly during pregnancy
3. Juvenile fibroadenoma: larger and rapidly growing mass seen in adolescent
girls.
c) ETIOLOGY OF BREAST CARCINOMA:
A) Hormonal factors: excess endogenous estrogen or exogenously administered
estrogen.
1. Women with prolonged reproductive life, early menarche late menopause
2. Higher risk in unmarried and nulliparous women
3. Women with first childbirth at a late age (over 30 years)
4. Functioning ovarian tumors e.g. granulosa cell tumor which elevated estrogen
5. Hormone replacement therapy (HRT) administered to post-menopausal women.
6. Long term use of oral contraceptives containing only estrogen

KMU Past Papers Solved SEQs


Endocrine System, Breast, Bones, Joints and CNS 122

7. Men who have been treated with estrogen for prostatic cancer have risk
Normal breast epithelium possesses estrogen and progesterone receptors. The breast
cancer cells secrete many growth factors which are estrogen dependent. In this way,
the interplay of high circulating levels of estrogen, estrogen receptors and growth
factors bring about progression of breast cancer.
B) Genetic factors: These mutations include the following,
1. Gene mutation in sporadic cases:
i) Overexpression of the HER2/NEU proto-oncogene, a member of the
EGFR family
ii) Amplification of RAS and MYC genes.
iii) Mutations in tumor suppressor genes RB and TP53
Gene expression profiling can separate breast cancer into four molecular
subtypes:
(1) luminal A (estrogen receptor–positive, HER2/NEU-negative);
(2) luminal B (estrogen receptor–positive, HER2/NEU overexpressing);
(3) HER2/ NEU positive (HER2/NEU over expressing, estrogen receptor–negative)
(4) basal-like (estrogen receptor–negative and HER2/NEU-negative).
2. Women with inherited mutation they,
1) are more likely to have bilateral cancer,
2) have other familial forms of cancer e.g ovarian cancer
3) have a positive family history
4) to develop breast cancer before menopause,
Gene involved is:
i) breast cancer, BRCA 1 gene:
located on chromosome 17, a DNA repair gene,
implicated in both breast and ovarian cancer in inherited cases.
The protein product of BRCA gene is a cell cycle regulated protein and it
can be detected by immunohistochemistry.
Men who have mutated BRCA1 have increased risk of developing cancer of
the prostate but NOT of male breast.
ii) BRCA 2 gene:
located on chromosome 13, another DNA repair gene,
higher incidence of inherited cancer of the breast and ovary in females, and
prostate in men.
In BRCA1 as well as BRCA2, both copies of the genes (homozygous state)
must be inactivated for development of breast cancer.
first genetic lesion caused by a germline mutation and the second by a
subsequent somatic mutation
3. Other genes
i) Mutation in p53 tumor suppressor gene:
on chromosome 17, mutation is seen in Li-Fraumeni syndrome having
multiple cancers including breast cancer in young women.
ii) ataxia telangiectasia (AT) gene carriers
iii) Cowden syndrome, caused by germline mutations in PTEN (phosphate and
tensin) tumor suppressor gene.
C) Environmental factors
1. Ionizing radiations in women before the age of 30

KMU Past Papers Solved SEQs


Endocrine System, Breast, Bones, Joints and CNS 123

Q-4A lady of 22 years presents with discreet solitary freely movable nodule, 4cms in diameter,
histopathology of nodule reveal fibroadenoma.
a) Write its gross and microscopic morphology
b) Write variables which influence prognosis of Breast carcinoma
ANS:
a) MORPHOLOGY:
Given somewhere else*
b) VARIABLES INFLUENCE PROGNOSIS OF BREAST CARCINOMA:
1. Tumor invasion and size.
2. Extent of lymph node involvement.
3. Distant metastases.
4. Histologic grade: grading system evaluates tubule formation, nuclear grade, and
mitotic rate.
5. The histologic type of carcinoma: medullary, and mucinous are associated with a
somewhat better prognosis than ductal carcinomas. inflammatory carcinoma, which
has a poor prognosis.
6. The presence or absence of estrogen or progesterone receptors:
The presence of hormone receptors confers a slightly better prognosis.
7. Overexpression of HER2/NEU: associated with a poorer prognosis
Protein level measure through immunohistochemistry or fluorescence in situ
hybridization for assessing the gene copy number.
Q-5
a) Classify tumors of the bone.
b) Give morphology of Osteogenic Sarcoma.
c) How will you diagnose the bone tumor?
ANS:
a) CLASSIFICATION OF BONE TUMOR
A. Bone-Forming
1. Benign
i) Osteoma
ii) Osteoid osteoma
iii) Osteoblastoma
2. Malignant
i) Primary osteosarcoma
ii) Secondary osteosarcoma
B. Cartilaginous
1. Benign
i) Osteochondroma
ii) Enchondroma
iii) Chondroblastoma
2. Malignant
i) Chondrosarcoma
C. Hematopoietic (marrow tumors)
1. Multiple myeloma
D. Miscellaneous
1. Giant cell tumor

KMU Past Papers Solved SEQs


Endocrine System, Breast, Bones, Joints and CNS 124

2. Ewing sarcoma
b) MORPHOLOGY OF OSTEOGENIC SARCOMA
C. Gross:
1. osteosarcomas are gritty-appearing, gray-white tumors,
2. Codman’s triangle may be obvious on macroscopic examination
D. Cut surface:
1. Cut surface of the tumor is grey white with areas of hemorrhages and necrotic
bone and cystic degeneration.
2. Tumors frequently destroy the surrounding cortices, producing soft tissue masses
3. Tumor spread extensively in the medullary canal, infiltrating and replacing the
marrow but only infrequently penetrating the epiphyseal plate or entering the
joint space.
E. Histology:
1. Tumor cells vary in size and shape and frequently have large hyperchromatic
nuclei, mitotic figures and bizarre tumor giant cells are present.
2. Osteogenesis is present (The anaplastic sarcoma cells produce mineralized or
unmineralized bone (osteoid)
3. When malignant cartilage is abundant, the tumor is called a chondroblastic
osteosarcoma.
F. Histochemically, these Tumor cells are positive for alkaline phosphatase.
G. Immunohistochemically, sarcoma cells of osteosarcoma express vimentin, osteocalcin,
osteonectin and type I collagen.
c) DIAGNOSIS
1. X ray  Codman’s triangle and sunburst pattern
2. MRI
3. Bone CT scan
4. Lung CT scan in case of lung metastasis
5. Needle biopsy

Q-6
a) Define osteoporosis.
b) Give a brief account of etiology and pathogenesis of osteoporosis.

ANS:
a) OSTEOPOROSIS:
Osteoporosis or osteopenia is a common clinical syndrome involving multiple bones in
which there is quantitative reduction of bone tissue mass but the bone tissue mass is
otherwise normal.
b) ETIOLOGY AND PATHOGENESIS:
1. AGING
 Decreased replicative activity of osteoprogenitor cells
 Decreased synthetic activity of osteoblasts and retained activity of osteoclasts
 Decreased biologic activity of matrix-bound growth factors
 Reduced physical activity
2. Hormonal influences
Decreased serum estrogen cause in
 Increased IL-1, IL-6, TNF levels
 Increased expression of RANK, RANKL

KMU Past Papers Solved SEQs


Endocrine System, Breast, Bones, Joints and CNS 125

 Suppress OPG production


 Increased osteoclast activity
3. Physical inactivity.
4. Genetic factors, altering vit-D calcium and PTH metabolism.
5. Calcium nutritional state
6. Secondary causes of osteoporosis.
 Endocrine causes
 Glucocorticoid excess (iatrogenic or endogenous)
 Hyperthyroidism
 Hypogonadism
 Nutritional causes
 Malabsorption
 Chronic liver disease
 Inflammatory disorders
 Rheumatoid arthritis
 Crohn’s disease
 Smoking
 Alcohol

Q-7
a) Define and classify Osteomyelitis
b) Enumerate the complications of osteomyelitis
c) Briefly give morphology of pyogenic osteomyelitis

ANS:
a) OSTEOMYELITIS:
Inflammation of bone and marrow,
Classification:
1. Pyogenic osteomyelitis
2. Tuberculous Osteomyelitis
b) COMPLICATION OF OSTEOMYELITIS:
1. Pathologic fracture,
2. Secondary amyloidosis,
3. Endocarditis,
4. Sepsis,
5. Development of squamous cell carcinoma if the infection creates a sinus tract,
6. Osteosarcoma.
c) MORPHOLOGY OF PYOGENIC OSTEOMYELITIS:
1. The infection begins in the metaphyseal end of the marrow cavity which is largely
occupied by pus. At this stage, microscopy reveals congestion, edema and an
exudate of neutrophils.
2. accumulation of pus and results in spread of infection along the marrow cavity,
into the endosteum, and into the haversian and Volkmann’s canal, causing
periosteitis.
3. The infection in sub-periosteal space form subperiosteal abscesses. It may
penetrate through the cortex creating draining skin sinus tracts.

KMU Past Papers Solved SEQs


Endocrine System, Breast, Bones, Joints and CNS 126

4. Sequestrum formation which is Combination of suppuration and impaired blood


supply to the cortical bone results in erosion, thinning and infarction necrosis of
the cortex.
5. involucrum formation which is formation of new bone beneath the periosteum
present over the infected bone. This forms an encasing sheath around the
necrosed bone
6. Involucrum has irregular surface and has perforations through which discharging
sinus tracts pass.
7. Brodie’s abscess when acute osteomyelitis is contained to a localized area and
walled off by fibrous tissue and granulation tissue

Q-8
a) Define osteomyelitis
b) Name the organism frequently causing acute pyogenic osteomyelitis
c) Name the organisms causing pyogenic osteomyelitis in neonates and in individuals with
sickle cell disease,
d) How the causative organism reaches the bone?
e) What do you mean by sequestrum and involucrum?
f) Name the possible complications of acute pyogenic osteomyelitis
g) Investigations

ANS:
a) OSTEOMYELITIS:
inflammation of bone and marrow
b) Staphylococcus aureus is the most frequent causative organism
c) Escherichia coli and group B streptococci are important causes of acute osteomyelitis in
neonates, and Salmonella is an especially common pathogen in persons with sickle cell
disease
d) ROUTE OF INFECTION:
1. hematogenous dissemination (most common);
2. extension from an infection in adjacent joint or soft tissue;
3. traumatic implantation after compound fractures or orthopedic procedures.
e) SEQUESTRUM AND INVOLUCRUM:
 Sequestrum:
the process in which there is acute inflammatory reaction, with consequent cell
death with Combination of suppuration and impaired blood supply to the cortical
bone results in erosion, thinning and infarction necrosis of the cortex.
 Involucrum:
formation of new reactive woven or lamellar bone beneath the periosteum present
over the infected bone. This forms an encasing sheath around the necrosed bone
f) GIVEN ABOVE**
g) INVESTIGATIONS
1. The erythrocyte sedimentation rate,
2. white blood count,
3. acute phase reactants such as C-reactive protein are usually elevated.
4. radiographs

KMU Past Papers Solved SEQs


Endocrine System, Breast, Bones, Joints and CNS 127

5. magnetic resonance imaging (MRI)

Q-9
a) What is Arthritis?
b) Define osteoarthritis just name its types.
c) Tabulate the differences between Rheumatoid Arthritis and Osteoarthritis
d) Describe briefly laboratory diagnosis of Rheumatoid arthritis

ANS:
a) ARTHRITIS:
inflammation of joints
b) OSTEOARTHRITIS:
osteoarthritis is primarily a degenerative disorder of articular cartilage which cause of
physical disability.
Primary osteoarthritis
Secondary osteoarthritis
c) DIFFERENCES BETWEEN RHEUMATOID ARTHRITIS AND OSTEOARTHRITIS

Features Rheumatoid arthritis Osteoarthritis


Age Any age Old age
Gender Predominantly Female Both
Nature Autoimmune systemic Non inflammatory
chronic inflammatory degenerative disease
disease
Pathogenesis CD4+ T cell derived Increase degradation
Cytokines and Anti CCP and decrease synthesis
antibodies mediated of matrix due to genetic
inflammation. factors or increase in
age
Fibrous ankyloses Present due to fibrosis and Absent
Bony ankyloses ossification
Chronic papillary Present Absent
synovitis
Periarticular edema and
Morphology

inflammation
pannus formation Marked minimal
Sub-chondral cysts Absent Present
Osteophytes
Subchondral sclerosis
Joint mice
Subcutaneous nodules Present Absent
on exterior surface of
forearms.
joint involved Small joints ie Large joints ie
Clinical features

Proximal interphalangeal Hip, knee, lumber and


Metacarpophalangeal cervical spine, joint of
Upper cervical spine fingers.
Pattern Symmetrical, multiple Nonsymmetrical single
Constitutional sign and Present, fever malaise Absent
symptoms

KMU Past Papers Solved SEQs


Endocrine System, Breast, Bones, Joints and CNS 128

Pain Decrease with movement Increase with


movement
Signs Ulnar deviation of fingers Heberden and
and redial deviation of Bouchard nodes
wrest, swan neck deformity,
boutonniere deformity
Risk of developing Present Absent
necrotizing vasculitis
Ocular involvement Present Absent
Radiologic finding Decrease joint space and Normal joint space
fusion, joint effusion Osteophyte,
Juxtra-articular osteopenia subchondral cyst
Synovial fluid Sterile turbid decrease Sterile, no neutrophil
Diagnosis

examination viscosity inclusion bearing bearing inclusions


neutrophils. Poor mucin
clot formation
Serum antibodies Anti CCP and Rheumatoid Not present
factor
Antinuclear antibodies

d) LAB DIAGNOSIS:
1. Positive serum anti-nuclear anti bodies test
2. Positive serum RF
3. Anti CCP antibodies
4. Increase serum C3 and decrease synovial C#
5. Increase total serum proteins ie increase gamma globulins (chronic inflammation)
6. X-ray shows narrow joint space and fusion

Q-10 A 50 years old lady presents with achy pain in her hip & knee joints that worsens with use
during the passage of the day. On examination she has a crepitus in her knee joint & limited
range of movement. On X-ray prominent osteophytes are visible at the upper end of tibia.
a) What is the most likely diagnosis? 2
b) Briefly describe the pathogenesis of the above condition. 4
c) Briefly discuss any four morphological features observed in the affected joint in the above
condition.

ANS:
a) OSTEOARTHRITIS
b) PATHOGENESIS:
1. Increase degradation and decrease synthesis of matrix due to genetic factors
including polymorphisms and mutations in genes encoding components of the
matrix and signaling molecules
2. Advancing age there is an imbalance in the expression, activity, and signaling of
cytokines and growth factors that results in degradation and loss of matrix
3. The type II collagen network also is diminished, presumably as a result of
decreased local synthesis and increased breakdown; chondrocyte apoptosis is
increased.
c) MORPHOLOGY:
1. Subsequently, vertical and horizontal fibrillation and cracking of the matrix
occur as the superficial layers of the cartilage are degraded

KMU Past Papers Solved SEQs


Endocrine System, Breast, Bones, Joints and CNS 129

2. articular cartilage surface appears as soft granular-appearing, a condition known


as chondromalacia.
3. bone eburnation when full-thickness portions of the cartilage are lost, and the
subchondral bone plate is exposed and is smoothened and burnished by friction,
giving it the appearance of polished ivory.
4. The underlying cancellous bone becomes rebuttressed by osteoblastic activity.
5. joint mice are present in joint spaces which are Small fractures can dislodge
pieces of cartilage and subchondral bone into the joint, forming loose bodies
6. Subchondral cyst when the synovial fluid is forced into the subchondral regions to
form fibrous walled cysts.
7. Mushroom-shaped osteophytes (bony outgrowths) develop at the margins of the
articular surface.
8. In severe disease, a fibrous synovial pannus covers the peripheral portions of the
articular surface.

Q-11
a) Enumerate tumors arising from bone forming cell.
b) Give the salient features of osteosarcoma.
c) Write down the criteria for diagnosis of Rheumatoid arthritis

ANS:
a) TUMOR ARISE FORM BONE CELLS:
1. Benign
i) Osteoma
ii) Osteoid osteoma
iii) Osteoblastoma
2. Malignant
i) Primary osteosarcoma
ii) Secondary osteosarcoma
b) SALIENT FEATURES OF OSTEOSARCOMA:
1. Osteosarcoma is a bone-producing malignant mesenchymal tumor
2. most tumors arise in the metaphyseal region of the long bones of the extremities, with
almost 60% occurring about the knee, 15% around the hip, 10% at the shoulder, and
8% in the jaw
3. two types medullary and cortical types.
4. osteosarcomas also frequently exhibit mutations in TP53, RB gene and genes that
regulate the cell cycle, including cyclins, cyclin dependent kinases, and kinase
inhibitors
5. grossly osteosarcomas are gritty-appearing, gray-white tumors
6. microscopically the Tumor cells vary in size and shape and frequently have large
hyperchromatic nuclei, mitotic figures and bizarre tumor giant cells are present.
7. A triangular shadow on the x-ray film between the cortex and raised periosteum
(Codman triangle) is characteristic of osteosarcomas
c) GIVEN SOMEWHERE ELSE**

KMU Past Papers Solved SEQs


Endocrine System, Breast, Bones, Joints and CNS 130

Q-12
a) What are the common causes of hyperthyroidism?
b) Name major subtypes of thyroid carcinoma. "Orphan Annie Nuclei" are a feature of which
kind of carcinoma thyroid?
c) What are the levels of T 3, T4 & TSH in case of Hashimoto's thyroiditis?

ANS:
a) CAUSES OF HYPERTHYROIDISM:
i) Primary:
1. Diffuse toxic hyperplasia (Graves disease)
2. Hyperfunctioning (“toxic”) multinodular goiter
3. Hyperfunctioning (“toxic”) adenoma
4. Iodine-induced hyperthyroidism
ii) Secondary
1. TSH-secreting pituitary adenoma (rare)
b) THYROID CARCINOMA SUBTYPES:
i) Papillary carcinoma (85%)
ii) Follicular carcinoma (5% to 15%)
iii) Anaplastic (undifferentiated) carcinoma (less than 5%)
iv) Medullary carcinoma (5%)
Orphan Annie eye is feature of papillary carcinoma.
c) HASHIMOTO’S THYROIDITIS TFTS
T4 and T3 levels are decrease
TSH level is increased (compensatory)
In (hashitoxicosis) reverse occur, increase in T4 and T3 while TSH is decreased

Q-13
a) Classify autoimmune diseases of thyroid gland.
b) How will you diagnose Grave's disease in the laboratory?
c) Enumerate the pituitary gland hormones.

ANS:
a) AUTOIMMUNE THYROID GLAND DISEASES
1. Grave’s disease
2. Hashimoto’s thyroiditis
3. Subacute Lymphocytic Thyroiditis (postpartum thyroiditis)
4. Riedel thyroiditis
b) GRAVE’S DIAGNOSIS
1. Elevated serum free T4 and T3
2. Decreased serum TSH
3. Radioactive iodine uptake is increased
4. radioiodine scans show a diffuse uptake of iodine.
c) PITUITARY GLAND HORMONES
A. Anterior pituitary hormone
1. Growth hormone

KMU Past Papers Solved SEQs


Endocrine System, Breast, Bones, Joints and CNS 131

2. Follicular stimulating hormone


3. Luteinizing hormone
4. ACTH
5. Thyroid stimulating hormone
6. Prolactin
B. Posterior pituitary hormones
1. Oxytocin
2. Antidiuretic hormone

Q-14
a) Define thyrotoxicosis.
b) What are the disorders associated with thyrotoxicosis?
c) Briefly explain the pathogenesis of Graves' disease.

ANS:
a) THYROTOXICOSIS:
Thyrotoxicosis is a hyper-metabolic state due to elevated circulating levels of free T3 and
T4 which may be due to hyperfunction of the thyroid gland (referred to as
hyperthyroidism), oversupply either due to release of pre-formed thyroid hormone
(thyroiditis) or comes from an extrathyroidal soure i.e. tumors (stuma overi)
b) DISORDERS ASSOCIATED
1. Diffuse toxic hyperplasia (Graves’ disease)
2. Toxic multinodular goiter
3. toxic adenoma
4. Iodine-induced hyperthyroidism
5. TSH-secreting pituitary adenoma
6. Granulomatous (de Quervain) thyroiditis
7. Subacute lymphocytic thyroiditis
8. Struma ovarii (ovarian teratoma with thyroid)
9. Factitious thyrotoxicosis (exogenous thyroxine intake)
c) PATHOGENESIS OF GRAVES’S DISEASE
Graves’ disease is characterized by a breakdown in self-tolerance to thyroid
autoantigens, of which the most important is the TSH receptor.
Following antibodies are produced.
1. Thyroid-stimulating immunoglobulin:
An IgG antibody that binds to the TSH receptor and mimics the action of TSH,
stimulating adenyl cyclase, with resultant increased release of thyroid hormones.
2. Thyroid growth-stimulating immunoglobulins:
Also directed against the TSH receptor, these antibodies have been implicated in
the proliferation of thyroid follicular epithelium.
3. TSH-binding inhibitor immunoglobulins:
These anti-TSH receptor antibodies prevent TSH from binding to its receptor on
thyroid epithelial cells and in so doing may actually inhibit thyroid cell function.
The coexistence of stimulating and inhibiting immunoglobulins in the serum of the
same patient is not unusual a finding that may explain why some patients with
Graves’ disease spontaneously develop episodes of hypothyroidism.

Infiltrative ophthalmopathy:
1. marked infiltration of the retroorbital space by mononuclear cells, predominantly
T cells;

KMU Past Papers Solved SEQs


Endocrine System, Breast, Bones, Joints and CNS 132

2. inflammatory edema and swelling of extraocular muscles;


3. accumulation of extracellular matrix components, specifically hydrophilic
glycosaminoglycans such as hyaluronic acid and chondroitin sulfate;
4. increased numbers of adipocytes (fatty infiltration).
5. These changes displace the eyeball forward, potentially interfering with the
function of the extraocular muscles

Q-15
a) Enumerate causes of Hypothyroidism
b) Define gravis disease and what the characteristics triad manifestations of this condition
are.
c) What abnormality is detected in TFT

ANS:
a) CAUSES OF HYPOTHYROIDISM:
i) Primary:
1. Rare developmental abnormalities (thyroid dysgenesis) mutations in PAX8,
FOXE1
2. Congenital biosynthetic defect (dyshormonogenetic goiter)
3. Postablative Surgery, radioiodine therapy, or external irradiation
4. Autoimmune hypothyroidism
5. Hashimoto thyroiditis
6. Iodine deficiency
7. Drugs (lithium, iodides, p-aminosalicylic acid)
ii) Secondary (Central)
1. Pituitary failure
2. Hypothalamic failure (rare)
b) GRAVE’S DISEASE:
Grave’s disease is an autoimmune disease which is associated with enlargement of the
thyroid gland and the most common cause of endogenous hyperthyroidism.
1. Thyrotoxicosis, caused by a diffusely enlarged, hyperfunctional thyroid, is present
in all cases.
2. Ophthalmopathy with resultant exophthalmos is noted in as many as 40% of
patients.
3. Dermopathy (sometimes designated pretibial myxedema) is seen in a minority of
cases.
c) TFT OF GRAVE’S DISEASE ARE GIVEN ABOVE**

Q-16
a) Write down only four common causes of solitary nodule of thyroid
b) What is the nature in all probabilities, of the nodule that takes up radioactive iodine
imaging studies?
c) Simply name different major sub types of carcinoma thyroid
d) Which subtype of carcinoma thyroid is the most common?
e) Which one carries excellent prognosis in the absence of extra thyroid lesion?
f) From which cells of the thyroid gland arises medullary subtype?

ANS:
a) COMMON CAUSES OF SOLITARY NODULE OF THYROID:

KMU Past Papers Solved SEQs


Endocrine System, Breast, Bones, Joints and CNS 133

1. follicular adenomas
2. dominant nodule in multinodular goiter
3. simple cysts
4. foci of thyroiditis
b) Nodules that take up radioactive iodine in imaging studies (hot nodules) are more likely
to be benign than malignant, reflecting well-differentiated cells.
c) CARCINOMA THYROID:
1. Papillary carcinoma (85%)
2. Follicular carcinoma (5% to 15%)
3. Anaplastic (undifferentiated) carcinoma (less than 5%)
4. Medullary carcinoma (5% of cases)
d) PAPILLARY CARCINOMA
e) PAPILLARY CARCINOMA
f) Para follicular cells (C cells) of thyroid

Q-17
a) Define Grave’s disease
b) What are its common causes?
c) Give brief account on its pathogenesis

ANS:
a) GRAVE’S DISEASE:
Grave’s disease is an autoimmune disease which is associated with enlargement of the
thyroid gland and the most common cause of endogenous hyperthyroidism.
b) COMMON CAUSES
1. As with other autoimmune disorders, a genetic susceptibility to Grave’s disease is
associated with the presence of certain human leukocyte antigen (HLA)
haplotypes, specifically HLA-DR3,
2. polymorphisms in genes encoding the inhibitory T cell receptor CTLA-4
3. tyrosine phosphatase PTPN22.
4. Other factors are female gender, emotional stress and smoking
c) PATHOGENESIS
Given somewhere else**

Q-18
a) Classify pituitary adenoma
b) Write own only three most common causes of thyrotoxicosis

ANS:
a) PITUITARY ADENOMA CLASSIFICATION:
i) On the basis of hormone production
1. Prolactinoma
2. Growth Hormone–Producing (Somatotroph Cell) Adenomas
3. Adrenocorticotropic Hormone–Producing (Corticotroph Cell) Adenomas
4. Gonadotroph (LH and FSH–producing) adenomas
5. Thyrotroph (TSH–producing) adenomas
ii) On the basis of functionality
1. Functional adenomas

KMU Past Papers Solved SEQs


Endocrine System, Breast, Bones, Joints and CNS 134

2. Non-functional adenoma
3. Hormone negative adenoma
iii) On the basis of occurrence
1. Hereditary
2. Sporadic
iv) On the basis of size
1. Microadenoma; diameter > 1 cm
2. Macroadenoma; diameter < 1 cm
b) CAUSES OF THYROTOXICOSIS.
1. Diffuse toxic hyperplasia/ Grave’s disease
2. Hyperfunctioning/toxic multinodular goiter
3. Hyperfunctioning/toxic adenoma

Q-19
a) Enumerate the different types of thyroiditis. 2
b) Give the gross and microscopic appearance of hashimoto’s thyroiditis. 4+4

ANS:
a) DIFFERENT TYPES OF THYROIDITIS:
i) Acute thyroiditis
1. Bacterial infection e.g. Staphylococcus, Streptococcus.
2. Fungal infection e.g. Aspergillus, Histoplasma, Pneumocystis
3. Radiation injury.
ii) Subacute thyroiditis
1. Subacute granulomatous thyroiditis (de Quervain’s thyroiditis, giant cell
thyroiditis, viral thyroiditis)
2. Subacute lymphocytic (postpartum, silent) thyroiditis
3. Tuberculous thyroiditis.
iii) Chronic thyroiditis
1. Autoimmune thyroiditis (Hashimoto’s thyroiditis or chronic lymphocytic
thyroiditis)
2. Riedel’s thyroiditis (or invasive fibrous thyroiditis).
b) MORPHOLOGY OF HASHIMOTO’S THYROIDITIS:
 Gross:
1. The thyroid usually is diffusely and symmetrically firm and rubbery enlarged,
although more localized enlargement may be seen in some cases.
2. The cut surface is pale and gray-tan in appearance, and the tissue is firm and
somewhat friable.
 Microscopy:
1. Widespread infiltration of the parenchyma by a mononuclear inflammatory
infiltrate containing small lymphocytes, plasma cells, and well-developed germinal
centers
2. The thyroid follicles are atrophic and are often devoid of colloid.
3. Follicles are lined in many areas by epithelial cells distinguished by the presence
of abundant eosinophilic, granular cytoplasm, termed hürthle, or oxyphil, cells.
(also called askanazy cells, or oxyphil cells, or oncocytes)
4. This is a metaplastic response of the normally low cuboidal follicular epithelium to
ongoing injury;

KMU Past Papers Solved SEQs


Endocrine System, Breast, Bones, Joints and CNS 135

5. Ultrastructural examination, the hürthle cells are characterized by numerous


prominent mitochondria.
6. Interstitial connective tissue is increased and may be abundant.
 Variants:
1. Fibrosing variant; the thyroid is small and atrophic as a result of more extensive
fibrosis which does NOT extend beyond the capsule of the gland.

Q-20 A young adult of 20-year age is brought to hospital with history of severe headache fever
and vomiting. Physical examination reveals stiff neck and positive kering’s sign lumbar
puncture reveals cloudy CSF with raised neutrophil count elevated protein and non-
detectable glucose level.
a) What is the diagnosis of this patient illness?
b) Enumerate the causative agent of this illness in different age group.
c) Which causative agent is responsible for causing outbreak of this disease?
d) How can you confirm the causative agent of outbreak by simple staining method of CSF?

ANS:
a) ACUTE BACTERIAL MENINGITIS
b) CAUSATIVE AGENTS:
1. Escherichia coli and group B streptococci  neonates and with neural tube defects.
2. Haemophilus influenzae  infants and children.
3. Neisseria meningitidis  adolescent and young adults and epidemic meningitis.
4. Streptococcus pneumoniae and Listeria monocytogenes  extremes of age and
following trauma.
c) NEISSERIA MENINGITIDIS.
d) LAB DIAGNOSIS OF MENINGITIDIS
1. Gram negative cocci on gram staining
2. Growth on chocolate ager
3. Oxidase positive
4. Latex agglutination test

Q-21
a) Define Meningitis.
b) Enumerate the causes of increased intracranial pressure.
c) Give the CSF findings in Acute Pyogenic Meningitis.

ANS:
a) MENINGITIS:
Meningitis is an inflammatory process involving the leptomeninges (pia and arachnoid
matter) within the subarachnoid space.
b) CAUSES OF INCREASE INTRACRANIAL PRESSURE:
1. Hydrocephalus
2. Cerebral edema
3. Pseudotumor cerebri (idiopathic intracranial hypertension)
4. Acute hypertensive encephalopathy
5. Acute meningitis
6. Acute epidural hematoma
7. Subarachnoid hemorrhage

KMU Past Papers Solved SEQs


Endocrine System, Breast, Bones, Joints and CNS 136

8. Brain tumors
9. Brain Abscesses
10. Straining at stool or sexual intercourse.
c) CSF FINDINGS IN ACUTE PYOGENIC MENINGITIS:
1. Naked eye appearance of cloudy or frankly purulent CSF
2. Elevated CSF pressure (above 180 mm water).
3. Polymorpho-nuclear neutrophilic leukocytosis in CSF (between 10-10,000/µl).
4. Raised CSF protein level (higher than 50 mg/dl).
5. Decreased CSF sugar concentration (lower than 40 mg/dl).
6. Bacteriologic examination by Gram’s stain or by CSF culture reveals causative
organism.

ANS:
Q-22 A 20-year-old female presented to emergency department with 3 days history of fever,
headache and irrelevant talk. On examination she was totally disoriented in time and space
neck stiffness was positive
a) What is the most likely diagnosis?
b) What are the common causative agents?
c) What laboratory findings will help you in this case?

a) ACUTE MENINGITIS
b) COMMON CAUSATIVE AGENTS:
Given above**
c) LAB FINDINGS:
Given above**

Q-23
a) Define meningitis and meningoencephalitis
b) Describe the morphology of tuberculous meningitis.
c) Tabulate differences between pyogenic and tuberculous meningitis.

ANS:
a) MENINGITIS:
Meningitis is an inflammatory process involving the leptomeninges (pia and arachnoid
matter) within the subarachnoid space.
Meningoencephalitis: if the infection spreads into the underlying brain it is termed
meningoencephalitis
b) MORPHOLOGY OF TUBERCULOUS MENINGITIS:
 Gross:
1. The subarachnoid space contains a gelatinous or fibrinous exudate, most often at
the base of the brain,
2. Obliterating the cisterns and encasing cranial nerves.
3. There may be discrete white granules scattered over the leptomeninges.
4. Arteries running through the subarachnoid space may show obliterative
endarteritis with inflammatory infiltrates and marked intimal thickening.
 Microscopic: on examination there are

KMU Past Papers Solved SEQs


Endocrine System, Breast, Bones, Joints and CNS 137

1. Mixtures of lymphocytes, plasma cells, and macrophages.


2. Florid cases show well-formed granulomas,
3. Often with caseous necrosis and giant cells, similar to the lesions of tuberculosis
elsewhere.
c) DIFFERENCES BETWEEN PYOGENIC AND TUBERCULOUS MENINGITIS:
Pyogenic meningitis Tuberculous meningitis
Acute Chronic
Neutrophil leukocytosis polymorphonuclear and mononuclear cells
Glucose level is reduced Glucose is reduced or normal
Not present circumscribed intraparenchymal mass
(tuberculoma)

Q-24
a) Define meningitis.
b) What is the etiology of acute pyogenic meningitis?
c) Enumerate laboratory investigations for acute pyogenic meningitis.

ANS: Given somewhere else**

Q-25 Answer the following question regarding primary brain tumors


a) Name the most commonly occurring primary brain tumor 1
b) It constitutes roughly which percentage of adult primary brain tumors. 1
c) Name different types of this common primary brain tumor. 2+2
d) Which one of the primary brain tumors carries the worst prognosis? 2
e) Which one of the primary brain tumors is very slow growing and permit long survival? 2

ANS:
a) GLIOMAS
b) 50-60%
c) TYPES OF GLIOMAS:
1. Astrocytoma
2. Oligodendroglioma
3. Ependymoma
4. Choroid plexus papilloma
d) GLIOBLASTOMA MULTIFORM
e) PILIOCYTIC ASTROCYTOMA

Q-26 A 4-year child presents with a space occupying lesion in the cerebellum. Histological
examination shows a Poorly-differentiated neoplasm with marked cellularity and sheets of
small round blue cells.
a) What is the most likely diagnosis? 2
b) How does the tumor spread? And what is its most ominous complication. 8

ANS:
a) MEDULLOBLASTOMA:
b) SPREAD OF TUMOR:
It invades locally and by the CSF to meninges, ventricles and subarachnoid space

KMU Past Papers Solved SEQs


Endocrine System, Breast, Bones, Joints and CNS 138

widespread metastases to extra neural sites such as to lungs, liver, vertebrae and pelvis.
Complications:
1. Increase intracranial pressure
2. Seizures
3. Focal sign and symptoms of cerebellar lesion ie
a. Loss of muscle coordination
b. Vertigo
c. Ataxia

Q-27 A 41 years old female came to the GP with the complaints of easy fatigability, weakness,
decreased memory & accumulation of fat in the posterior neck & back. On examination blood
pressure was raised & abdominal striae were visible. On laboratory evaluation Serum cortisol
was raised.
a) What is the most likely diagnosis? 1
b) Give any four causes of the above condition. 2
c) Briefly discuss the morphological changes in the glands which may occur in the above
mentioned condition. 7

ANS:
a) CUSHING SYNDROME
b) CAUSE:
1. Iatrogenic excess of glucocorticoids
2. Primary adrenal neoplasm or hyperplasia
3. ACTH producing pituitary adenoma
4. ACTH producing non-pituitary adenoma
c) MORPHOLOGY OF CUSHING SYNDROME
The main lesions of Cushing syndrome are found in the pituitary and adrenal glands.
THE PITUITARY:
1. Crooke hyaline change, resulting from high levels of endogenous or exogenous
glucocorticoids
2. the normal granular, basophilic cytoplasm of the ACTH-producing cells in the
anterior pituitary is replaced by homogeneous, lightly basophilic material.
3. This alteration is the result of the accumulation of inter mediate keratin filaments
in the cytoplasm.
THE ADRENAL GLANDS:
1. cortical atrophy
i) In patients in whom the syndrome results from exogenous glucocorticoids
ii) suppression of endogenous ACTH results in bilateral cortical atrophy,
iii) due to a lack of stimulation of the zona fasciculata and zona reticularis by
ACTH.
iv) The zona glomerulosa is of normal thickness (independent of ACTH)
2. diffuse hyperplasia
i) Diffuse hyperplasia is found in patients with ACTH dependent Cushing
syndrome
ii) Both glands are enlarged, either subtly or markedly
iii) The adrenal cortex is diffusely thickened
iv) The yellow color of diffusely hyperplastic glands derives from presence of
lipid-rich cells, which appear vacuolated under the microscope
3. macronodular or micronodular hyperplasia
i) occur in primary cortical hyperplasia,

KMU Past Papers Solved SEQs


Endocrine System, Breast, Bones, Joints and CNS 139

ii) the cortex is replaced almost entirely by darkly pigmented nodules.


iii) The pigment is believed to be lipofuscin, a wear-and-tear pigment.
4. an adenoma or a carcinoma.
i) Adrenocortical adenomas yellow tumors surrounded by thin or well developed
capsules
On microscopic examination, they are composed of cells similar to those
encountered in the normal zona fasciculate.
ii) The carcinomas tend to be larger than the adenomas. These tumors are non -
encapsulated masses, having all of the anaplastic characteristics of cancer, the
adjacent adrenal cortex and that of the contralateral adrenal gland are
iii) atrophic, as a result of suppression of endogenous ACTH by high cortisol levels.

Q-28
a) Classify types of pituitary adenoma and the hormones released by those adenomas. 3
b) Which one is the most common pituitary adenoma? 1
c) Name any two clinical manifestations of the most common pituitary adenoma. 1
d) Give any six causes of hypopituitarism. 3
e) Classify & define the two types of diabetes insipidus. 2

ANS:
a) GIVEN SOMEWHERE ELSE**
b) PROLACTINOMA
c) CLINICAL MANIFESTATIONS:
1. Amenorrhea
2. Galactorrhea
3. Loss of libido
4. Infertility.
d) CAUSES OF HYPOPITUITARISM:
1. Nonfunctioning pituitary adenomas
2. Metastatic carcinoma
3. Craniopharyngioma
4. Ischemic necrosis of the anterior pituitary
a. Sheehan syndrome; postpartum pituitary necrosis in females
b. Simmond’s disease; occur in males without preceding pregnancy
c. Disseminated intravascular coagulation
d. Sickle cell anemia
e. Elevated intracranial pressure
f. Traumatic injury
g. Shock of any origin
5. Empty Sella syndrome
6. Ablation of the pituitary by surgery or irradiation
7. Sarcoidosis
8. Tuberculosis
9. Trauma
e) DIABETES INSIPIDUS:
a condition characterized by excessive urination (polyuria) caused by an inability of the
kidney to properly resorb water from the urine.

KMU Past Papers Solved SEQs


Endocrine System, Breast, Bones, Joints and CNS 140

Types
1. Central diabetes insipidus: result from ADH deficiency.
2. Nephrogenic Diabetes insipidus: result from renal tubular unresponsiveness to
circulating ADH

Q-29
a) Define Diabetes.
b) Enumerate tests advised for diagnosis of diabetes mellitus.
c) Write about glucose tolerance test.

ANS:
a) DIABETES:
As per the WHO, diabetes mellitus (DM) is defined as a heterogeneous metabolic disorder
characterized by common feature of chronic hyperglycemia with disturbance of
carbohydrate fat and protein metabolism.
b) DIAGNOSTIC TESTS FOR DM
1. Urine testing: for the presence of glucose and ketones.
2. A random blood glucose concentration:
> 200 mg/dL  diabetic
3. A fasting glucose concentration
< 110 mg/dL,  euglycemic
>110 but < 126 mg/dL  pr-ediabetic
> 126mg/dL  diabetic
4. Oral glucose tolerance test (OGTT)
< 140 mg/dL  euglycemic
> 140 but < 200 mg/dL  pre-diabetic
> 200 mg/dL  diabetic
5. Glycosylated hemoglobin (HbA1C)
6. Glycated albumin
7. Insulin, proinsulin and C peptide assay
c) GLUCOSE TOLERANCE TEST.
The patient who is scheduled for oral GTT is instructed to eat a high carbohydrate diet for
at least 3 days prior to the test and come after an overnight fast on the day of the test (for
at least 8 hours). A fasting blood sugar sample is first drawn. Then 75 gm of glucose
dissolved in 300 ml of water is given. Blood and urine specimen are collected at half-
hourly intervals for at least 2 hours. Blood or plasma glucose content is measured and
urine is tested for glucosuria to determine the approximate renal threshold for glucose.
Venous whole blood concentrations are 15% lower than plasma glucose values.

Q-30
a) Give the lab diagnosis of Diabetes Mellitus.
b) Describe the complications of Diabetes Mellitus

ANS:
a) GIVEN ABOVE**
b) COMPLICATIONS OF DM:
i) Acute metabolic complications:
1. diabetic ketoacidosis (type 1 DM)

KMU Past Papers Solved SEQs


Endocrine System, Breast, Bones, Joints and CNS 141

It can develop in patients with severe insulin deficiency combined with glucagon
excess.
Severe lack of insulin  lipolysis in the adipose tissues  release of free fatty
acids into the plasma  taken up by the liver  oxidized through acetyl
coenzyme-A to ketone bodies  rate of ketogenesis exceeds the rate at which the
ketone bodies can be utilized by the muscles and other tissues  ketonaemia and
ketonuria occur
If urinary excretion is prevented due to dehydration, systemic metabolic
ketoacidosis occurs ie anorexia, nausea, vomiting, deep and fast breathing, mental
confusion and coma.
2. hyperosmolar nonketotic coma (Type 2 DM)
sustained hyperglycemic diuresis  severe dehydration loss of electrolytes 
diabetic coma
3. hypoglycemia (type 1 DM)
excessive administration of insulin, missing a meal, or due to stress
ii) Late systemic complications:
1. atherosclerosis,
hyperlipidemia, reduced HDL levels, nonenzymatic glycosylation, increased
platelet adhesive ness, obesity and associated hypertension in diabetes.
2. diabetic microangiopathy,
recurrent hyper glycaemia  increased glycosylation collagen and basement
membrane material  thickening of basement membrane
3. diabetic nephropathy,
a. Diabetic glomerulosclerosis which includes diffuse and nodular lesions of
glomerulosclerosis.
b. Vascular lesions that include hyaline arteriolosclerosis of afferent and
efferent arterioles and atheromas of renal arteries.
c. Diabetic pyelonephritis and necrotising renal papillitis.
d. Tubular lesions or Armanni-Ebstein lesion.
4. diabetic neuropathy,
due to diffuse microangiopathy
increase in intracellular glucose  sorbitol by the enzyme aldose reductase and
eventually to fructose, in a reaction that uses NADPH. Decrease NADPH 
decrease GSH activity one of the important antioxidant mechanisms  increases
cellular susceptibility to oxidative stress.
5. diabetic retinopathy
background and proliferative retinopathy
cataract and glaucoma
6. infections
impaired leucocyte functions, reduced cellular immunity, poor blood supply due to
vascular involvement and hyper glycaemia.

Q-31 A 50 years old woman consulted her family doctor because of tiredness, general body
aches and weight gain. On questioning she admitted to feeling thirst and had notice that she
had been passing more urine than normal.
a) What is diagnosis?
b) What lab investigations are recommended?
c) What are major characteristics of type I and type II diabetes mellitus?

ANS:

KMU Past Papers Solved SEQs


Endocrine System, Breast, Bones, Joints and CNS 142

a) DIABETES MELLITUS
b) GIVEN ABOVE**
c) DIFFERENCES

Q-32
a) How do you diagnose Diabetes Mellitus in laboratory? 3
b) Tabulate characteristics of type I and type Il Diabetic Mellitus. 7

ANS: GIVEN ABOVE**

Q-33
a) Give the classification of Diabetes Mellitus 6
b) Enumerate the acute complication of diabetes. 4

ANS:

KMU Past Papers Solved SEQs


Endocrine System, Breast, Bones, Joints and CNS 143

a) CLASSIFICATION OF DIABETES MELLITUS:

B) GIVEN ABOVE**

Q-34 During 4th year MBBS class a student falls in class rank from first place to last place. She
has also noted a lower pitch to her voice and coarsening of her hair along with increased
tendency towards weight gain, menorrhagia and increased intolerance to cold.
a) What is the diagnosis? 2
b) What would be the expected laboratory abnormalities? 3
c) What will be the microscopic findings in such disease? 5

ANS:
a) HASHIMOTOS THYROIDITIS:
b) LABORATORY ABNORMALITIES:
1. Serum TSH will be increased
2. Serum T3 and T4 will be decreased
3. Decrease radioactive iodine uptake
c) MICROSCOPIC FINDING
1. Infiltration of parenchyma by mononuclear inflammatory infiltrate
2. Well-developed germinal centers
3. Atrophied follicles
4. Hurthles cells or oxiphils cells lines the atrophied follicles
5. Hurthles cells have numerous predominant mitochondria.
6. Increased interstitial tissue when fibrosis is extinctive this is called fibrosing var

KMU Past Papers Solved SEQs

Você também pode gostar